TOP 20 đề thi HSG Toán 6 cấp Huyện (có đáp án)

TOP 20 đề thi HSG Toán 6 cấp Huyện có đáp án. Tài liệu được biên soạn dưới dạng file PDF bao gồm 75 trang tổng hợp các kiến thức chọn lọc giúp các bạn tham khảo, ôn tập và đạt kết quả cao trong kỳ thi sắp tới. Mời các bạn đón xem!

Trang 1
PHÒNG GIÁO DỤC VÀ ĐÀO TẠO
ĐỀ KHẢO SÁT HỌC SINH GIỎI LỚP 6 CẤP HUYỆN
Môn: Toán Lớp 6
Thời gian làm bài: 120 phút (Không kể thời gian giao đề)
ĐỀ 1
i 1: (1,0điểm) Thực hiện phép tính( tính hợp lý nếu có th )
a/ 1968 : 16 + 5136 : 16 -704 : 16
b/ 2
3
. 5
3
- 3 {400 -[ 673 - 2
3.
(7
8
: 7
6
+7
0
)]}
i 2: ( 1,0điểm) M có là một số chính phương không nếu :
M = 1 + 3 + 5 +…+ (2n-1) ( Với n
N , n
0 )
i 3: (1,5điểm) Chứng tỏ rằng:
a/ (3
100
+19
990
) 2
b / Tổng của 4 số tự nhiên liên tiếp kng chia hết cho 4
i 4 : (1,0điểm) So sánh A và B biết :
A =
117
117
19
18
+
+
, B =
117
117
18
17
+
+
i 5: ( 2,0điểm ) Tím tất cả c số nguyên n để:
a) Phân s
1
2
n
n
+
có giá tr một số nguyên
b) Phân số
230
112
+
+
n
n
phân số ti giản
i 6: (2,5điểm)
Cho góc xBy = 55
0
.Trên các tia Bx, By lần lượt lấy các điểm A, C ( A
B, C
B ). Trên
đon thẳng AC lấy điểm D sao cho góc ABD = 30
0
a/ Tính độ dài AC, biết AD = 4cm, CD = 3cm
b/ Tính số đo c DBC
c/ Từ B vẽ tia Bz sao cho góc DBz = 90
0
. nh số đo ABz.
i 7: (1,0điểm) Tìm các cặp số tự nhiên x , y sao cho : (2x + 1)( y 5) = 12
---------- HẾT ----------
(Đề thi gồm có 01 trang).
Thí sinh không được sử dụng tài liệu. Cán bộ coi thi không giải thích gì thêm.
Họ và n thí sinh:.......................................; Số báodanh.........................
Trang 2
PHÒNG GIÁO DỤC VÀ ĐÀO TẠO
ĐỀ KHẢO SÁT HỌC SINH GIỎI LỚP 6 CẤP HUYỆN
Môn: Toán Lớp 6
Thời gian làm bài: 120 phút (Không kể thời gian giao đề)
HƯỚNG DẪN CHM ĐỀ 1
i 1: (1,0 điểm)
Ý/Phần
Đáp án
Điểm
a
= 16(123+ 321 - 44):16
0,25
= 400
0,25
b
=8.125-3.{400-[673-8.50]}
0,25
= 1000-3.{400-273}
=619
0,25
i 2: (1,0 điểm)
Ý/Phần
Đáp án
Điểm
M = 1 + 3 + 5 +…+ (2n-1) ( Với n
N , n
0 )
Tính số số hạng = ( 2n-1-1): 2 + 1 = n
0,5
Tính tng = ( 2n-1+1 ) n : 2 = 2n
2
: 2 = n
2
KL: M là số chính pơng
0,5đ
i 3: (1,5 điểm)
Ý/Phần
Đáp án
Điểm
a
Ta có:
3
100
= 3.3.3….3 (có 100 thừa số 3)
= (3
4
)
25
= 81
25
có chữ số tận cùng bằng 1
19
990
= 19.19…19 ( có 990 thứa số 19 )
= (19
2
)
495
= 361
495
( có chữ số tận cùng bằng 1
Vậy 3
100
+19
990
có chữ số tận cùng bằng 2 nên tổngy chia
hết cho 2
0,25
0,25
0,5
b
Gọi 4 số tự nhiên liên tiếp : a ; (a +1) ;( a + 2) ;( a + 3 ) ; ( a
N
)
Ta có : a + (a+1) + (a+2) + (a+3) = 4a + 6
4a 4 ; 6 không chia hết 4 nên 4a+ 6 kng chia hết 4
0,25
0,25
i 4 : ( 1,0 điểm)
Ý/Phần
Đáp án
Điểm
Vì A =
117
117
19
18
+
+
< 1
A=
117
117
19
18
+
+
<
16117
16117
19
18
++
++
=
0,75
0,25
Trang 3
)
(
)
(
11717
11717
18
17
+
+
=
117
117
18
17
+
+
= B
Vậy A < B
i 5: (2,0 điểm)
Ý/Phần
Đáp án
Điểm
a
1
2
n
n
+
số nguyên khi ( n+1)
(n-2)
Ta có (n+1) =
( 2) 3n−+
Vậy (n+1)
(n-2) khi 3
(n-2)
(n-2)
Ư(3) =
3; 1;1;3−−
=> n
1;1;3;5
0.5
0,5
b
Gọi d là ƯC của 12n+1 và 30n+2 ( d
N
*
)
dndn 230,112 ++
0,25
dnn )230(2)112(5 ++
(60n+5-60n-4)
d
1
d mà d
N
*
d = 1
0,5đ
Vậy phân số đã cho tối giản
0,25
i 6: (2,5 điểm)
Ý/Phần
Đáp án
Điểm
a
b
Vẽ hình đúng
TH1 TH2
Vì D thuộc đoạn thẳng ACn D nằm giữa A và C :
AC= AD + CD = 4+3 = 7 cm
Chng minh được tia BD nằm giữa hai tia BA và BC
Ta có đẳng thức :
ABC =
ABD +
DBC
DBC =
ABC -
ABD
=55
0
30
0
= 25
0
0,25
0,25
0,25
0,25
0,5
c
Xét hai trường hợp:
- Trường hợp 1: Tia Bz và tia BD nằm về hai phía nửa mặt phẳng
có bờ là AB nên tia BA nằm giữa hai tiaBz và BD
0,25
z
D
C
A
y
x
B
z
D
C
A
y
x
B
Trang 4
Tính được
ABz = 90
0
-
ABD = 90
0
- 30
0
= 60
0
- Trường hợp 2 :Tia Bz và tia BD nằm về cùng nửa mặt phng có
bờ AB nên tia BD nằm giữa hai tia Bz và BA
Tính đưc
ABz = 90
0
+
ABD = 90
0
+ 30
0
= 120
0
0,25
0,25
0,25
i 7: (1,0 điểm)
Ý/Phần
Đáp án
Điểm
(2x+ 1); (y - 5) là c ước ca 12
0,25
Ư(12) =
12;6;4;3;2;1
0,25
Vì 2x + 1 lẻ nên :
2x + 1= 1
x=0 , y =17
2x + 1= 3
x=1 , y=9
Vậy với x = 0 thì y = 17 ; Với x = 1 ty = 9
0,25
0,25
PHÒNG GIÁO DỤC VÀ ĐÀO TẠO
ĐỀ KHẢO SÁT HỌC SINH GIỎI LỚP 6 CẤP HUYỆN
Môn: Toán Lớp 6
Thời gian làm bài: 120 phút (Không kể thời gian giao đề)
ĐỀ 2
i 1: (5,0 điểm) . Cho
50 444 6 4 248 6
A 5 5 5 +5 - 5 55 1. += + +
a) Tính A.
b) Tìm số tự nhiên n biết
n
26.A 1 5+=
c) Tìm số dư trong phép chia A cho 100.
i 2: (3,0 điểm). Tìm số tự nhn x ,biết:
( )
1 3 5 7 9 2 1 225) + + + + ++ =ax
x x 1 x 2 x 3 x 2015 2019
b) 2 2 2 2 +2 2 .8
+ + + +
+ + + + =
i 3: (5,0 điểm)
a) Cho số
abc
chia hết cho 37. Chứng minh rằng số
cab
cũng chia hết cho 37.
b) Tìm số x, y nguyên biết
x.y 12 x y+ = +
i 4 (3,0 điểm): Tìm số tự nhiên a nhnhất sao cho: a chia cho 2 dư 1, a chia cho 3 dư 1, a chia
cho 5 dư 4, a chia cho 7 dư 3.
i 5: (4,0 điểm)
1. Cho 30 điểm phân biệt trong đó có a điểm thẳng hàng, cứ qua 2 điểm ta vẽ được 1 đường
thẳng. Tìm a, biết số đường thẳng tạo thành là 421 đường thẳng.
2. Vẽ đoạn thẳng
AB 6cm=
. Lấy hai điểm C và D nằm giữa A và B sao cho
AC BD 9cm.+=
a) Chứng tỏ D nằm giữa A và C.
b) Tính độ dài đoạn thẳng CD ?
--- Hết ---
Trang 5
PHÒNG GIÁO DỤC VÀ ĐÀO TẠO
ĐỀ KHẢO SÁT HỌC SINH GIỎI LỚP 6 CẤP HUYỆN
Môn: Toán Lớp 6
Thời gian làm bài: 120 phút (Không kể thời gian giao đề)
HƯỚNG DẪN CHM ĐỀ 2
i 1: (4,0 điểm)
Đáp án
Điểm
50 444 6 4 28 46
a. A 5 5 5 +5 - 5 5 15 + .= + +
( )
48 46 44 62 450 2
525A 5 . 5 5 5 +5 - 5 +.51= + +
0,25
5052 46 4 248 8 6
5 5 5 5 +5 - . 5 5 5+ = + +
0,25
Suy ra
52
25A A 5 1+ =
0,50
Vậy
( )
52
A 5 1 : 26=−
0,25
b) Tìm s tự nhiên n biết
n
26.A 1 5+=
Ta có
n
26.A 1 5+=
52
26A 5 1=−
nên
52 n
5 1 1 5 + =
0,25
Suy ra
52 n
5 5 n 52= =
.Vậy
n 52=
0,25
c). Tìm s dư trong phép chia A cho 100.
50 4448 46 6 4 2
A 5 5 5 +5 - 5 55.1+= + +
( có 26 số hạng)
0,25
( ) ( ) ( )
50 4448 46 6 4 2
5 5 5 + 5 - 5 55.1+= + +
0,25
( ) ( ) ( ) ( )
50 4448 46 6 4 2
5 5 5 + 5 - 5 55.1+= + +
0,25
( ) ( ) ( ) ( )
2 44 248 4 22
5 5 1 5 1 +5 5 1. 5 . . + 5 .1= + +
0,25
48 444
.24 5 .24 5 +5 .24+ 24. = + +
0,25
42426
.25.24 5 .25. 5 +524 .25.24+ . 24= + +
0,50
( )
46 2 44 62 42 2
.600 5 .600 .600+ . 6.100. 5 +5 24 5 .. 25 45.= + + +=+ ++
0,25
Suy ra A chia cho 100 dư 24.
0,25
i 2: (3,0 điểm). Tìm số tự nhiên x ,biết:
Đáp án
Điểm
( )
a) 1 3 5 7 9 2 1 225+ + + + ++ =x
Với mọi x N ta có 2x 1 là số lẻ
0,25
Đặt
( )
A = 1 + 3 + 5 + 7 + 9 ++ 2x1
A là tổng của các số lẻ liên tiếp từ 1 đến 2x 1
0,25
Số số hạng của A là:
( )
2x 11 : 2 1 x+=
(Số hạng)
0,25
( )
2
A 2x 1 1 .x : 2 x = + =


0,25
22
A 225 x 225 15= = =
0,25
x 15=
Vậy
x 15=
0,25
Trang 6
x x 1 x 2 x 3 x 2015 2019
.b) 2 2 2 2 +2 2 8
+ + + +
+ + + + =
x x x 2 x 3 x 2015 2019 3
2 .1 2 .2 2 .2 2 .2 +2 .2 2 2 .+ + + + =
0,25
( ) ( )
x 2 3 2015 3 2016
2 . 1 2 2 2 +2 2 1 ..2+ + + + =
0,25
Đặt
2 3 2015
M 1 2 2 2 +2= + + + +
0,25
Ta được
2 3 4 2016
2.M 2 2 2 2 +2= + + +
Suy ra
2016
M 2 1=−
0,25
Vậy ta
( ) ( )
x 2016 3 2016
2 . 2 1 2 . 2 1 . =
0,25
x3
2 2 x 3 = =
.Vậy
x3=
0,25
i 3: (5,0 điểm).
Đáp án
Điểm
a) Cho số
abc
chia hết cho 37. Chứng minh rng s
cab
cũng chia hết cho 37.
Ta có
abc 37 100.abc 37 abc00 37
0,50
( )
ab .1000 c00 37+
0,25
( )
ab .999 c00 ab 37

+ +

0,25
( )
ab .999 cab 37+
0,25
ab .999 ab .37.27 37=
0,25
cab 37
0,25
Vậy nếu
abc 37
thì
cab 37
0,25
b) Tìm số x, y ngun biết
x.y 12 x y+ = +
Ta có
x.y 12 x y x.y x y 12 0+ = + + =
0,25
( )
x. y 1 y 12 0 + =
0,25
( ) ( )
x. y 1 y 1 11 0 + =
0,25
( ) ( ) ( )
x 1 . y 1 11 1 =
0,25
x,y Z
nên
x 1 Z; y 1 Z
0,25
Do đó từ
( )
1 x 1; y 1
các ước ca -11
0,25
Các ước của -11 là -11; -1;1;11
0,25
+) Với
x 1 11 =
thì
y 1 1.−=
Suy ra
x 10; y = 2=−
( Thỏa mãn)
0,25
+) Với
x 1 1 =
thì
y 1 11.−=
Suy ra
x 0; y = 12=
( Thỏa mãn)
0,25
+) Với
x 1 1−=
thì
y 1 11. =
Suy ra
x 2; y = -10=
( Thỏa mãn)
0,25
+) Với
x 1 11−=
thì
y 1 1. =
Suy ra
x 12; y = 0=
( Thỏa mãn)
0,25
Vậy
( ) ( ) ( ) ( ) ( )
x;y 10;2 ; 0;12 ; 2; 10 ; 12;0 .
0,25
i 4: (3,0 điểm).
Trang 7
Đáp án
Điểm
Vì a chia cho 2 dư 1, a chia cho 31, a chia cho 5 dư 4, a chia cho 73
0,25
Nên
a 1 2 ;a 1 3 ; a 4 5 ; a 3 7
a 1 2 ;a 2 3 ; a 1 5 ; a 4 7 + + + +
0,25
a 11 2 ;a 11 3 ; a 11 5 ; a 11 7 + + + +
0,50
( )
a 11 BC 2;3;5;7 . +
0,25
Mà a là số tự nhiên nhỏ nhất
0,25
( )
a 11 BCNN 2;3;5;7 . + =
0,25
Mà các số 2; 3; 5; 7 nguyên tố ng nhau
0,25
( )
BCNN 2;3;5;7 2.3.5.7 210 = =
0,25
a 11 210. + =
0,25
a 199.=
0,25
Vậy số tự nhiên cần tìm là 199.
0,25
i 5: (4,0 điểm)
Đáp án
Điểm
1. Giả sử trong 30 điểm phân biệt không có 3 điểm nào thẳng hàng :
0,25
+ Chọn một điểm bất kì trong 30 điểm đã cho. Qua điểm đó và từng điểm trong
29 điểm n lại ta vẽ được 29 đường thẳng.
+ Làm như vậy với 30 điểm thì ta vẽ đưc tất clà 29.30 đường thẳng.
0,25
+ Nhưng mỗi đường thẳng đã được tính hai lần nên số đưng thẳng thực tế vẽ
được
( )
29.30 : 2 435=
đường thẳng.
0,25
Vậy qua 30 điểm phân biệt mà không có 3 điểm nào thẳng hàng ta vẽ được 435
đường thẳng.
Tương tự như tn, giả sử trong a điểm phân biệt không có 3 điểm nào thẳng
hàng ta vẽ đưc
( )
a. a 1 :2
đường thẳng.
0,25
Nhưng qua a điểm thẳng hàng ta chỉ vẽ được một đường thẳng nên sđường thẳng
bị giảm đi là
( )
a. a 1 :2 1−−
đường thẳng.
0,25
Theo bài ra ta có :
( )
a. a 1 :2 1 435 421 14 = =
0,25
( )
a. a 1 30 6.5 = =
0,25
Vì a-1 và a hai số tự nhiên liên tiếp
a 1 a−
nên
a 6.=
0,25
a) Chứng tỏ D nằm giữa A và C.
Vì D nằm giữa A và B nên:
AD DB AB+=
0,25
Thay
AB 6 cm=
ta
AD DB 6 cm+=
.
0,25
Lại có
AC DB 9 cm+=
AD DB AC DB + +
hay
AD AC.
0,25
Trên tia AB có :
AD AC
suy ra D nằm giữa A và C
0,25
b) Tính độ dài đoạn thẳng CD ?
Vì D nằm giữa A và C suy ra
AD DC AC.+=
0,25
Lại có
AC DB 9 cm+=
, suy ra
AD DC DB 9cm+ + =
0,25
Hay
( )
AD DB DC 9cm+ + =
0,25
B
C
D
A
2. Hình
v :
Trang 8
Thay
AD DB 6 cm+=
, ta có
( )
6cm DC 9 cm+=
. Vậy
( )
DC 3 cm=
0,25
Chú ý: Học sinh giải theo cách khác mà đúng thì vẫn cho điểm tương ứng với từng câu, từng bài
theo hướng dẫn trên./.
PHÒNG GIÁO DỤC VÀ ĐÀO TẠO
ĐỀ KHẢO SÁT HỌC SINH GIỎI LỚP 6 CẤP HUYỆN
Môn: Toán Lớp 6
Thời gian làm bài: 120 phút (Không kể thời gian giao đề)
ĐỀ 3
i 1 (5 điểm)
1): Rút gọn các biểu thức sau:
M = 3 3
2
+ 3
3
3
4
+ … + 3
2015
3
2016
.
2) Chứng tỏ rằng:
a)
2 2 2 2 2
1 1 1 1 1 3
...
2 3 4 99 100 4
+ + + + +
i 2 (3 điểm): m số tự nhiên x biết:
a) 1 + 3 + 5 + 7 + 9 + … + (2x – 1 ) = 225
b) 2
x
. 2
x + 1
. 2
x + 2
= 1000 … 0 : 5
18
18 chữ số 0
i 3: (5 điểm)
a) Cho 3a + 2b 17 (a , b N). Chứng minh 10a + b 17
b) Tìm số x,y nguyên biết xy + x y = 4
i 4: (4 điểm)
Cho 30 điểm phân biệt trong đó có a điểm thẳng hàng cứ qua 2 điểm ta vđưc 1 đường thẳng.
Tìm a, biết số đường thẳng tạo thành là 421 đường thẳng .
i 5 (3 điểm)
Tìm số tự nhn a nhỏ nhất sao cho: a chia cho 2 dư 1, a chia cho 3 dư 1, a chia cho 5 dư 4, a chia
cho 7 dư 3.
= Hết =
PHÒNG GIÁO DỤC VÀ ĐÀO TẠO
ĐỀ KHẢO SÁT HỌC SINH GIỎI LỚP 6 CẤP HUYỆN
Môn: Toán Lớp 6
Thời gian làm bài: 120 phút (Không kể thời gian giao đề)
HƯỚNG DẪN CHM ĐỀ 3
Bài
Nội dung
Điểm
Bài 1 (5 điểm)
1.1.a)
M = 3 3
2
+ 3
3
3
4
+ + 3
2015
3
2016
Ta có :3M = 3
2
3
3
+ 3
4
3
5
+ + 3
2016
3
2017
0,5
Trang 9
(2,0 đ)
3M + M = 3 + (3
2
3
2
) + (3
3
3
3
)+ + (3
2016
3
2016
) 3
2017
0,5
4M = 3 + 0 + 0 + . . . + 0 3
2017
0,5
4M = 3 3
2017
M = (3 3
2017
) : 4
0,5
1.2.
(3,0 đ)
Ta có:
2
11
3 2.3
2
11
4 3.4
2
11
5 4.5
. . . . . . .
2
11
99 98.99
2
11
100 99.100
0,5
2 2 2 2 2
1 1 1 1 1 1 1 1 1 1 1
... ...
2 3 4 99 100 4 2.3 3.4 4.5 98.99 99.100
+ + + + + + + + + + +
0,5
1 1 1 1 1 1
...
4 2.3 3.4 4.5 98.99 99.100
+ + + + + +
=
1 1 1 1 1 1 1 1 1 1 1
...
4 2 3 3 4 4 5 98 99 99 100
+ + + + + +
0,5
=
1 1 1
4 2 100
+−
0,5
=
3 1 3
4 100 4
−
0,5
2 2 2 2 2
1 1 1 1 1 3
...
2 3 4 99 100 4
+ + + + +
0,5
Bài 2 (3 điểm)
2.a)
(1,5)
Với mọi x N ta có 2x 1 là số lẻ
0,25
Đặt A = 1 + 3 + 5 + 7 + 9 + + (2x 1)
A là tổng của các số lẻ liên tiếp từ 1 đến 2x 1
0,25
Số số hạng của A là: (2x 1 1) : 2 + 1 = x (Số hạng)
0,25
A = [(2x 1) + 1] . x : 2 = x
2
0,25
Mà A = 225 x
2
= 225 = 15
2
0,25
x = 15
Vậy x = 15
0,25
2.b)
(1,5 đ)
2
x
. 2
x + 1
. 2
x + 2
= 1000 0 : 5
18
18 chữ số 0
2
x + x + 1+ x + 2
= 10 . 10 . 10 . . 10 : 5 . 5 . 5 . . 5
18 thừa số 10 18 thừa số 5
0,25
Trang 10
2
3x + 3
= (10 : 5 ).(10 : 5). (10 : 5) . .(10 : 5)
18 thừa số (10 : 5)
0,25
2
3x + 3
= 2 . 2 . 2 . . 2
18 thừa số 2
0,25
2
3x + 3
= 2
18
0,25
3x + 3 = 18
3x = 18 3
3x = 15
x = 15 : 3
x = 5
0,25
Vậy x = 5
0,25
Bài 3: (5 điểm)
3.a)
(2,0đ)
Vì 3a + 2b 17 10(3a + 2b) 17
0,5
(30a + 20b) 17
0,25
(30a + 3b + 17b) 17
0,25
[3(10a + b) + 17b] 17
0,25
Vì 17b 17
0,25
3(10a + b) 17
0,25
10a + b 17 (vì 3 và 17 nguyên tố cùng nhau)
0,25
3.b)
(3,0đ)
xy + x y = 4
x(y + 1) y = 4
0,25
x(y + 1) y 1 + 1 = 4
0,5
x(y + 1) - ( y + 1 ) + 1= 4
0,25
( y + 1 )( x 1) + 1 = 4
0,25
( y + 1)( x 1)= 3
0,25
Vì x, y là số nguyên nên y + 1,x 1 là ước của 3.
0,25
Nếu x -1 = 1 và y + 1 = 3 thì x = 2 và y = 2
0,25
Nếu x -1 = -1 và y + 1 = -3 thì x = 0 và y = -4
0,25
Nếu x -1 = 3 và y + 1 = 1 thì x = 4 và y = 0
0,25
Nếu x -1 = -3 và y + 1 = -1 thì x = -2 và y = -2
0,25
Vậy x = 2 và y = 2 hoặc x = 0 và y = -4 hoặc x = 4 và y = 0 hoặc
x = -2 và y = -2
0,25
Bài 4( 4điểm)
4.
(4 đ)
Giả sử trong 20 điểm không có 3 điểm nào thẳng hàng.
Gọi 20 điểm đó là A
1
, A
2
, A
3
, ... ,A
20
.
Vì cứ qua 2 điểm ta vẽ được 1 đường thẳng nên
0,25
Qua điểm A
1
và từng điểm trong 19 điểm còn lại A
2
, A
3
,...,A
20
ta
vẽ được 19 đường thẳng.
0,5
Qua điểm A
2
và từng điểm trong 18 điểm còn lại A
3
, A
4
,...,A
20
ta
vẽ được 18 đường thẳng.
0,5
… ….
Qua điểm A
19
và điểm A
20
ta vẽ được 1 đường thẳng.
0,5
Trang 11
Do đó số đường thẳng tạo thành là: 1 + 2 + 3 +... + 19 + 20 =
( 1+ 20).20 : 2 = 190 ( đường thẳng)
0,5
Với a điểm trong đó không 3 điểm nào thẳng hàng tta số
đường thẳng tạo thành là 1 + 2 + 3 +... +( a 1) = (a- 1). a: 2
0,5
Với a điểm thẳng hàng thì ta chỉ vẽ được 1 đường thẳng
0,25
Vậy trong 20 điểm có a diểm thẳng hàng thì sổ đưởng thẳng
giảm đi
( a- 1).a: 2 - 1 = 190 170
0,5
( a- 1).a: 2 - 1 = 20
( a- 1).a: 2 = 21
0,25
( a- 1).a = 42
( a- 1).a = 6.7
Mà a-1 và a là 2 số tự nhiên liên tiếp a -1 < a nên a -1 = 6 và a =7
Vậy a = 7
0,25
5.
(3,0 đ)
Gọi số phải tìm là a
a = 2k + 1
a = 3q + 1
a = 5m + 4
a = 7r + 3
(k, q, m, r N)
0,5
a + 11 = 2k + 12 2
a + 11 = 3q + 12 3
a + 11 = 5m + 15 5
a + 11 = 7r + 14 7
a + 11 BC(2; 3; 5; 7)
1,0
Mà a là số tự nhiên nhỏ nhất
a + 11 = BCNN(2; 3; 5; 7)
0,5
Mà 2; 3; 5; 7 nguyên tố cùng nhau
BCNN(2; 3; 5; 7) = 2.3.5.7 = 210
0,25
a + 11 = 210
a = 210 11
a = 199
0,5
Vậy a = 199
0,25
Trang 12
PHÒNG GIÁO DỤC VÀ ĐÀO TẠO
ĐỀ KHẢO SÁT HỌC SINH GIỎI LỚP 6 CẤP HUYỆN
Môn: Toán Lớp 6
Thời gian làm bài: 120 phút (Không kể thời gian giao đề)
ĐỀ 4
i 1 : (5 điểm) Thực hiện các phép tính sau một cách hợp lý :
a)
( ) ( )
2 2 2 2 2
10 11 12 : 13 14+ + +
.
b)
2
1.2.3...9 1.2.3...8 1.2.3...7.8−−
c)
( )
2
16
13 11 9
3.4.2
11.2 .4 16
d) 1152 - (374 + 1152) + (-65 + 374)
e) 13 - 12 + 11 + 10 - 9 + 8 - 7 - 6 + 5 - 4 + 3 + 2 - 1
i 2 : (4 điểm) m x, biết:
a)
( )
( )
2
22
19x 2.5 :14 13 8 4+ =
b)
( ) ( ) ( )
x x 1 x 2 ... x 30 1240+ + + + + + + =
c) 11 - (-53 + x) = 97
d) -(x + 84) + 213 = -16
i 3 :(2 điểm) m hai số tự nhiên a và b, biết: BCNN(a,b)=300; ƯCLN(a,b)=15 và a+15=b.
i 4 :(3 điểm)
a)Tìm số nguyên x và y, biết : xy - x + 2y = 3.
b) So sánh M và N biết rằng :
102
103
101 1
M
101 1
+
=
+
.
103
104
101 1
N
101 1
+
=
+
.
i 5 : (6 điểm) Cho đoạn thẳngAB, điểm O thuộc tia đối của tia AB. Gọi M, N thứ tự trung
điểm của OA, OB.
a) Chứng tỏ rằng OA < OB.
b) Trong ba điểm O, M, N điểm nào nằm giữa hai điểm còn lại ?
c) Chứng tỏ rằng độ dài đoạn thẳng MN không phụ thuộc vào vị trí của điểm O (O thuộc
tia đối của tia AB).
PHÒNG GIÁO DỤC VÀ ĐÀO TẠO
ĐỀ KHẢO SÁT HỌC SINH GIỎI LỚP 6 CẤP HUYỆN
Môn: Toán Lớp 6
Thời gian làm bài: 120 phút (Không kể thời gian giao đề)
HƯỚNG DẪN CHM ĐỀ 7
Bài 1 : (5 điểm) Thực hiện các phép tính sau một cách hợp :
Đáp án
Điểm
( ) ( )
( ) ( )
2 2 2 2 2
a) 10 11 12 : 13 14 100 121 144 : 169 196
365:365 1
+ + + = + + +
==
1
Trang 13
( )
2
b) 1.2.3...9 1.2.3...8 1.2.3...7.8 1.2.3...7.8. 9 1 8 1.2.3...7.8..0 0 = = =
1
( ) ( )
( ) ( )
( )
( )
2 2 2
16 2 16 2 18
11 9
13 11 9 13 22 36
13 2 4
2 36 2 36 2 36 2
13 22 36 35 36 35
3.4.2 3.2 .2 3 . 2
c)
11.2 .4 16 11.2 .2 2
11.2 . 2 2
3 .2 3 .2 3 .2 3 .2
2
11.2 .2 2 11.2 2 2 11 2 9
==
−−
= = = = =
1
d) 1152 - (374 + 1152) + (-65 + 374) = 1152 - 374 - 1152 + (-65) + 374
= (1152 - 1152) + (-65) + (374 - 374) = -65
1
e) 13 - 12 + 11 + 10 - 9 + 8 - 7 - 6 + 5 - 4 + 3 + 2 - 1 =
= 13 - (12 - 11 - 10 + 9) + (8 - 7 - 6 + 5) - (4 - 3 - 2 + 1) = 13
1
Bài 2 : (4 điểm) m x :
Câu
Đáp án
Điểm
a.
( )
( )
2
22
19x 2.5 :14 13 8 4+ =
( )
2
22
x 14. 13 8 4 2.5 :19
x4

=

=
1
b.
( ) ( ) ( )
x x 1 x 2 ... x 30 1240+ + + + + + + =
( )
( )
31 So hang
x x ... x 1 2 ... 30 1240
30. 1 30
31x 1240
2
31x 1240 31.15
775
x 25
31

+ + + + + + + =


+
+ =
=
= =
1
c.
11 - (-53 + x) = 97
x 11 97 ( 53) 33 = =
1
d.
-(x + 84) + 213 = -16
(x 84) 16 213
(x 84) 229
x 84 229
x 229 84 145
+ =
+ =
+ =
= =
1
Bài 3 :(3 điểm)
Đáp án
Điểm
Từ dữ liệu đề bài cho, ta có :
+ Vì ƯCLN(a, b) = 15, nên ắt tồn tại các số tự nhiên m và n khác 0, sao cho:
a = 15m; b = 15n (1)
ƯCLN(m, n) = 1 (2)
+ Vì BCNN(a, b) = 300, nên theo trên, ta suy ra :
3
Trang 14
( )
( )
BCNN 15m; 15n 300 15.20
BCNN m; n 20 (3)
= =
=
+ Vì a + 15 = b, nên theo trên, ta suy ra :
15m 15 15n + =
( )
15. m 1 15n m 1 n (4) + = + =
Trong c trường hợp thoả mãn c điều kiện (2) và (3), thì chỉ trường hợp : m
= 4, n = 5 là thoả mãn điều kiện (4).
Vậy với m = 4, n = 5, ta được các số phải tìm là : a = 15 . 4 = 60; b = 15 . 5 = 75
Bài 4 :(2 điểm)
Câu
Đáp án
Điểm
a.
Chứng minh đẳng thức:
- (-a + b + c) + (b + c - 1) = (b - c + 6) - (7 - a + b) + c.
Biến đổi vế trái của đẳng thức, ta được :
VT = -(-a + b + c) + (b + c - 1)
= -(-a) - (b + c) + (b + c) + (-1) = a - 1
Biến đổi vế phải của đẳng thức, ta được :
VP = (b - c + 6) - (7 - a + b) + c
= b + (-c) + 6 - 7 + a - b + c = [b + (-b)] + [(-c) + c] + a + [6 + (-7)] = a -
1
So sánh, ta thấy : VT = VP = a - 1
Vậy đẳng thức đã được chứng minh.
1
b.
Với a > b và S = -(-a - b - c) + (-c + b + a) - (a + b), ta có :
( ) ( ) ( )
S a b c c b a a b
S ( a b)+c ( c) (b a) (a b) S ( a b) a b
= + + + +
= + + + + = = +
Tính
S
: theo trên ta suy ra :
S a b = +
* Xét với a và b cùng dấu, ta có các trường hợp sau xảy ra :
+ a và b cùng dương, hay a > b > 0, thì a + b > 0 :
S a b a b = + = +
+ a và b cùng âm, hay 0 > a > b, thì a + b < 0
(a b) 0 +
, nên suy ra :
( ) ( )
S a b a b a b = + = + = +
* Xét với a và b khác dấu :
a > b, nên suy ra : a > 0 và b < 0
b0
, ta cần xét các trường hợp
sau xảy ra :
+
ab
,hay a > -b > 0, do đó
a b a ( b) 0+ =
, suy ra:
S a b a b = + = +
+
ab
, hay -b > a > 0, do đó
a b a ( b) 0+ =
, hay
( )
a b 0 +
suy ra
:
S a b (a b) a ( b) = + = + = +
Vậy, với : +
S a b=+
(nếu
b
< a < 0)
+
( )
S a b= +
(nếu b < a < 0, hoặc b < 0 <
ab
)
1
Bài 5 : (6 điểm)
Câu
Đáp án
Điểm
Hình
vẽ
b
m
n
a
o
Trang 15
a.
Hai tia AO, AB đối nhau, nên điểm A nằm giữa hai điểm O B, suy
ra :
OA < OB.
2
b.
Ta có M và N thứ tự là trung điểm của OA, OB, nên :
OA OB
OM ; ON
22
= =
Vì OA < OB, nên OM < ON.
Hai điểm M và N thuộc tia OB, OM < ON, nên điểm M nằm giữa
hai điểm O và N.
2
c.
Vì điểm M nm giữa hai điểm O và N, nên ta có :
OM MN ON + =
suy ra :
MN ON OM =
hay :
OB OA AB
MN
22
= =
Vì AB có đdài kng đổi, nên MN có độ dài không đổi, hay độ dài
đoạn thẳng MN kng phụ thuộc vào vị trí của điểm O (O thuộc tia
đối của tia AB).
2
PHÒNG GIÁO DỤC VÀ ĐÀO TẠO
ĐỀ KHẢO SÁT HỌC SINH GIỎI LỚP 6 CẤP HUYỆN
Môn: Toán Lớp 6
Thời gian làm bài: 120 phút (Không kể thời gian giao đề)
ĐỀ 5
Câu 1(3,0 điểm): nh giá trị của các biểu thức sau:
a.
42
2 .5 [131 (13 4) ]
b.
3 28.43 28.5 28.21
5 5.56 5.24 5.63
+ +
Câu 2(4,0 điểm): Tìm các số nguyên x biết.
a.
3
5 24 5
.
3 35 6
x




b.
32
(7 11) ( 3) .15 208x = +
c.
2 7 20 5.( 3)x = +
Câu 3(5,0 điểm):
a, Một số tự nhiên chia cho 7 dư 5,chia cho 13 dư 4. Nếu đem sđó chia cho 91 thì
dư bao nhiêu?
b, Học sinh khối 6 khi xếp hàng; nếu xếp hàng 10, hàng 12, hàng15 đều 3 học
sinh. Nhưng khi xếp hàng 11 thì a đủ. Biết số học sinh khối 6 chưa đến 400 học
sinh.Tính số học sinh khối 6?
Câu 4 (6,0 điểm):
Cho góc bẹt xOy. Trên cùng một nửa mặt phẳng có bờ xy,vẽ các tia Oz và Ot sao
cho
00
70 ; 55xOz yOt==
.
a. Chứng tỏ tia Oz nằm giữa hai tia Ox và Ot ?
b. Chứng tỏ tia Ot là tia phân giác của góc yOz?
c.Vẽ tia phân giác On của góc xOz. Tínhc nOt?
Câu 5 (2,0 điểm):
Cho n là số nguyên tố lớn hơn 3. Hỏi n
2
+ 2006 là số nguyên tố hay là hợp số.
---------Hết---------
Trang 16
PHÒNG GIÁO DỤC VÀ ĐÀO TẠO
ĐỀ KHẢO SÁT HỌC SINH GIỎI LỚP 6 CẤP HUYỆN
Môn: Toán Lớp 6
Thời gian làm bài: 120 phút (Không kể thời gian giao đề)
HƯỚNG DẪN CHM ĐỀ 5
Câu
Nội dung
Thang
điểm
Câu
1(4điểm)
a (1,5)
2
16.5 (131 9 )
80 50
30
=
=−
=
0.5
0.5
0.5
b (1,5)
3 28 43 5 1
.( )
5 5 56 24 3
3 28 129 35 56
.( )
5 5 168 168 168
3 28 108
.
5 5 168
3 18
55
3
+ +
= + +
=+
=+
=
0.5
0.5
0,25
0.25
câu 2
(4điểm)
a (1,0)
0.5
0.5
b (1,5)
32
3
33
(7 11) ( 3) .15 208
(7 11) 9.15 208
(7 11) 7
18
7 11 7
7
x
x
x
xx
= +
= +
−=
= =
(không thỏa mãn)
0.5
0.5
0.5
c (1,5)
2 7 20 5.( 3)
2 7 5
2 7 5 2 12 6
[ [ [
2 7 5 2 2 1
x
x
x x x
x x x
= +
−=
= = =
=− = =
Vậy
1;6x
0.5
0.5
0.5
Câu3(4,0)
a (2,0)
Gọi số đó là a
Vì a chia cho 7 dư 5, chia cho 13 dư 4
9 7; 9 13aa + +
mà (7,13)=1 nên
9 7.13a +
a+9=91k
a=91k-9 =91k-91+82=91(k-1)+82 (k
N)
Vậy a chia cho 91 dư 82.
0.25
1.0
1.0
0.25
b (2,0)
Gọi số Hs khối 6 là a (3<a<400)
0.25
Trang 17
Vì khi xếp hàng 10,hàng 12, hàng 15 đều dư 3
3 10;12;15a
3 (10,12,15)a BC−
ta có
BCNN(10,12,15)=60
3 60;120;180;240;300;360;420;....a−
63;123;183;243;303;363;423;...a
11; 400aa
a=363
Vậy số HS khối 6 là 363 học sinh.
0.5
0.5
0.75
0.5
Câu 4
(6,0)
Vẽ hình
z t
n
x O y
0.5
a (1,5)
Vì góc xOy là góc bẹt nên suy ra trên cùng một
nưả mặt phẳng có bờ xy có
xOt
và
tOy
là hai góc kề bù.
xOt
+
tOy
=
0
180
0
00
180 55 125xOt xOt= =
Trên cùng một nửa mặt phẳng có bchứa tia Ox có:
00
(70 125 )xOz xOt
Tia Oz nằm giữa hai tia Ox và Ot.
0.75
0.75
b (2,0)
Trên cùng một nửa mặt phẳng có bxy ,ta có
xOz
zOy
là hai góc kề bù
0
180xOz zOy + =
hay
0 0 0 0 0
70 180 180 70 110zOy zOy+ = = =
Trên cùng một nửa mặt phẳng có bchứa tia Oy có:
00
(55 110 )yOt yOz
Tia Ot nằm giữa hai tia Oy và Oz (1)
nên ta có:
yOt tOz yOz+=
hay
0 0 0 0 0
55 110 110 55 55tOz tOz+ = = =
0
( 55 )yOt tOz = =
(2).Từ (1) và (2) suy ra Ot là tia phân giác
của góc yOz.
0.75
0.75
0.5
Trang 18
c (2,0)
xOy
là góc bẹt nên suy ra tia Ox và tia Oy là hai tia đối
nhau
Hai tia Ox và Oy nằm trên hai nửa mặt phẳng đối nhau
có bờ chứa tia Oz (1)
Vì On là tia phân giác của góc xOz nên
0
0
70
35
22
xOz
nOz = = =
hai tia On và Ox cùng nằm trên mặt phẳng có bờ chứa tia Oz
(2)
Ta lại có tia Ot là tia phân giác của góc yOz (theo b,)
Hai tia Ot và Oy cùng nằm trên một nửa mặt phẳng có b
chứa tia Oz (3) . Từ (1),(2), (3) suy ra tia On và tia Ot nằm
trên hai nửa mặt phẳng đối nhau có bờ chứa tia Oz
tia Oz
nằm giữa hai tia On và Ot nên ta có:
nOz zOt nOt+=
hay
0 0 0
35 55 90nOt = + =
.Vậy
0
90nOt =
0.5
0.5
0.5
0.5
Câu 5
(2,0)
n là số nguyên tố, n > 3 nên n không chia hết cho 3.
Vậy n
2
chia hết cho 3 dư 1
do đó n
2
+ 2006 = 3m + 1 + 2006
= 3m+2007
= 3( m+669) chia hết cho 3.
Vậy n
2
+ 2006 là hợp số.
0.5
0.5
0.75
0.25
Trang 19
PHÒNG GIÁO DỤC VÀ ĐÀO TẠO
ĐỀ KHẢO SÁT HỌC SINH GIỎI LỚP 6 CẤP HUYỆN
Môn: Toán Lớp 6
Thời gian làm bài: 120 phút (Không kể thời gian giao đề)
ĐỀ 6
Câu 1. (4 điểm)
a) Thực hiện phép tính: A = 81.
12 12 12 5 5 5
12 5
158158158
7 289 85 13 169 91
:.
4 4 4 6 6 6
711711711
46
7 289 85 13 169 91

+ + +



+ + +

b) Tìm x biết: 1) -
2 1 1
( ) (2 1)
3 4 3
xx =
2)
1 7 8
1 1 1 1
.2 .2 .2 .2
5 3 5 3
xx+
+ = +
c. T×m hai sè tù nhiªn a vµ b biÕt tæng BCNN vµ ¦CLN cña chóng lµ 15
d. Tìm x nguyên thỏa mãn:
1 2 7 5 10x x x x+ + + + =
Câu 2. (4 điểm)
a. Thực hiện phép nh:
2 2 9 2 6 2 14 4
28 18 29 18
5.(2 .3 ) .(2 ) 2.(2 .3) .3
A
5.2 .3 7.2 .3
=
b. Tìm các số nguyên n sao cho: n
2
+ 5n + 9 là bội của n + 3
c. Chứng minh rằng bình phương của một số nguyên tố khác 2 và 3 khi chia cho 12 đều dư 1
d. Tìm x, y nguyên sao cho: xy + 2x + y + 11 = 0
Câu 3. (4 điểm)
a) Tìm số tự nhiên nhỏ nhất sao cho khi chia cho 11 dư 6, chia cho 4 1và chia cho 19 dư 11.
b) Tìm 3 sốtổng bằng 210, biết rằng
7
6
số thứ nhất bằng
11
9
số thứ 2 và
11
9
số thứ 2 bằng
3
2
số thứ 3.
c. Tìm số tự nhiên a, b, c, d nhỏ nhất sao cho:
15 9 9
;;
21 12 11
a b c
b c d
= = =
d. Tìm hai số biết tỉ số của chúng bằng 5 : 8 và tích của chúng bằng 360.
Câu 4. (5 điểm)
1. a) Cho đoạn thẳng AB dài 7cm. Trên tia AB lấy điểm I sao cho AI = 4 cm. Trên tia BA lấy điểm K sao
cho BK = 2 cm.
Hãy chứng tỏ rằng I nằm giữa A và K. Tính IK.
b) Trên tia Ox cho 4 điểm A, B, C, D. biết rằng A nằm giữa B và C; B nằm giữa C và D ; OA = 5cm; OD
= 2 cm ; BC = 4 cm và độ dài AC gấp đôi độ dài BD. Tìm độ dài các đoạn BD; AC.
2. Trªn n÷a mÆt ph¼ng cho trc cã bê Ox vÏ hai tia Oy vµ Oz sao cho sè ®o
xOy = 70
0
vµ sè ®o
yOz = 30
0
.
a) X¸c ®Þnh ®o cña
xOz
b) Trªn tia Ox lÊy 2 ®iÓm A vµ B (§iÓm A kh«ng trïng víi ®iÓm O vµ ®é dµi OB lín h¬n ®é dµi
OA).
Gäi M lµ trung ®iÓm cña OA. H·y so s¸nh ®é dµi MB víi trung b×nh céng ®é dµi OB vµ AB.
Câu 5. ( 3 điểm)
a. Chứng minh rằng: 3
2
+ 3
3
+ 3
4
+……+ 3
101
chia hết cho 120.
b. Cho hai số a và b thỏa mãn: a b = 2(a + b) =
a
b
Trang 20
Chứng minh a = -3b ; Tính
a
b
; Tìm a và b
c. Tìm x, y, z biết: ( x – y
2
+ z)
2
+ ( y 2)
2
+ ( z +3)
2
= 0
Giám thị coi thi không gii thích gì thêm - SBD:.......................
Trang 21
PHÒNG GIÁO DỤC VÀ ĐÀO TẠO
ĐỀ KHẢO SÁT HỌC SINH GIỎI LỚP 6 CẤP HUYỆN
Môn: Toán Lớp 6
Thời gian làm bài: 120 phút (Không kể thời gian giao đề)
HƯỚNG DẪN CHM ĐỀ 6
Câu
Phần
Nội dung
Điểm
Câu 1
(4 điểm)
a
Ta có: .
12 12 12 5 5 5
12 5
158158158
7 289 85 13 169 91
81. : .
4 4 4 6 6 6
711711711
46
7 289 85 13 169 91
A

+ + +

=


+ + +

1 1 1 1 1 1
12 1 5 1
158.1001001
7 289 85 13 169 91
81. : .
1 1 1 1 1 1
711.1001001
4 1 6 1
7 289 85 13 169 91

+ + +


=

+ + +


12 5 158
81. : .
4 6 711

=


18 2 324
81. .
5 9 5
==
1
0,5
0,5
b
(x + 1) + ( x + 2 ) + . . . . . . . . + (x + 100) = 5750
=> x + 1 + x + 2 + x + 3 + . . . . . . .. . .. . . . + x + 100 = 5750
=> ( 1 + 2 + 3 + . . . + 100) + ( x + x + x . . . . . . . + x ) = 5750
101 . 50 + 100 x = 5750
100 x + 5050 = 5750
100 x = 5750 5050
100 x = 700
x = 7
0.5
0.5
0.5
0.5
Câu 2
( 4 điểm )
a
Ta có:
2 2 9 2 6 2 14 4
28 18 29 18
5.(2 .3 ) .(2 ) 2.(2 .3) .3
A
5.2 .3 7.2 .3
=
18 18 12 28 14 4
28 18 29 18
5.2 .3 .2 2.2 .3 .3
5.2 .3 7.2 .3
=
30 18 29 18
28 18
5.2 .3 2 .3
2 .3 (5 7.2)
=
29 18
28 18
2 .3 (5.2 1) 2.9
2
2 .3 (5 14) 9
= = =
−−
0.5
0.5
1
b
S =(3)
0
+(3)
1
+ (3)
2
+(3)
3
+...+ (3)
2015
.
3S = (3).[(3)
0
+(3)
1
+(3)
2
+ ....+(3)
2015
]
= (3)
1
+ (3)
2
+ ....+(3)
2016
]
0,5
0,5
Trang 22
3S S = [(3)
1
+ (3)
2
+...+(3)
2016
] - (3)
0
-(3)
1
-...-(3)
2015
.
2S = (3)
2016
-1.
S =
2016
(3) 1
2
0,5
0,5
Câu 3
(4 điểm)
a
Gọi số cần tìm là a ta có: (a-6) 11 ;(a-1) 4; (a-11) 19.
(a-6 +33) 11 ; (a-1 + 28) 4 ; (a-11 +38 ) 19.
(a +27) 11 ; (a +27) 4 ; (a +27) 19.
Do a là số tự nhiên nhỏ nhất nên a+27 nhỏ nhất
Suy ra: a +27 = BCNN (4 ;11 ; 19 ) .
Từ đó tìm được : a = 809
0.5
0.5
0.5
0.5
b
Số thứ nhất bằng:
11
9
:
7
6
=
22
21
(số thứ hai)
Số thứ ba bằng:
11
9
:
3
2
=
22
27
(số thứ hai)
Số thứ hai bằng:
22
22
(số thứ hai)
Tổng của 3 số bằng:
22
272122 ++
(số thứ hai) =
22
70
(số thứ hai)
Số thhai : 210 :
22
70
= 66 ; số thứ nhất là:
22
21
. 66 = 63 ; số th
3 là:
22
27
.66 = 81
0.5
0.5
0.5
0.5
Câu 4
(6 điểm )
a
1) Trên tia BA ta có BK = 2 cm. BA = 7cm nên BK< BA do
đó điểm K nằm giữa A và B. Suy ra AK + KB = AB hay AK + 2
= 7
AK = 5 cm. Trên tia AB có điểm I và K mà AI < AK (và 4
<5) nên điểm I nằm giữa A và K
2,5
2) Do I nằm giữa A K nên AI + IK = AK. Hay 4 + IK = 5
IK = 5 4 = 1.
1,5
b
Vì A nằm giữa B và C nên BA +AC = BC
BA +AC = 4 (1)
Lập luân
B nằm giữa A và D.
Theo gt OD < OA D nm giữa O và A.
Mà OD + DA = OA
2 + DA =5
DA =3 cm
Ta có DB + BA = DA
DB +BA = 3 (2)
Lấy (1) – (2): AC DB = 1 (3)
Theo đề ra : AC = 2BD thay và (3)
Ta có 2BD BD = 1 BD = 1
AC = 2BD AC = 2 cm
0,5
0,5
0,5
0,5
Câu 5
( 2 điểm )
Ta có 3
2
+ 3
3
+ 3
4
+…… + 3
101
= (3
2
+ 3
3
+ 3
4
+ 3
5
) + (3
6
+ 3
7
+ 3
8
+ 3
9
)+…+ (3
98
+ 3
99
+ 3
100
+ 3
101
)
= 3
1
(3+3
2
+3
3
+3
4
) + 3
5
(3+3
2
+3
3
+3
4
) +…+3
97
(3+3
2
+3
3
+3
4
)
0,5
0,5
Trang 23
= 3
1
.120 + 3
5
.120 +…+3
97
.120
= 120(3
1
+ 3
5
+…+3
97
) 120 (đpcm)
0,5
0,5
Lưu ý .Học sinh có cách giải khác đúng vẫn cho điểm tối đa.
PHÒNG GIÁO DỤC VÀ ĐÀO TẠO
ĐỀ KHẢO SÁT HỌC SINH GIỎI LỚP 6 CẤP HUYỆN
Môn: Toán Lớp 6
Thời gian làm bài: 120 phút (Không kể thời gian giao đề)
ĐỀ 7
Bài 1: (4,0 điểm)
a) Cho
7 5 8 4.n a b=+
Biết a – b = 6 và n chia hết cho 9. Tìm a và b.
b) Tìm các số tự nhiên x, y sao cho: 5
x
+ 12
y
= 26.
Bài 2: (4,0 điểm)
a)Tìm c số nguyên a, b biết rằng:
11
7 2 3
a
b
−=
+
b) Tìm x, biết : (
3.2.1
1
+
4.3.2
1
+ . . . +
10.9.8
1
) . x =
22
45
Bài 3: (4,0 điểm)
a) Cùng một ng việc nếu mỗi người làm riêng t 3 người A, B, C hoàn thành
công việc trong thời gian lần lượt 6 giờ, 8 giờ, 12 giờ. Hai người B và C làm chung
trong 2 giờ sau đó người C chuyển đi m việc khác, người A cùng làm với người B tiếp
tục công việc cho đến khi hoàn thành. Hỏi người A làm trong mấy giờ?
b) Cho D = 5 + 5
2
+ 5
3
+ 5
4
+ ... + 5
19
+ 5
20
. Tìm số dư khi chia D cho 31.
Bài 4: (4,0 điểm)
a) So sánh M và N biết: M =
519
519
31
30
+
+
; N =
519
519
32
31
+
+
b) Thực hiện tính:
( ) ( ) ( ) ( )
1 1 1 1
E = 1+ 1 + 2 + 1 + 2 + 3 + 1 + 2 + 3 + 4 + ... + 1 + 2 + ... + 200
2 3 4 200
Bài 5: (4,0 điểm)
a) Cho:
xOy
= 120
0
,
xOz
= 50
0
. Gọi Om là tia phân của góc
yOz
. Tính
xOm
b) Cho 20 điểm phân biệt trong đó đúng 7 điểm thẳng hàng, ngoài ra không ba
điểm nào thẳng hàng. Cứ qua hai điểm ta vđược một đường thẳng. Hỏi từ 20 điểm đó vẽ
được tất cả bao nhiêu đường thẳng?
Trang 24
PHÒNG GIÁO DỤC VÀ ĐÀO TẠO
ĐỀ KHẢO SÁT HỌC SINH GIỎI LỚP 6 CẤP HUYỆN
Môn: Toán Lớp 6
Thời gian làm bài: 120 phút (Không kể thời gian giao đề)
HƯỚNG DẪN CHM ĐỀ 7
Câu
Phần
Nội dung cần trình bày
Điểm
1
(4đ)
a
(2đ)
Cho
7 5 8 4.n a b=+
Biết a – b = 6 và n chia hết cho 9. Tìm a và b.
0,5
0,5
0,5
0,5
Ta có: n =
7 5 8 4 9 7 5 8 4 9a b a b+ + + + + +MM
24 9 3;12a b a b + + + M
(vì a + b < 19).
Mà a b = 6 nên a + b > 3. Do đó a + b = 12.
Kết hợp với a b = 6, suy ra a = 9, b = 3.
b
(2đ)
Tìm các số tự nhiên x, y sao cho: 5
x
+ 12
y
= 26.
0,5
0,5
1
Ta có 12
2
= 144 > 26 và y N => 0
y
1 => y
{
0; 1
}
+) Với y = 1 => 5
x
+ 12
1
= 26 => 5
x
= 14 => không tìm được
x N.
+) Với y = 0 => 5
x
+ 12
0
= 26 => 5
x
= 25=5
2
=> x = 2
2
(4đ)
a
(2đ)
Tìm các số nguyên a, b biết rằng:
11
7 2 3
a
b
−=
+
0,5
0,5
0,5
0,5
1 1 2 7 1
(2 7)( 3) 14.
7 2 3 14 3
aa
ab
bb
= = + =
++
Do
,a b Z
nên 2a 7
Ư(14)
Vì 2a 7 lẻ nên 2a – 7
7; 1;1;7 0;3;4;7 .a
Từ đó tính được: (a; b) = (0; -5), (3; -17), (4; 11), (7; -1)
b
(2đ)
Tìm x , biết : (
3.2.1
1
+
4.3.2
1
+ . . . +
10.9.8
1
) . x =
22
45
0,5
0,5
0,5
0,5
(
3.2.1
1
+
4.3.2
1
+ . . . +
10.9.8
1
) . x =
22
45
2
1
(
1 1 1 1 1 1
1.2 2.3 2.3 3.4 8.9 9.10
+ ++
) . x =
22
45
2
1
(
11
2 90
) . x =
22
45
x = 2
3
(4đ)
a
(2đ)
Cùng một công việc nếu mỗi người làm riêng thì 3 người A, B,
C hoàn thành công việc trong thời gian lần lượt 6 giờ, 8 giờ,
12 giờ. Hai người B C m chung trong 2 giờ sau đó người
C chuyển đi làm việc khác, người A cùng làm với người B tiếp
tục ng việc cho đến khi hoàn thành. Hỏi người A làm trong
mấy giờ?
Trong 1 giờ mỗi người A, B, C lần lượt làm được
)(
6
1
CV
,
Trang 25
)(
8
1
CV
,
)(
12
1
CV
, B và C làm được
)(
24
5
12
1
8
1
CV=+
2 giờ B và C làm được
)(
12
5
2
24
5
CV=
A và B làm được
)(
12
7
12
5
1 CV=
1 giờ A và B cùng làm được:
)(
24
7
8
1
6
1
CV=+
Thời gian A cùng làm với B là:
2
24
7
:
12
7
=
giờ.
0,5
0,5
0,5
0,5
b
(2đ)
Cho D = 5 + 5
2
+ 5
3
+ 5
4
+ ... + 5
19
+ 5
20
. Tìm số dư khi chia D
cho 31.
0,5
0,5
1
D + 1 = (1+ 5 + 5
2
) + 5
3
(1+ 5 + 5
2
) + 5
6
(1+ 5 + 5
2
) + ... +
5
18
(1+ 5 + 5
2
)
Do 1 + 5 + 5
2
= 31 nên D + 1 chia hết cho 31
=> D chia 31 dư 30.
4
(4đ)
a
(2đ)
So sánh M và N biết M =
519
519
31
30
+
+
; N =
519
519
32
31
+
+
0,5
0,5
0,5
0,5
M =
519
519
31
30
+
+
nên 19M =
519
)519.(19
31
30
+
+
=
519
9519
31
31
+
+
= 1 +
519
90
31
+
N =
519
519
32
31
+
+
nên 19N =
519
)519.(19
32
31
+
+
=
519
9519
32
32
+
+
= 1 +
519
90
32
+
519
90
31
+
>
519
90
32
+
Suy ra 1 +
519
90
31
+
> 1 +
519
90
32
+
Hay 19M > 19N
Nên M > N
b
(2đ)
Thực hiện tính:
( ) ( ) ( ) ( )
1 1 1 1
E = 1+ 1 + 2 + 1 + 2 + 3 + 1 + 2 + 3 + 4 + ... + 1 + 2 + ... + 200
2 3 4 200
0,5
0,5
0,5
Từ
( 1)
1 2 3 ...
2
nn
n
+
+ + + + =
được:
1 2.3 1 3.4 1 4.5 1 200.201
1 . . . ... .
2 2 3 2 4 2 200 2
E = + + + + +
3 4 5 201
1 ...
2 2 2 2
= + + + + +
( )
1 1 2 3 4 5 201
...
2 2 2 2 2 2 2
1
1 2 4 ... 201
2
E + = + + + + + +
= + + + +
E =
( )
1 1 1 201.202 1
1 2 4 ... 201 .
2 2 2 2 2
+ + + + =
Trang 26
= 10150
0,5
5
(4đ)
a
(2đ)
Cho:
xOy
= 120
0
,
xOz
= 50
0
. Gọi Om là tia phân củac
yOz
.
Tính
xOm
0,5
0,5
0,5
0,5
a, Trường hợp 1 : 2 tia Oy, Oz thuộc cùng một nửa mặt phẳng
có bờ Ox
xOz
= 50
0
< 120
0
=
xOy
nên tia
Oz nằm giữa Ox và Oy.
yOz
=
xOy
xOz
= 120
0
50
0
=
70
0
zOm
=
2
yOz
= 35
0
xOm
= 35
0
+ 50
0
= 85
0
b, Trường hợp 2 : 2 tia Oy, Oz thuộc hai nửa mặt phẳng đối
nhau bờ Ox.
Từ đầu bài ta có Ox nm giữa 2 tia Oy
và Oz.
yOz
= 120
0
+ 50
0
= 170
0
zOm
=
170
2
=
85
0
xOm
= 85
0
50
0
= 35
0
b
(2đ)
Cho 20 điểm phân biệt trong đó đúng 7 điểm thẳng hàng,
ngoài ra kng ba điểm nào thẳng hàng. Cứ qua hai điểm ta
vẽ được một đường thẳng. Hỏi từ 20 điểm đó vđược tất cả
bao nhiêu đường thẳng?
0,5
0,5
0,5
0,5
Nếu trong 20 điểm không ba điểm nào thẳng hàng thì v
được
190
2
)120.(20
=
. (Đường thẳng).
Trong 7 điểm kng ba điểm nào thẳng hàng ttạo thành
21
2
)17.(7
=
(Đường thẳng).
7 điểm thẳng hàng tạo thành 1 đường thẳng nên số đường
thẳng giảm 21 - 1 = 20 (Đường thẳng).
Vậy có 190 20 = 170 (Đường thẳng).
Lưu ý : Mọich giải khác đúng đều cho điểm tối đa
Trang 27
PHÒNG GIÁO DỤC VÀ ĐÀO TẠO
ĐỀ KHẢO SÁT HỌC SINH GIỎI LỚP 6 CẤP HUYỆN
Môn: Toán Lớp 6
Thời gian làm bài: 120 phút (Không kể thời gian giao đề)
ĐỀ 8
Bài 1: (4.0 điểm) Thực hiện phép tính
a) A=1.2.3…9 - 1.2.3…8 - 1.2.3…8.8
b) B=
( )
2
16
13 11 9
3.4.2
11.2 .4 16
c) C = 70.(
565656
131313
+
727272
131313
+
909090
131313
)
d) Thực hiện phép tính: B =
1 1 1 1
...
4.9 9.14 14.19 64.69
+ + + +
Bài 2: (4.0 điểm) Tìm x biết :
a)
x2
2
1
+
3
2
=
7
3
b)
( )
3 54 .8 :4 18x −=


c)
( ) ( )
53
2 15 2 15xx =
d) x + (x + 1) + (x + 2) +…+ ( x + 2013) = 2035147
i 3: (4.0 điểm)
a). m số tự nhiên nhỏ nht, biết rng số đó khi chia cho 3, cho 4, cho 5, cho 6 đều
2, còn chia cho 7 thì 3.
b) Tìm x, y nguyên biết: x + y + xy = 40
c) Khi chia một số tự nhiên a cho 4 ta được số dư là 3 còn khi chia a cho 9 ta được số
dư là 5. Tìm số dư trong phép chia a cho 36.
Bài 4: (6.0 điểm) Cho góc
xBy
= 55
0
. Trên các tia Bx; By lần lượt lấy các điểm A, C sao
cho A
B; C
B. Trên đoạn thẳng AC lấy điểm D sao cho
ABD
= 30
0
a. Tính độ dài AC, biết AD = 4cm, CD = 3cm.
b. Tính số đo của
DBC
.
c. Từ B vẽ tia Bz sao cho
DBz
= 90
0
. Tính số đo
ABz
.
Bài 5: (2.0 điểm) Cho tổng T =
1
2
2
+
2
2
3
+
3
2
4
+
..............
+
2015
2
2016
+
2016
2
2017
So sánh T với 3
---------------------------------------
- Họ và tên thí sinh: …………………………………..; Số báo danh ………………
Chú ý: Cán bộ coi giao lưu kng được giải thích gì thêm.
Trang 28
PHÒNG GIÁO DỤC VÀ ĐÀO TẠO
ĐỀ KHẢO SÁT HỌC SINH GIỎI LỚP 6 CẤP HUYỆN
Môn: Toán Lớp 6
Thời gian làm bài: 120 phút (Không kể thời gian giao đề)
HƯỚNG DẪN CHM ĐỀ 8
Bài 1:
(4.0
điểm)
Thực hiện phép tính
a) A = 1.2.3…9- 1.2.3…8- 1.2.3…8.8
= 1.2.3…8.(9 - 1 - 8)
= 0
b) B =
( )
2
16
13 11 9
3.4.2
11.2 .4 16
( )
( )
2
2 16
13 22 36
36
35 36
36
35
36
35
3.2 .2
11.2 .2 2
9.2
11.2 2
9.2
2 . 11 2
9.2
2
2 .9
=
=
=
==
c) C = 70.(
565656
131313
+
727272
131313
+
909090
131313
)
= 70.(
56
13
+
72
13
+
90
13
)
= 70.13.(
8.7
1
+
9.8
1
+
10.9
1
)
= 70.13.(
7
1
-
10
1
)
= 39
d ) B =
1 1 1 1
...
4.9 9.14 14.19 64.69
+ + + +
=
1 1 1 1 1 1 1 1 1
( ... )
5 4 9 9 14 14 19 64 69
+ + + +
=
1 1 1
()
5 4 69
=
13
276
0,5
0,5
0,25
0,25
0,25
0,25
0,25
0,25
0,25
0,25
0,5
0,25
0,25
Bài 2:
(4.0
điểm)
a)
x2
2
1
+
3
2
= 4
x2
2
1
= 4 -
3
2
x2
2
1
=
3
10
TH1:
2
1
- 2x =
3
10
2x =
2
1
-
3
10
2x =
6
17
x =
12
17
TH2:
2
1
- 2x =
3
10
2x =
2
1
+
3
10
2x =
6
23
x =
12
23
0,25
0,25
0,25
Trang 29
Vậy x=
12
17
; x =
12
23
b)
( )
3 54 .8 :4 18x −=


( )
3 54 .8 72
3 54 9
3 63
21
x
x
x
x
−=
−=
=
=
Vậy x = 21
c)
( ) ( )
53
2 15 2 15xx =
( ) ( )
( ) ( )
( )
( )
( ) ( )
53
32
3
2
22
2
2 15 2 15 0
2 15 . 2 15 1 0
2 15 0
2 15 1 0
*2 15 0 7,5
* 2 15 1 0 2 15 1
2 15 1 8
2 15 1 7
xx
xx
x
x
xx
xx
xx
xx
=

=

−=
=
= =
= =
= =



= =

Vậy
7;7,5;8x
d) x + (x + 1) + (x + 2) +…+ ( x + 2013) = 2035147
2014x + (1+2+3+…+2013) = 2035147
2014x + 2027091 = 2035147
2014x = 8056
x = 4
Vậy x = 4
0,25
0,25
0,25
0,25
0,25
0,25
0,25
0,25
0,25
0,25
0,25
0,25
0,25
Bài 3:
(4.0
điểm)
a) Gọi số tự nhiên cần tìm là a
Vì a chia cho 3, cho 4, cho 5, cho 6 đều là 2 n a - 2 chia hết cho 3,
cho 4, cho 5, cho 6 do đó a - 2 BC(3, 4, 5, 6)
+ BCNN(3, 4, 5, 6) = 60
+ Lp lun a - 2
0;60;120;180;...
a
2;62;122;182;...
a số t nhiên nhỏ nhất chia cho 7 thì dư 3 n a = 122
b) x + y + xy = 40
(y+1)x + y + 1= 41
(x + 1)(y + 1) = 41
Mà x, y nguyên => x +1 và y + 1 là ước của 41
Tính được (x, y)
( ) ( ) ( ) ( )
40;0 ; 0;40 ; 2; 42 ; 42; 2
0,5
0,5
0,5
0,25
0,25
0,5
Trang 30
c) Theo đề bài ta có: a = 4p+3 = 9q + 5 ( p, q nguyên)
Suy ra a + 13 = 4p + 3 + 13 = 4(p + 4) (1)
a + 13 = 9q + 5 + 13 = 9(q + 2) (2)
Từ (1) và (2) ta nhận thấy a + 13 là bội của 4 và 9 mà (4,9) = 1 nên a
+ 13 là bội của 4.9 = 36
Ta có a + 13 = 36k (k nguyên) => a = 36k 13 = 36(k - 1) + 23
Vậy a chia cho 36 dư 23
0,25
0,25
0,5
0,25
0,25
Bài 4:
(6.0
điểm)
a) D thuộc đoạn thẳng AC nên D nằm giữa A và C
=> AC = AD + CD
= 4 + 3 = 7(cm)
Vậy AC = 7cm
b) Chứng minh tia BD nm giữa hai tia BA và BC
ta có đẳng thức:
ABC
=
ABD
+
DBC
=>
DBC
=
ABC
-
ABD
= 55
0
30
0
= 25
0
c) Xét hai trường hợp ( Học sinh vẽ hình trong hai trường hợp)
- Trường hợp 1: Tia Bz BA nằm trên cùng 1 nửa mặt phẳng b
là BD
+ Lập luận tia BA nằm giữa hai tia Bz và BD
Tính được
ABz
=
DBz
-
ABD
=
000
603090 =
- Trường hợp 2: Tia Bz
,
BA nằm trên hai nửa mặt phẳng đối nhau
có bờ là BD
+ Lập luận tia BD nằm giữa hai tia Bz và BA
Tính được
ABz
=
DBz
+
ABD
=
000
1203090 =+
0,75
0,5
0,5
0.25
1.0
0.5
0.5
0.5
0.5
0.5
0.5
Bài 5:
(2.0
điểm)
T =
1
2
2
+
2
2
3
+
3
2
4
+
..............
+
2015
2
2016
+
2016
2
2017
2T = 2 +
1
2
3
+
2
2
4
+
..............
+
2014
2
2016
+
2015
2
2017
2T T=
y
x
z
z
B
C
A
D
Trang 31
2 +
1
2
3
-
1
2
2
+
2
2
4
-
2
2
3
+…….+
2014
2
2016
-
2014
2
2015
+
2015
2
2017
-
2015
2
2016
-
2016
2
2017
T= 2+
1
2
1
+
2
2
1
+………+
2015
2
1
-
2016
2
2017
Đặt N =
1
2
1
+
2
2
1
+………+
2015
2
1
Ta có 2N = 1+
1
2
1
+
2
2
1
+………+
2014
2
1
2N-N= 1-
2015
2
1
Vậy N < 1
Nên T< 2+1-
2016
2
2017
=3-
2016
2
2017
Vậy T<3
0.75
0.5
0.5
0.25
Ghi chú:
- i hình không có hình vẽ hoặc hình vẽ sai thì không chấm điểm.
- Học sinh làm cách khác đúng thì vẫn cho điểm tối đa.
PHÒNG GIÁO DỤC VÀ ĐÀO TẠO
ĐỀ KHẢO SÁT HỌC SINH GIỎI LỚP 6 CẤP HUYỆN
Môn: Toán Lớp 6
Thời gian làm bài: 120 phút (Không kể thời gian giao đề)
ĐỀ 9
Câu 1: (4 điểm)nh:
a)
1 2 3 4 5 6 7 8 9 ... 2013 2014 2015 2016A = + + + + + + +
b)
2.4.10 4.6.8 14.16.20
3.6.15 6.9.12 21.24.30
B
++
=
++
Câu 2: (6 điểm)
a) So sánh
2014
2015
10 2016
10 2016
A
+
=
+
2015
2016
10 2016
10 2016
B
+
=
+
b) Tìm x biết:
1 1 1 1 119
( ... ).
1.2.3.4 2.3.4.5 3.4.5.6 7.8.9.10 720
x+ + + + =
c) Chứng minh rằng: nếu p và p
2
+2 là c số nguyên tố thì p
3
+2 cũng là số nguyên tố.
Câu 3: (4 điểm)
a) Tìm số tự nhiên n để phân số
21
2
n
n
+
+
phân số rút gọn được.
b) Trong đợt tổng kết năm học tại một trường THCS, tổng số học sinh giỏi ca ba lớp 6A,
6B, 6C 90 em. Biết rằng
2
5
số học sinh gii ca lớp 6A bằng
1
3
số học sinh giỏi của lớp 6B và
bằng
1
2
số học sinh giỏi của lớp 6C. Tính số học sinh giỏi mỗi lớp.
Trang 32
Câu 4: (4 điểm)
Cho tam giác ABC có
·
0
ACB 60=
, AB=6cm. Trên cạnh AB lấy điểm D (D khác A,B) sao
cho AD=2cm.
a) Tính độ dài đon thẳng BD.
b) Tính số đo ca
·
DCB
biết
·
0
ACD 20=
.
c) Dựng tia Cx sao cho
·
0
DCx 90=
. Tính
·
ACx
.
d) Trên cạnh AC lấy điểm E (E khác A,C). Chứng minh hai đoạn thẳng CD và BE cắt
nhau.
Câu 5: (2 điểm) Tìm bộ ba snguyên dương a, b, c sao cho:
1 1 1 4
5abc
+ + =
------ HẾT ------
Cán bộ coi thi không giải thích gì thêm
Họ và tên thí sinh: ................................................Số báo danh:...........................
Giám thị 1 (Hn và ký)...........................................................................................................
Giám thị 2 (Hn và ký)...........................................................................................................
PHÒNG GIÁO DỤC VÀ ĐÀO TẠO
ĐỀ KHẢO SÁT HỌC SINH GIỎI LỚP 6 CẤP HUYỆN
Môn: Toán Lớp 6
Thời gian làm bài: 120 phút (Không kể thời gian giao đề)
HƯỚNG DẪN CHM ĐỀ 9
Câu
Đáp án
Điểm
1.1
(2.0
điểm)
Tính
1 2 3 4 5 6 7 8 9 ... 2013 2014 2015 2016A = + + + + + + +
1 2 3 4 5 6 7 8 9 ... 2013 2014 2015 2016A = + + + + + + +
Tính được số các số hạng của A là (2016 - 1) : 1 + 1 = 2016 s hạng
0,75
Nhóm 4 số hạng liên tiếp vào một nhóm:
(1 2 3 4) (5 6 7 8) ... (2013 2014 2015 2016)A= + + + + + +
0.75
ó 504 sô'
4 ( 4) ... ( 4) 4.504 2016
c
A = + + + = =
1444444442444444443
Vậy A=-2016
0.5
1.2
(2.0
điểm)
2.4.10 4.6.8 14.16.20
3.6.15 6.9.12 21.24.30
B
++
=
++
2.4.10 4.6.8 14.16.20 8.(1.2.5 2.3.4 7.8.10) 8
3.6.15 6.9.12 21.24.30 27.(1.2.5 2.3.4 7.8.10) 27
B
+ + + +
= = =
+ + + +
1.75
Vậy B=
8
27
0.25
2.1
(2.0
điểm)
Sonh
2014
2015
10 2016
10 2016
A
+
=
+
2015
2016
10 2016
10 2016
B
+
=
+
Trang 33
Ta có
2014 2014 2016
2015 2015 2016
10 2016 (10 2016)(10 2016)
10 2016 (10 2016)(10 2016)
A
+ + +
==
+ + +
4030 2014 2016 2
2015 2016
4030 2014 2
2015 2016
10 2016.(10 10 ) 2016
(10 2016)(10 2016)
10 2016.10 .101 2016
(1)
(10 2016)(10 2016)
+ + +
=
++
++
=
++
0.75
Ta có
2015 2015 2015
2016 2016 2015
10 2016 (10 2016)(10 2016)
10 2016 (10 2016)(10 2016)
B
+ + +
==
+ + +
4030 2015 2
2016 2015
4030 2014 2
2016 2015
10 2.2016.10 2016
(10 2016)(10 2016)
10 20.2016.10 2016
(2)
(10 2016)(10 2016)
++
=
++
++
=
++
0.75
Từ (1) và (2) suy ra A>B
0.25
Vậy A>B
0.25
2.2
(2.0
điểm)
Tìm x biết:
1 1 1 1 119
( ... ).
1.2.3.4 2.3.4.5 3.4.5.6 7.8.9.10 720
x+ + + + =
(1)
Ta có:
1 1 1 1
...
1.2.3.4 2.3.4.5 3.4.5.6 7.8.9.10
+ + + +
1 1 1 1 1 1 1
( ... )
3 1.2.3 2.3.4 2.3.4 2.3.4 7.8.9 8.9.10
1 1 1 1 119
( ) .
3 6 720 3 720
= + + +
= =
1,25
Nên từ (1) suy ra:
1 119 119
..
3 720 720
x =
=>x=3
0.5
Vậy x=3
0.25
2.3
(2.0
điểm)
Chng minh rằng: nếu p và p
2
+2 là c số nguyên tố thì p
3
+2 cũng số nguyên tố.
Ta nhận xét rằng mọi số nguyên tố lớnn 3 thì chia cho 3 đều có dạng
p=3k+1 hoặc p=3k+2 (
*
kN
)
0.5
Với p=3k+1 thì p
2
+2=9k
2
+6k+3 chia hết cho 3.
Với p=3k+2 thì p
2
+2=9k
2
-6k+6 chia hết cho 3
0.5
p nguyên tố nên p
2
khi đó trong cả 2 trường hợp trên thì p
2
+2 đều lớn
hơn 3 và chia hết cho 3. Tức là p
2
+2 là hợp số
=> p
2
+2 chỉ là nguyên tố khi p=3 (khi đó p
2
+2=11 là số nguyên tố)
=> p
3
+2=27+2=29 số nguyên tố
0.75
Vậy nếu p và p
2
+2 là c số nguyên tố thì p
3
+2 cũng là số nguyên tố.
0.25
3.1
(2.0
điểm)
Tìm số tự nhiên n để pn số
21
2
n
n
+
+
phân số rút gọn đưc.
Gọi d là ƯCLN(2n+1,n+2) (d
*
N
)
0.75
Trang 34
Ta có 2n+1
M
d, n+2
M
d => [(2n+4)-(2n+1)]
M
d
=> 3
M
d
Vì d
*
N
nên d
{1;3}
Để phân số
21
2
n
n
+
+
rút gọn được thì d=3
0.75
=> n+2=3k (
*
kN
)
=> n=3k-2 (
*
kN
)
Vậy với n=3k-2 (
*
kN
) thì phân số
21
2
n
n
+
+
là phân số rút gọn được.
0.5
3.2
(2.0
điểm)
Trong đợt tổng kết năm học tại một trường THCS, tổng số học sinh giỏi của ba lớp 6A,
6B, 6C 90 em. Biết rằng
2
5
số học sinh gii ca lớp 6A bằng
1
3
số học sinh giỏi của
lớp 6B và bằng
1
2
số học sinh gii ca lớp 6C. Tính số học sinh giỏi mỗi lớp.
Số học sinh giỏi ca lp 6B bằng
2 1 6
:
5 3 5
=
( số học sinh giỏi lớp 6A)
Số học sinh giỏi lớp 6C bằng
2 1 4
:
5 2 5
=
( số học sinh giỏi lớp 6A)
Số học sinh giỏi ca cả 3 lớp bằng
64
13
55
+ + =
( số học sinh giỏi lớp 6A)
Vậy số học sinh gii lớp 6A là 90: 3 = 30 học sinh, của lớp 6B là 36 học sinh và
của lớp 6C là 24 học sinh
0.5đ
0.5đ
0.5đ
0.5đ
4
(4.0
điểm)
Cho tam giác ABC có
·
0
60ACB =
, AB=6cm. Trên cạnh AB lấy điểm D sao cho
AD=2cm.
a) Tính đdài đon thẳng BD.
b) Tính sđo của góc DCB biết
·
0
20ACD =
.
c) Dựng tia Cx sao cho
·
0
90DCx =
. Tính
·
ACx
.
d) Trên cạnh AC lấy điểm E. Chứng minh hai đoạn thẳng CD và BE cắt nhau.
Trường hp 1 Trường hợp 2
a) D nằm giữa A và B => AD+BD=AB=>BD=6-2=4cm
KL...
0.75
0.25
E
E
Trang 35
b) Tia CD nằm giữa hai tia CA và tia CB
=>
·
·
·
ACD DCB ACB+=
=>
·
DCB
=40
0
KL...
0.75
0.25
c) Xét hai trường hợp:
- Trường hp 1: Hai tia CD và Cx nằm về một phía so với đường thẳng CB
Tính được góc ACx = 90
0
-
·
ACD
= 70
0
K.L...
- Trường hp 2: Hai tia CD và Cx nằm về hai phía so với đường thẳng CB
Tính được góc ACx = 90
0
+
·
ACD
= 110
0
K.L ...
0.5
0.5
- Xét đường thẳng CD.
Do CD cắt AB nên đường thẳng CD chia mặt phẳng m 2 nửa: 1 nửa MP có bờ
CD chứa điểm B và nửa MP bCD chứa điểm A => tia CA thuộc nửa MP chứa
điểm A.
E thuộc đoạn AC => E thuộc nửa MP bờ CD chứa điểm A
=> E và B ở 2 nửa MP bờ CD
=> đường thẳng CD cắt đoạn EB
- Xét đường thng BE.
Lập luận tương tự: ta có đưng thẳng EB cắt đoạn CD.
Vậy 2 đoạn thẳng EB và CD cắt nhau.
0.5
0.5
5
(1.0
điểm)
Tìm bộ ba số nguyên dương a, b, c sao cho:
1 1 1 4
5abc
+ + =
Khôngm mấtnh tổng quát, ta giả sử: a
b
c khi đó ta có:
3 4 15
, a
54a

Nếu a=1 thì không thể được, do đó a= 2 hoặc a=3
0.5
Nếu a=2 thì
1 1 3
10bc
+=
Suy ra
2 3 20
, b
10 3b

Suy ra b=4 hoặc b= 5 hoặc b=6 vì
3
10
<
1
3
Suy ra c số a, b, c thỏa mãn là (a=2,b=4,c=20) và (a=2,b=5,c=10)
0.5
Nếu a=3 thì
1 1 7
15bc
+=
từ đó
2 7 30
, b
15 7b

suy ra b=3 hoặc b=4. Không có trường hợp nào tha mãn
0.5
K.L có 12 b số tha mãn là c hoán vị ca hai bộ ba số (2,4,20) và (2,5,10)
0.5
Điểm toàn bài
20
điểm
Ghi chú:
- i hình không có hình vẽ hoặc hình vẽ sai thì không chấm điểm.
- Học sinh làm cách khác đúng thì vẫn cho điểm tối đa.
Trang 36
PHÒNG GIÁO DỤC VÀ ĐÀO TẠO
ĐỀ KHẢO SÁT HỌC SINH GIỎI LỚP 6 CẤP HUYỆN
Môn: Toán Lớp 6
Thời gian làm bài: 120 phút (Không kể thời gian giao đề)
ĐỀ 10
Câu 1(2,0 điểm): Tính hợp
a) 21.7
2
- 11.7
2
+ 90.7
2
+ 49.125.16
c)
15 9 20 9
9 19 29 6
5.4 .9 4.3 .8
5.2 .6 7.2 .27
Câu 2(6,0 điểm): Tìm x là số tự nhn, biết:
a) x : (
2
1
9
-
2
3
) =
11
8
9
8
6,1
11
2
9
2
4,0
+
+
b)
2
1+x
=
1
8
+x
c) 5
2x - 3
2.5
2
= 5
2
.3 d)
2 7 20 5.( 3)x = +
Câu 3(6,0 điểm):
a) Tìm số nguyên x và y, biết : xy - x + 2y = 3.
b) Tìm các số tự nhiên x, y biết: 2
x + 1
. 3
y
= 12
x
c) Cho số
16*4*710*155
có 12 chữ số. Chứng minh rằng nếu thay các dấu (*) bởi
các chữ số khác nhau trong ba chữ số 1; 2; 3 một cách tuỳ ý thì số đó luôn chia hết cho
396.
d) Tìm số tự nhn n để biểu thức sau là số tự nhn:
B =
2 2 5 17 3
2 2 2
n n n
n n n
++
+−
+ + +
Câu 4(5,0 điểm):
Cho đoạn thẳng AB = 5cm. Lấy điểm M thuộc đoạn thẳng AB, trên tia đối của tia AB lấy điểm N
sao cho AN = AM.
a) Tính BN khi BM = 2cm.
b) Trên cùng một nửa mặt phẳng bờ đường thẳng AB, vẽ c tia Ax Ay sao cho
00
BAx 40 , BAy 110==
. Tính
yAx, NAy
.
c) Xác định vị trí của điểm M tn đon thẳng AB để đoạn thẳng BN có độ dài lớn nhất.
Câu 5(1,0 điểm):
Tìm số tự nhiên n và chữ số a biết rằng: 1 + 2 + 3 + …….+ n =
aaa
.................................... Hết ......................................
Họ và tên thí sinh: ........................................................ Số báo danh: ....................
Trang 37
PHÒNG GIÁO DỤC VÀ ĐÀO TẠO
ĐỀ KHẢO SÁT HỌC SINH GIỎI LỚP 6 CẤP HUYỆN
Môn: Toán Lớp 6
Thời gian làm bài: 120 phút (Không kể thời gian giao đề)
HƯỚNG DẪN CHẤM ĐỀ 10
Câu
Nội dung
Điểm
1
(3,0đ)
a) 21.7
2
- 11.7
2
+ 90.7
2
+ 49.125.16 = 7
2
(21 11 + 90) + 49.125.16
= 49. 100 + 49. 100. 20 = 49.100(1 + 20) = 49.100.21
0.5đ
0.5đ
b)
15 9 20 9
9 19 29 6
5.4 .9 4.3 .8
5.2 .6 7.2 .27
=
30 18 2 20 27
9 19 19 29 18
5.2 .3 2 .3 .2
5 .2 .2 .3 7.2 .3
=
29 18 2
28 18
2 .3 (5.2 3 )
2
2 .3 (5.3 7.2)
=
0.5đ
0.5đ
2
(6,0đ)
a) x : (
2
1
9
-
2
3
) =
11
8
9
8
6,1
11
2
9
2
4,0
+
+
22
0,4
7 11
:8
22
4 0,4
7 11
x
+−
=

+−


x : 8 =
1
4
x = 2 .Vậy x = 2
0.5đ
1.0đ
b)
2
1+x
=
1
8
+x
(x + 1)
2
= 16 = (
4)
2
*) x + 1 = 4
x = 3
*) x + 1 = - 4
x = - 5 .
Do x
N nên x = 3.
0.75đ
0.5đ
0.25đ
c) 5
2x - 3
2.5
2
= 5
2
.3
5
2x - 3
= 5
2
.3 + 2.5
2
5
2x - 3
= 5
2
.5
5
2x - 3
= 5
3
2x - 3 = 3
2x = 6
x = 3. Vậy x = 3
0.5đ
0.5đ
0.5đ
d)
2 7 20 5.( 3)x = +
2 7 5 2x 7 5x = =
*) 2x 7 = 5
2x =12
x = 6
*) 2x 7 = - 5
2x = 2
x = 1
Vậy x
6;1
0.75đ
0.5đ
0.25đ
3
(6,0đ)
a) Tìm số nguyên x và y, biết : xy - x + 2y = 3.
xy - x + 2y = 3
( xy x) + (2y 2) = 1
x( y 1) + 2( y 1) = 1
(y 1)( x + 2) = 1
*)
1 1 2
2 1 1
yy
xx
= =


+ = =

*)
1 1 0
2 1 3
yy
xx
= =


+ = =

Vậy x = - 1 ; y = 2 hoặc x = -3 ; y = 0
0.75đ
0.75đ
Trang 38
b) 2
x + 1
. 3
y
= 12
x
2
x + 1
. 3
y
= (4.3)
x
= 2
2x
.3
x
2
1
1
23
23
23
xy
x y x
xx
−−
+
= =
Nhận thấy : ( 2, 3) = 1
x 1 = y - x = 0
x = y = 1
0.5đ
0.5đ
0.5đ
c) Ta thấy, vị trí của c chữ số thay thế ba dấu sao trong s trên đều
hàng chẵn và vì ba chữ số đó đôi một khác nhau, lấy từ tập hợp
3;2;1
nên tổng của chúng luôn bằng 1+ 2+ 3 = 6.
Mặt khác 396 = 4.9.11 trong đó 4;9;11 đôi một nguyên t cùng nhau nên
ta cần chứng minh
A =
16*4*710*155
chia hết cho 4 ; 9 và 11.
Thật vậy :
*) A
4 vì số tạo bởi hai chữ số tận cùng của A là 16 chia hết cho 4
*) A
9 vì tng các chữ số chia hết cho 9 :
1+ 5+ 5 +7+ 1 + 4 + 1+ 6 + (*+*+*) = 30 + 6 = 36 chia hết cho 9
*) A
11 vì hiệu số giữa tổng các chữ số hàng chẵn tổng các chữ số
hàng lẻ là 0, chia hết cho 11.
{1+5+7+4+1)-(5+1+6+(*+*+*)} = 18 12 6 = 0
Vậy A
396
0.5đ
0.75đ
0.25đ
d) B =
2 2 5 17 3 2 2 5 17 3 4 19
2 2 2 2 2
n n n n n n n
n n n n n
+ + + + + +
+ = =
+ + + + +
B =
4 19 4( 2) 11 11
4
2 2 2
nn
n n n
+ + +
= = +
+ + +
Để B là số tự nhiên thì
11
2n+
số tự nhiên
11
(n+2)
n + 2
Ư(11) =
1; 11
Do n + 2 > 1 nên n + 2 = 11
n = 9
Vậy n = 9 thì B
N
0,5đ
0,5đ
0,5đ
4
(5.0đ)
Vẽ hình
0.5đ
a) Vì M thuộc AB nên AM + MB = AB
Þ
AM + 2 = 5
AM = 3 cm
AN = AM
AN = 3 cm
Do N thuộc tia đối ca tia AB nên điểm A nằm giữa N và B
BN = AB + AN = 5 + 3 = 8 cm. Vậy BN = 8cm
1.5đ
b) + Tn cùng một nửa mặt phẳng có bờ chứa tia AB có:
00
BAx BAy(40 110 )
Þ
Tia Ax nằm giữa hai tia AB Ay nên ta có:
BAx xAy BAy+=
hay
· ·
0 0 0 0 0
40 xAy 110 xAy 110 40 70+ = Þ = - =
0.75đ
y
A
M
N
)
x
B
40
0
Trang 39
+ Trên cùng một nửa mặt phẳng bờ AB, ta
BAy
NAy
hai
c kề bù
·
·
0
BAy NAy 180Þ + =
hay
·
00
110 NAy 180+=
·
0 0 0
NAy 180 110 70Þ = - =
0.75đ
c) Vì BN = AB + AN = 5 + AN
BN có đdài lớn nhất khi AN có
độ dài lớn nhất
Mà AN = AM
BN có đdài lớn nhất khi AM có độ dài lớn nhất
Có AM
AB
AM lớn nhất khi AM = AB khi đó điểm M trùng
với điểm B.
Vậy khi điểm M trùng với điểm B thì BN có độ dài lớn nhất.
1,5đ
5
(1.0đ)
Dãy số 1; 2; ………; n có n số hạng
1 + 2 +…+ n =
2
).1( nn +
Mà 1 + 2 + 3+…..+ n =
aaa
Suy ra
2
).1( nn +
=
aaa
= a . 111 = a . 3.37
n(n + 1) = 2.3.37.a
tích n(n + 1) Chia hết cho số nguyên tố 37 nên n 37 hoặc n + 1
37
Vì số
2
).1( nn +
có 3 chữ số
n+1 < 74
n = 37 hoặc n + 1 = 37
+) Với n = 37 thì
703
2
38.37
=
( loại)
+) Với n + 1 = 37 thì
666
2
37.36
=
( thoả mãn)
Vậy n = 36 và a = 6. Ta có: 1+ 2 + 3+…..+ 36 = 666
0.5đ
0.5đ
Chú ý:
1. Thí sinh có thể làm bài bằng cách khác, nếu đúng vẫn được điểm tối đa.
2. Nếu thí sinh chứng minh bài nh mà không vẽ hình thì không chấm điểm bài hình.
Trang 40
PHÒNG GIÁO DỤC VÀ ĐÀO TẠO
ĐỀ KHẢO SÁT HỌC SINH GIỎI LỚP 6 CẤP HUYỆN
Môn: Toán Lớp 6
Thời gian làm bài: 120 phút (Không kể thời gian giao đề)
ĐỀ 11
Câu 1: (4 điểm).
1) Tìm tự nhiên n sao cho 4n 5 chia hết cho 2n 1.
2) Cho S = 3
1
+ 3
3
+ 3
5
+ ... + 3
2011
+ 3
2013
+ 3
2015
. Chứng tỏ:
a) S không chia hết cho 9
b) S chia hết cho 70.
Câu 2: (5 điểm)
a) Tìm x biết: ( x + 1) + ( x + 2) + . . . + ( x + 100) = 5750.
b) Tìm số nguyên x, y biết x
2
y x + xy = 6
c) Cho
A 1-5 9-13 17-21 ...= + + +
Biết A = 2013. Hỏi A bao nhiêu shạng? Giá trị
của số hạng cuối cùng là bao nhiêu?
Câu 3: (2 điểm)
Tìm giá trị nhỏ nhất của phân số
ba
ab
+
(
ab
là số có 2 chữ số)
Câu 4. (4 điểm)
Trong một buổi đi tham quan, số nđăng kí tham gia bằng
1
4
số nam. Nhưng sau đó một
bạn nữ xin nghỉ, một bạn nam xin đi thêm nên số nđi tham quan bằng
1
5
số nam. Tính
số học sinh nữ và học sinh nam đã đi tham quan.
Câu 5: (5 điểm)
Cho
0
xOy 120=
,
1
xOz xOy
3
=
. Kẻ tia Om tia phân giác của c xOy.Tính số đo
mOz
.
-Hết-
Trang 41
HD CHẤM THI HỌC SINH GII CẤP HUYỆN
NĂM HỌC 2019 2020
MÔN THI: TN LỚP 6
Câu
Nội dung
Điểm
Câu 1
(4đ)
1) 4n 5 = 4n 2 3 = 2(2n 1) 3 2n 1
2(2n 1) 2n 1 nên 3 2n 1
-> 2n 1
Ư
(3)
Lí luận đi đến 2n – 1
;;1 1 3
Kết luận n
;;0 1 2
0.5đ
0.5đ
0.5đ
0.5đ
2) a) Vì 3 kng chia hết cho 9
Các hạng tử còn lại đều chia hết cho 9
Nên S kng chia hết cho 9
b) Tính được số số hạng của tổng S là 1008 số hạng
S = (3
1
+ 3
3
+ 3
5
) + (3
7
+ 3
9
+ 3
11
) + ... + (3
2011
+ 3
2013
+ 3
2015
)
S = 3. 91+ 3
7
. 91 + …..+ 3
2011
. 1 91
Kết luận S 7
S = (3
1
+ 3
3
) + (3
5
+ 3
7
) + ... + (3
2013
+ 3
2015
)
S = 3. 10+ 3
5
. 10 + …..+ 3
2013
. 10 10
Kết luận S 10
Vì (10, 7) = 1 nên S 70
0.5đ
0.5đ
0.5đ
0.5đ
0.5đ
0.5đ
Câu 2
(4đ)
a)
( x + 1) + ( x + 2) + . . . + ( x + 100) = 5750.
Tính được 100x + 101. 100 : 2 = 5750
Kết luận đúng x = 7
b) x
2
y x + xy = 6
(xy 1)(x + 1) = 5 = 1.5 = (-1)(-5)
Xét 4 trường hợp và kết luận
(x;y) = (-2;2), (-4;0).
1 đ
1 đ
1 đ
1 đ
c) Số số hạng của A là
( )
:. + =


2013 1 4 2 1 1007
(số hạng)
Số hạng cuối cùng là: (1007 1).4 + 1 = 4025
1 đ
1 đ
Câu 3
(2đ)
Đặt A =
ba
ab
+
=
ab
ab
+
+
10
=
a
ab
+
+
9
1
=
b
a
+
+
9
1
1
0.5đ
Trang 42
A Có GTNN 1 +
b
a
có GTLN
b
a
Có GTLN b = 9 và a = 1
Khi đó số
ab
= 19
GTNN của A là 1,9
0.5đ
Câu 4
(4đ)
Tổng số học sinh nam và nữ dự định đi tham quan và đã đi tham
quan là như nhau nên ta lấy làm đơn vị.
Số hs nữ đăng kí đi tham quan bằng
1
4
số nam nên bằng
1
5
tổng số.
Số hs nữ đã đi tham quan bằng
1
5
số nam đã đi tham quan nên bằng
1
6
tổng số.
Số nữ dự định đi nhiều hơn số nữ đã đi là:
1
5
-
11
=
6 30
tổng số hay
1 học sinh
Tổng số hs là
1
:1
30
= 30 (học sinh)
Số hs nữ đã đi tham quan là: 30 .
1
6
= 5 (học sinh)
Số hs nam đã đi tham quan là: 30 25 = 5 (học sinh)
0.5đ
0.5đ
0.5đ
0.5đ
0.5đ
0.5đ
Câu 4
(5đ)
Tính
0
xOz 40=
2 trường hợp :
a, Trường hợp 1: 2 tia Oy, Oz thuộc cùng một nửa mặt phẳng có bờ
Ox
Lí luận để tính
0
xOm 0= 6
Chứng minh tia Oz nằm giữa Ox và Oy.
Tính đúng
0
zOm 0= 2
b, Trường hợp 2 : 2 tia Oy và Oz thuộc hai nửa mặt phẳng đối nhau
bờ Ox.
0.5đ
0.5đ
0.5đ
0.5đ
0.5đ
m
z
x
y
O
Trang 43
Chú ý:
1. Thí sinh có thể làm bài bằng cách khác, nếu đúng vẫn đưc điểm tối đa.
2. Nếu thí sinh chứng minh bài nh mà không vẽ hình thì không chấm điểm bài hình.
Chỉ được Ox nằm giữa Om và Oz.
Tinh đúng
0
zOm 0= 10
O
y
x
z
m
Trang 44
PHÒNG GIÁO DỤC VÀ ĐÀO TẠO
ĐỀ KHẢO SÁT HỌC SINH GIỎI LỚP 6 CẤP HUYỆN
Môn: Toán Lớp 6
Thời gian làm bài: 120 phút (Không kể thời gian giao đề)
ĐỀ 12
Bài 1( 4 điểm)
a, Chứng tỏ 4x + 3y chia hết cho 7 khi 2x + 5y chia hết cho 7
b, Tìm các số tự nhiên có bốn chữ số sao cho khi chia nó cho 130 , cho 150 được các số dư
lần lượt là 88 và 108.
Bài 2 ( 5,0 điểm) :
a) Tính A =
7777 77 7777 77 123498766
.
8585 85 16362 162 987661234

+


b, Tìm phân số lớn nhất, khi chia các phân số
24
7
và
18
11
cho ta đều được các thương
số nguyên.
Bài 3 (2,0 điểm) :
a, Cho biết S =
1 1 1
...
101 102 130
+ + +
. Chứng minh rằng
1
4
< S <
91
330
Bài 4 (4,0 điểm): Tổng bình phương của 3 số tự nhiên là 2596. Biết rằng tỉ số giữa số th
nhất và số thứ hai là
2
3
, giữa số thứ hai và số thứ ba là
5
6
. Tìm ba số đó.
Bài 5 ( 5,0 điểm) :
Cho tia Oz nằm trongc vuông xOy. Vẽ tia Ot sao cho Ox là tia phân giác của góc tOz.
Vẽ tia Om sao cho tia Oy là phân giác của góc zOm.
a, Chứng minh rằng tia Om và tia Ot là hai tia đối nhau .
b, Gọi Ox’ là tia đối của tia Ox, biết góc x’Om bằng 30
0
. Tính góc tOz .
c, Vẽ thêm 2014 tia phân biệt gốc O (kng trùng với các tia Ox,Oz,Oy,Om,Ox’ và Ot ).
Hỏi trong hình vẽ có tất cả bao nhiêu góc ?
Trang 45
PHÒNG GIÁO DỤC VÀ ĐÀO TẠO
ĐỀ KHẢO SÁT HỌC SINH GIỎI LỚP 6 CẤP HUYỆN
Môn: Toán Lớp 6
Thời gian làm bài: 120 phút (Không kể thời gian giao đề)
HƯỚNG DẪN CHM ĐỀ 12
Bài
Tóm tắt nội dung hướng dẫn
Điểm
Bài 1
(4,0 đ)
Câu a ( 2 điểm)
Ta có 4x + 3y 7
4( 4x + 3 y) 7
16x + 12 y 7
14x + 7y + 2x + 5y 7
Mà 14x + 7y = 7(2x + y) 7
Nên 2x + 5y 7
Vậy 4x + 3y 7 khi 2x + 5y 7
Câu b ( 2 điểm)
Gọi số phải tìm là a .
Ta có a + 42 chia hết cho 130 và 150 nên a + 42 là BC(130,150)
Tìm đúng a = 1908; 3858 ;5808; 7758; 9708 ( mỗi gtrị 0,25 đ)
0,5 đ
0,5 đ
0,5 đ
0,5 đ
0,7
1,2
Bài 2
( 5,)
Câu a ( 2,0 điểm)
Ta có
7777 7777:101 77
8585 8585:101 85
7777 7777:101 77
16362 16362:101 162
==
==
A =
75 75 77 77 123498766
.
85 85 162 162 987661234

+


Vậy A = ( 0 + 0) .
123498766
987661234
= 0
Câu b(3,0 điểm)
Từ
18
1
y
3
9
x
=
ta có:
18
1x2
18
1
9
x
y
3
==
(x,y
N)
Suy ra: y(2x-1) = 54 do đó y
Ư(54) =
54;27;18;9;6;3;2;1
,
54 là số chẵn mà 2x-1 là số lẻ nên y là ước chẵn của 54.
0,5 đ
0,5 đ
0,5 đ
0,5 đ
0,5 đ
0,5 đ
0,5 đ
Trang 46
Vậy y
541862 ;;;
Ta có bảng sau:
y
2
6
18
54
2x-1
27
9
3
1
x
14
5
2
1
Vậy (x;y)
)54;1();18;2();6;5();2;14(
0,5 đ
0,75 đ
0,25 đ
Bài 3
(2 đ)
* Chứng minh S <
91
330
S =
1 1 1 1 1 1 1
... ... ...
101 102 110 111 120 121 130
+ + + + + + +
S <
1 1 1 1 1 1 1
... ... ...
100 100 100 110 110 120 120
+ + + + + + +
S <
1 1 1 1 1 1
10 10 10
100 110 120 10 11 12
+ + = + +
S<
66 60 55
660
++
S <
181
660
<
182
660
hay S <
91
330
(1)
* Chứng minh
1
4
< S
S >
1 1 1 1 1 1
... ... ...
110 110 120 120 130 130
+ + + + + + +
S >
1 1 1 1 1 1
10 10 10
110 120 130 11 12 13
+ + = + +
S >
156 143 132
1716
++
S >
431
1716
>
429
1716
Hay S >
1
4
0,25 đ
0,25 đ
0,5 đ
0,25 đ
0,25 đ
Trang 47
Từ (1) và (2) ta có
1
4
< S <
91
330
0,5 đ
Bài 4
(4 đ)
Gọi a, b, c là 3 số tự nhiên phải tìm.
Theo đề bài ta có:
25
;
36
ab
bc
==
( 1) và
2 2 2
2596abc+ + =
(2)
Từ ( 1) suy ra
26
;
35
a b c b==
, thay vào (2) ta có:
2 2 2
4 36
2596
9 25
b b b+ + =
2
2
649
2596
225
900
b
b
=
=
Tính được b = 30,
26
30 20; 30 36
35
ac= = = =
Vậy 3 số tự nhiên cần tìm là: 30; 20; 36
1 đ
1 đ
1 đ
0,75 đ
0,25 đ
Bài 5
(5,0 đ)
- Bài làm không có hình vẽ kng cho điểm.
- Hình vẽ chính xác phần a, b được 0,5 điểm
Câu a : 2,0 điểm
* Chứng minh c tOz + c zOm = 180
0
Tia Oz nằm trong c xOy nên góc xOz + góc zOy = góc xOy =
90
0
0,5 đ
0,25 đ
x’
O
t
x
z
y
m
Trang 48
Chú ý:
1. Thí sinh có thể làm bài bằng cách khác, nếu đúng vẫn đưc điểm tối đa.
2. Nếu thí sinh chứng minh bài hình mà không vẽ hình thì không chấm điểm bài hình.
Theo giả thiết có các tia phân giác nên góc xOz =
1
2
c tOz
góc zOy =
1
2
c zOm
Từ đó suy ra
1
2
c tOz +
1
2
c zOm = 90
0
Hay góc tOz + góc zOm = 180
0
* Chứng minhc tOz và góc zOm là hai góc kề nhau:
* Kết luận : Cho 0,5 điểm
0,25 đ
0,5 đ
0,5 đ
0,25
Câu b : 1,5 điểm
Chứng minh góc tOx = mOx’ = 30
0
( Cùng kề bù với c mOx)
Góc tOx = góc xOz = 30
0
Góc tOz = 60
0
0,5 đ
0,5 đ
0,5 đ
Câu c : 1,0 điểm
Giả sử vẽ thêm n tia phân biệt gốc O không trùng với các tia
Ox,Oy,Oz,Ot,Om,Ox’. Tất cả trong hình vẽ có n+6 tia phân biệt .
Cứ 1 tia trong n+6 tia đó tạo với n+5 tia còn lại thành n+5c .
n+6 tia nên tạo thành (n+5)(n+6) góc , nhưng như thế mỗi góc
được tính 2 lần . Vậy có tất cả là
( )( )
56
2
nn++
góc
Thay = 2014 ta được số góc có
(2014+6)(2014+5) : 2 = 2 039 190 góc
0,25 đ
0,5 đ
0,25 đ
Trang 49
PHÒNG GIÁO DỤC VÀ ĐÀO TẠO
ĐỀ KHẢO SÁT HỌC SINH GIỎI LỚP 6 CẤP HUYỆN
Môn: Toán Lớp 6
Thời gian làm bài: 120 phút (Không kể thời gian giao đề)
ĐỀ 13
Câu 1: (5 điểm)
a) Tìm x biết (x + 1) + (x + 2) + (x + 3) + ... + (x + 100) = 5750.
b) Tìm x; y
Z biết 2
x
+ 124 = 5
y
.
c) Tìm kết quả của phép nhân A =
sc /100
6...666
.
sc/100
9...999
Câu 2 : (4 điểm)
a) Chứng minh rằng :
72
810
2014
+
là một số tự nhiên.
b) Cho
abc
7. Chứng tỏ rằng 2a + 3b + c
7
c) Cho các số tự nhiên từ 11 đến 21 được viết theo thttùy ý, sau đó đem cộng
mỗi số đó với số chỉ thứ tự của nó ta được một tổng . Chứng minh rằng trong các tổng
nhận được bao giờ cũng tìm ra hai tổng hiệu của chúng là một số chia hết cho 10.
Câu 3 : (2 điểm) Cho S =
49
5
...
23
5
22
5
21
5
20
5
+++++
. Chứng minh rằng 3 < S < 8.
Câu 4 : (4 điểm) m 3 số tổng bằng 420, biết rằng
7
6
số thứ nhất bằng
11
9
số thứ hai
và bằng
3
2
số thứ ba.
Câu 5 : (5 điểm)
a) Choc xOy bằng 80
0
, góc xOz bằng 30
0
. Tính số đo góc yOz ?
b) Cho 4 điểm A; B; C; D không nằm trên đường thẳng a. Chứng minh rằng đường
thẳng a hoặc không cắt, hoặc cắt ba, hoặc cắt bốn đoạn thẳng trong số các đoạn thẳng sau :
AB; AC; BC; BD; CD; AD.
Trang 50
PHÒNG GIÁO DỤC VÀ ĐÀO TẠO
ĐỀ KHẢO SÁT HỌC SINH GIỎI LỚP 6 CẤP HUYỆN
Môn: Toán Lớp 6
Thời gian làm bài: 120 phút (Không kể thời gian giao đề)
HƯỚNG DẪN CHM ĐỀ 13
Câu
Nội dung
Điểm
Câu 1
(5 điểm)
a) Ta có (x + 1) + (x + 2) + (x + 3) + ... + (x + 100) = 5750.
=> 100x + 101.50 = 5750
=> 100x = 700 = > x = 7
b) +) x = 0 => 2
0
+ 124 = 5
y
=> 125 = 5
y
=> 5
3
= 5
y
=> y = 3
+) x
0 => 2
x
+ 124 là số chẵn => 2
x
+ 124 = 5
y
là vô
Vậy x = 0 và y = 5 thì thỏa mãn đề bài.
c) A =
sc /100
6...666
.
sc/100
9...999
= A =
sc /100
6...666
.(1
sc /100
0...00
- 1)
=
sc /100
6...666
sc /100
0...000
-
sc /100
6...666
=
sc /99
6...666
5
sc/99
3...333
4
0,75
0,75
0,5
0,25
0,5
0,25
0,5
0,5
1,0
Câu 2
(4 điểm)
a) Chứng minh : 10
2014
+ 8
8
10
2014
+ 8
9
Mà (8; 9) = 1 => 10
2014
+ 8
72
=>
72
810
2014
+
là một số tự nhiên.
b)
abc
7 => 100a + 10b + c
7 => 98a + 7b + ( 2a + 3b + c)
7
=> 7(14a + b) + ( 2a + 3b + c)
7
Mà 7(14a + b)
7 => ( 2a + 3b + c)
7
c) Khi xét 1 số tự nhiên khi chia cho 10
=> Có thể xảy ra 10 trường hợp về số dư
9;...;2;1;0
(1)
Mà các số tự nhiên từ 11 --> 21 gồm (21 - ) + 1 = 11 số.
Biết mỗi số cộng với đúng số thứ tự của nó được 1 tổng
=> Có 11 tổng , mỗi tổng đều có giá trị là 1 số tự nhiên (2)
Từ (1) và (2) => Trong 11 tổng trên chắc chắn có 2tổng
cùng số dư khi chia cho 11
0,25
0,25
0,25
0,25
0,25
0,25
0,5
0,5
0,5
0,5
0,5
Trang 51
=> Luôn
hai tổng có hiệu chia hết cho 10.
Câu 3
(2 điểm)
Xét tổng S =
49
5
...
23
5
22
5
21
5
20
5
+++++
có 30 số hạng
50
5
49
5
;...;
50
5
22
5
;
50
5
21
5
;
50
5
20
5
=>
3
50
5
.30 = SS
(1)
Lại có :
20
5
49
5
;...;
20
5
22
5
;
20
5
21
5
;
20
5
20
5
=
=> S <
20
150
20
5
.30 =
=> S < 8 (2)
Từ (1) và (2) => 3 < S < 8.
0,25
0,5
0,25
0,5
0,25
0,25
Câu 4
(4 điểm)
Lập luận => Số thứ nhât bằng
22
21
số thứ hai.
Số thứ ba bằng
22
27
số thứ hai.
=> Tổng của ba số bằng
22
70
22
272122
=
++
số thứ hai
=> Số thứ hai là : 420 :
22
70
= 132
=> Số thứ nhất là :
126132.
22
21
=
=> Số thứ nhất là :
162132.
22
27
=
0,75
0,75
0,75
0,75
0,5
0,5
Câu 5
(5 điểm)
a) +) TH1: Hai tia Oy và Oz nằm trên hai nửa mp đối nhau b
chứa tia Ox :
Lập luận => Tia Ox nm giữa hai tia OyOz
Trang 52
=> góc yOz = 80
0
+ 30
0
= 110
0
+) TH2: Hai tia Oy và Oz cùng nằm trên một nửa mp bờ chứa tia
Ox
Lập luận => Tia Oz nằm giữa hai tia Ox và Oy .
=> góc yOz = 80
0
- 30
0
= 50
0
b) +) TH1: Bốn điểm A; B; C; D cùng thuộc một nửa mp bờ là a.
=> Đường thẳng a không cắt đoạn thẳng nào trong
các đoạn thẳng AB; AC; AD; BC; BD; CD
+) TH2: Trong hai nửa mp đói nhau bờ a, mỗi nửa mp chứa 2
trong bốn điểm A; B; C; D
=> Đường thẳng a cắt 4 đoạn thẳng trong số 6 đoạn
thẳng AB; AC; AD; BC; BD; CD.
+) TH2: Trong hai nửa mp đối nhau bờ a, một nửa mp chứa 1
điểm, nửa mp còn lại chứa 3 trong số bốn điểm A; B; C; D
=> Đường thẳng a cắt 3 đoạn thẳng trong số 6 đoạn
thẳng AB; AC; AD; BC; BD; CD.
Suy ra điều phải chứng minh .
0,5
0,5
0,5
0,5
0,5
0,5
0,5
0,5
0,5
0,5
O
y
z
x
z
x
y
O
Trang 53
Chú ý:
1. Thí sinh có thể làm bài bằng cách khác, nếu đúng vẫn đưc điểm tối đa.
2. Nếu thí sinh chứng minh bài nh mà không vẽ hình thì không chấm điểm bài hình.
PHÒNG GIÁO DỤC VÀ ĐÀO TẠO
ĐỀ KHẢO SÁT HỌC SINH GIỎI LỚP 6 CẤP HUYỆN
Môn: Toán Lớp 6
Thời gian làm bài: 120 phút (Không kể thời gian giao đề)
ĐỀ 14
Câu 1: ( 4 điểm)
1) Chứng minh rằng số A = 10
n
+ 18n - 1 chia hết cho 27 ( n là số tự nhiên)
2) Chứng tỏ rằng với mọi số tự nhiên n phân số sau tối giản:
16 3
12 2
n
n
+
+
Câu 2: (5 điểm)
1) Tìm các số nguyên x, y sao cho: (x - 1)(3 - y) = 2
2) Tìm tập hợp số nguyên x , biết :
(
3 6 1 2 1 3 2 3
1 ):(1 2 20%) 1 .1 3 :2
4 4 5 5 5 4 11 21
x + + +
3) Tìm số tự nhiên x biết:
1 1 1 2 2013
...
3 6 10 ( 1) 2015xx
+ + + + =
+
Câu3:(2điểm)
Chứng minh rằng : 1 +
1999
1 1 1
... 1000
2 3 2
+ + +
Câu 4: (4 điểm)
Sau buổi biểu diễn văn nghệ, ntrường tặng cam cho các tiết mục. Lần đầu tiết mục
đồng ca hết
5
6
số cam và
1
6
quả; lần 2 tặng tiết mục tốp ca hết
6
7
số cam còn lại và
1
7
quả;
lần 3 tặng tiết mục đơn ca hết
3
4
số cam còn lại lần 2
1
4
quthì vừa hết. Tính số cam
trường đó đã tặng và số cam riêng cho các tiết mục đồng ca, tốp ca và đơn ca.
Câu 5: ( 5 điểm)
Cho tia Ox. Trên hai nửa mặt phẳng đối nhau bờ Ox. Vẽ hai tia Oy Oz sao cho
c xOy và xOz bằng 120
0
. Chứng minh rằng:
a.
xOy xOz yOz==
b. Tia đối của mỗi tia Ox, Oy, Oz là phân giác của góc hợp bởi hai tia còn lại.
..........................................HẾT...........................................
Trang 54
PHÒNG GIÁO DỤC VÀ ĐÀO TẠO
ĐỀ KHẢO SÁT HỌC SINH GIỎI LỚP 6 CẤP HUYỆN
Môn: Toán Lớp 6
Thời gian làm bài: 120 phút (Không kể thời gian giao đề)
HƯỚNG DẪN CHM ĐỀ 14
Câu
Nội dung
Điểm
Câu 1
(4điểm)
1) Ta viết số A dưới dạng sau :
A = 10
n
+ 18n - 1 = 10
n
- 1 - 9n + 27n
=
99...9
n
- 9n + 27n = 9(
11...1
n
- n) + 27n
n là tổng các chữ số của
11...1
n
nên (
11...1
n
- n) 3
Từ đó
A 27
2) Gọi ƯCLN của tử và mẫu là d (d
N, d
1) thì 16n + 3 d
12n + 2 d
3(16n + 3) d và 4(12n + 2) d
Do đó 3(16n + 3) - 4(12n + 2) d
48n + 9 - 48n - 8 d
1 d
Vậy d = 1
Phân số
16 3
12 2
n
n
+
+
là phân số tối giản.
0,5đ
0,5đ
0,5đ
0,5đ
0,5đ
0,5đ
0,5đ
0,5đ
Câu 2
(5điểm)
1) Ta có: (x - 1)(3 - y) = 2
Vì 4 có các ước là - 2; -1; 1; 2 và
;x y Z
nên (x - 4), (3 - y)
Z
.Do
đó ta có:
+
11
32
x
y
=
=
11
32
x
y
= +
=+
0
5
x
y
=
=
.
+
12
31
x
y
=
=
21
31
x
y
= +
=+
1
4
x
y
=−
=
+
12
31
x
y
−=
−=
21
31
x
y
=+
=−
3
2
x
y
=
=
+
11
32
x
y
−=
−=
11
32
x
y
=+
=−
2
1
x
y
=
=
Vậy các cặp số nguyên x; y thỏa mãn là:
(x; y) =
(0;5);( 1;4);(3;2);(2;1)
.
0,5đ
0,5đ
0,5đ
Trang 55
2) (
3 6 1 2 1 3 2 3
1 ):(1 2 20%) 1 .1 3 :2
4 4 5 5 5 4 11 21
x + + +
7 6 1 2 1 12 7 32 245
( ):(1 2 ) . :
4 4 5 5 5 7 4 11 121
x + + +
14
:3
45
x
35 121
3.
11 245
+
5 32
76 7
x
mà x
Z
nên x
1;2;3;4
3. Nhân
1
2
vào hai vế ta được :
1 1 1 1 1 2013
... .
6 12 20 ( 1) 2 2015xx
+ + + + =
+
1 1 1 1 1 1 1 1 1 2013
... .
2 3 3 4 4 5 1 2 2015xx
+ + + + =
+
1 1 1 2013
.
2 1 2 2015x
−=
+
1 1 2013
.
2( 1) 2 2015
x
x
=
+
1 2013
1 2015
x
x
=
+
2015 2015 2013 2013xx = +
2 4028x=
2014x=
0,5đ
0,5đ
0,5đ
0,5đ
0,5đ
Câu 3
(2điểm)
1999
1 1 1
1 ...
2 3 2
+ + + + =
1 +
1
2
+ (
2
11
32
+
) + (
3
11
...
52
++
) + (
4
11
...
92
++
) +
…+(
1998 1999
11
...
2 1 2
++
+
)
> 1 +
2 3 1998
2 3 4 1999
1 1 1 1 1
.2 .2 .2 ... .2
2 2 2 2 2
+ + + + +
= 1+
1
2
. 1999 = 1000,5 > 1000 ( ĐPCM)
0,5đ
0,5đ
Câu 4
(4điểm)
- Nhận xét :
1
4
qucuối cùng chính
4 3 1
4 4 4
−=
số cam còn lại sau
lần 3. Vậy bài này phải tính ngược từ dưới lên.
Tiết mục đơn ca được tặng
1 1 1 4
:.
4 4 4 1
=
= 1 (quả).
Tương tự trên, tiết mục đơn ca và tốp ca được tặng :
(
1
1
7
+
) :
1
7
= 8 (quả).
Tương tự số cam của trường đó đã tặng :
(8 +
1
6
) :
1
6
= 49 (quả)
Số cam tặng tiết mục tốp ca : 8 - 1 = 7 (quả)
Số cam tặng tiết mục đồng ca : 49 - 8 = 41 (quả).
0,7
0,7
0,5đ
0,5đ
0,5đ
Câu 5
Trang 56
(5điểm)
z
,
y
x
,
O x
z
y
,
a) Kẻ tia Ox
,
là tia đối của tia Ox Ta có:
,
x Oy
+
yOx
= 180
0
(kề bù)
'
xOy
= 180
0
- 120
0
= 60
0
Tương tự:
'0
60xOz =
Ta có :
' 0 ' 0
60 , 60xOy xOz==
và tia Ox’ nằm giữa hai tia Oy, Oz nên
' ' 0
120yOz yOx xOz= + =
vậy
xOy yOz zOx==
b) Do tia Ox’ nằm giữa hai tia Oy, Oz và
''
xOy xOz=
nên Ox’ là tia phân giác của góc hợp bởi hai tia Oy, Oz.
-Tương tự tia Oy(tia đối của Oy) và tia Oz’ (tia đối của tia Oz) là
phân giác của góc xOz và xOy.
0,5đ
0,5đ
0,5đ
0,5đ
0,5đ
0,5đ
Chú ý:
1. Thí sinh có thể làm bài bằng cách khác, nếu đúng vẫn được điểm tối đa.
2. Nếu thí sinh chứng minh bài nh mà không vẽ hình thì không chấm điểm bài hình.
PHÒNG GIÁO DỤC VÀ ĐÀO TẠO
ĐỀ KHẢO SÁT HỌC SINH GIỎI LỚP 6 CẤP HUYỆN
Môn: Toán Lớp 6
Thời gian làm bài: 120 phút (Không kể thời gian giao đề)
ĐỀ 15
Bài 1( 4 điểm)
a) Cho A = 5 - 5
2
+ 5
3
- 5
4
+ …- 5
98
+ 5
99
. Tính tổng A.
b) Chứng tỏ (
2
n
+ 1).(
2
n
+ 2) chia hết cho 3 với mọi n là số tự nhiên.
Bài 2 ( 5 điểm)
a) Tìm các số nguyên x, y biết rằng : (x - 2)
2
.(y - 3) = - 4
b) Tìm n Z để (4n - 3) (3n 2)
Bài 3 ( 2 điểm)
Chứng minh
2 2 2 2 2 2
1 1 1 1 1 1 3
... 1
1 2 3 4 99 100 4
A = + + + + + +
Bài 4 ( 4 điểm)
Trang 57
Trong một buổi đi tham quan, số nđăng tham gia bằng
1
4
số nam. Nhưng sau đó
một bạn nữ xin nghỉ, một bạn nam xin đi thêm nên số nđi tham quan bằng
số nam.
Tính số học sinh nữ và nam đã đi tham quan.
Bài 5: (5 điểm)
Cho 4 tia chung gc theo th tự Ox, Oy, Oz, Ot sao cho
1
2
xOy zOt =
;
1
2
yOz xOy =
, biết số đo c zOt bằng 60
0
.
a) Tính số đo các góc xOy; yOz; tOx?
b) Vẽ tia Om sao cho số đoc mOt bằng 20
0
. Tính số đo góc zOm?
c) Vẽ thêm 10 tia phân biệt chung gốc với các tia Ox, Oy, Oz, Ot, Om. Hỏi bao
nhiêu góc tạo thành từ tất cả các tia trên?
-----------------------Hết-----------------------
PHÒNG GIÁO DỤC VÀ ĐÀO TẠO
ĐỀ KHẢO SÁT HỌC SINH GIỎI LỚP 6 CẤP HUYỆN
Môn: Toán Lớp 6
Thời gian làm bài: 120 phút (Không kể thời gian giao đề)
HƯỚNG DẪN CHM ĐỀ 15
Bài
Nội dung
Điểm
Bài 1
(4điểm)
Câu a( 2 điểm)
A = 5 5
2
+ 5
3
5
4
+ …- 5
98
+ 5
99
5A = 5
2
5
3
+ 5
4
- …+ 5
98
5
99
+ 5
100
Tính và rút gn được 6A = 5 + 5
100
100
55
6
A
+
=
Câu b ( 2 điểm)
Ta có:
2
n
. (
2
n
+1).(
2
n
+ 2) là tích của 3 số tự nhiên liên tiếp nên
chia hết cho 3.
2
n
không chia hết cho 3
nên (
2
n
+ 1).(
2
n
+ 2) 3 n N
0,5 đ
1 đ
0,5 đ
0,75 đ
0,5 đ
0,5đ
0,2
Bài 2
(5điểm)
Câu a( 3 điểm)
Ta có : -4 = 1
2
.(-4) = 2
2
.(-1) nên ta có c trường hợp sau:
TH1: ( x - 2)
2
= 12 và y - 3 = -4
x - 2 = 1 ; y = -1 hoặc x - 2 = -1; y = -1
x = 3; y = -1 hoặc x = 1; y = -1
TH2: ( x - 2)
2
= 2
2
và y - 3 = -1
0,5 đ
0,5 đ
0,5 đ
0,5 đ
Trang 58
x - 2 = 2 ; y = 2 hoặc x - 2 = -2; y = 2
x = 4; y = 2 hoặc x = 0; y = 2
KL: Vậy ta có các cặp (x, y) nguyên tha mãn là:
(3; -1); (1; -1); (4; 2); (0; 2)
Câu b( 2 điểm)
Ta có: 4n 3 + 3n 2
Mà 3n + 2 + 3n + 2
→ 3(4n 3) 4(3n 2) + 3n 2
→ ( 12n - 9 - 12n + 8 ) + 3n 2
- 1 + 3n 2
3n 2
Ư(-1)
+) 3n 2 = 1 suy ra n = 1
+) 3n 2 = -1 suy ra n = 1/3
Kết hợp điều kiện n nguyên ta được n = 1
0,5 đ
0,5 đ
0,5đ
0,7
0,2
0,2
0,2
Bài 3
(2điểm)
Giữ nguyên
22
11
12
+
Ta có:
2 2 2
1 1 1 1 1 1
; ;...;
3 2.3 4 3.4 100 99.100
2 2 2 2 2 2
22
1 1 1 1 1 1
...
1 2 3 4 99 100
1 1 1 1 1 1 1
( ) ( ... ) 1 '
1 2 2.3 3.4 4.5 99.100 4
1 1 1 1
( ' ... )
2.3 3.4 4.5 99.100
A
A
A
= + + + + + +
+ + + + + + = + +
= + + + +
Chứng minh A’ =
11
2 100
Do đó
1 1 1 3 1 3
1 1 1
4 2 100 4 100 4
A + + = +
0,25 đ
0,5 đ
0,5 đ
0,75 đ
Bài 4
(4điểm)
Tổng số học sinh nam và nữ dự định đi tham quan và đã đi tham
quan là như nhau, ta lấy làm đơn vị.
Số nữ dự định đi tham quan bằng
1
4
số nam nên bằng
1
5
tổng số
nam và nữ.
Số nữ đi tham quan bằng
1
5
số nam nên bằng
1
6
tổng số nam và nữ.
Số nữ dự định đi nhiều hơn số nữ đã đi là:
1 1 1
5 6 30
−=
( tổng số học
sinh) hay
1
30
tổng số học sinh tương ứng với 1 học sinh.
Tổng số học sinh là: 1 :
1
30
= 30 ( học sinh)
0,5 đ
0,5đ
0,5đ
0,5đ
0,5đ
0,5đ
Trang 59
Số học sinh nữ đã đi tham quan là: 30 .
1
6
= 5 (học sinh)
Số học sinh nam đã đi tham quan là: 30 5 = 25 ( học sinh)
Vậy có 5 học sinh nữ và 25 học sinh nam đi tham quan.
0,5đ
0,5đ
Bài 5
(5điểm)
Vẽ hình đúng u a, b được 0,5 điểm ( hs kng vẽ được hình không
tính điểm bài làm)
Câu a ( 1 điểm)
11
;
22
xOy zOt yOz xOy = =
0
60zOt=
nên
0 0 0 0
1 1 1
60 30 ; 30 15
2 2 2
xOy yOz xOy = = = = =
Tính được
0 0 0 0
30 15 60 105xOt xOy zOy zOt = + + = + + =
Câu b ( 2,5 điểm)
Ta có 2 trường hợp:
TH1: Tia Om nằm giữa tia Oz và tia Ot
0,5 đ
0,5 đ
0,5 đ
1,25 đ
x
O
y
z
t
0
60
Trang 60
Tính được
0 0 0
60 20 40zOm zOt tOm = = =
TH2: Tia Ot nằm giữa 2 tia Om và Oz
Tính được
0 0 0
20 60 80zOm mOt tOz = + = + =
Câu c ( 1 điểm)
Từ hai tia chung gốc ta vẽ được 1 góc.
Vẽ thêm 10 tia phân biệt gốc O kng trùng với các tia Ox, Oy, Oz,
Ot, Om. Tất cả trong hình vẽ có 15 tia phân biệt .
Cứ 1 tia trong 15 tia đó tạo với 14 tia còn lại thành 14 góc .
15 tia nên tạo thành 15.14 ( góc)
nhưng như thế mỗi góc được tính 2 lần .
nên có tất cả sốc tạo thành từ 15 tia phân biệt chung gốc là :
1,25 đ
0,5 đ
0,5 đ
x
O
y
z
t
0
20
m
x
O
y
z
t
0
20
m
Trang 61
15.14
105
2
=
góc
Chú ý:
1. Tsinh có thể m bài bằng cách khác, nếu đúng vẫn đưc điểm tối đa.
2. Nếu thí sinh chứng minh bài nh mà không vẽ hình thì không chấm điểm bài hình.
PHÒNG GIÁO DỤC VÀ ĐÀO TẠO
ĐỀ KHẢO SÁT HỌC SINH GIỎI LỚP 6 CẤP HUYỆN
Môn: Toán Lớp 6
Thời gian làm bài: 120 phút (Không kể thời gian giao đề)
ĐỀ 16
Bài 1: (4,0 điểm ) ,
1. Chứng tỏ rằng: 2x + 3y chia hết cho 17
9x + 5y chia hết cho 17.
2. Cho C = 3 + 3
2
+ 3
3
+ 3
4
………+ 3
100
chứng tỏ C chia hết cho 40.
3. Tìm các nguyên tố x, y thỏa mãn : (x-2)
2
.(y-3) = - 4
Bài 2 :(5,0đ)
Tìm x, biết:
1. a) 3
2x
= 81 ; b) 5
2x-3
2.5
2
= 5
2
.3
2. Tính
629199
920915
27.2.76.2.5
8.3.49.4.5
3. Tính tổng: B =
100.97
2
....
10.7
2
7.4
2
4.1
2
++++
4. Tìm số tự nhiên n để phân số
34
1938
+
+
=
n
n
A
Có giá trị là số tự nhiên.
Bài 3: (2,) Chứng minh rằng :
1
100
1
4
1
3
1
2
1
2222
++++
Bài 4: ( 4,0 điểm)
Tổng số trang của 8 quyển vloại 1 ; 9 quyển vở loại 2 và 5 quyển vloại 3 1980
trang. Strang của một quyển vloại 2 chỉ bằng
3
2
số trang của 1 quyển vở loại 1. Số
trang của 4 quyển vở loại 3 bằng số trang của 3 quyển vở loại 2. Tính số trang của mỗi
quyển vở mỗi loại.
Bài 5: (5,0đ) Cho tam giác ABC BC = 5cm. Điểm M thuộc tia đối của tia CB sao cho
CM = 3 cm.
a. Tình đdài BM
b. Cho biết góc BAM = 80
0
, góc BAC = 60
0
. Tínhc CAM.
c. Vẽ các tia Ax, Ay lần lượt là tia phân giác của góc BAC và CAM . Tính góc xAy.
d. Lấy K thuộc đoạn thẳng BM và CK = 1 cm. Tính độ dài BK.
Trang 62
PHÒNG GIÁO DỤC VÀ ĐÀO TẠO
ĐỀ KHẢO SÁT HỌC SINH GIỎI LỚP 6 CẤP HUYỆN
Môn: Toán Lớp 6
Thời gian làm bài: 120 phút (Không kể thời gian giao đề)
HƯỚNG DẪN CHM ĐỀ 16
Bài
Nội dung
Điểm
Bài 1
(4điểm)
1. Ta có 4 (2x + 3y ) + ( 9x + 5y ) = 17x + 17y chia hết cho 17
Do vậy ; 2x + 3y chia hết cho 17
4 ( 2x +3y ) chia hết cho 17
9x + 5y chia hết cho 17
Ngược lại Ta có 4 ( 2x + 3y ) chia hết cho 17 mà ( 4 ; 17 ) = 1
2x + 3y chia hết cho 17
2. B = (3 + 3
2
+ 3
3
+ 3
4
) +……+ (3
97
+3
98
+3
99
+3
100
)
= 3 (1 + 3 + 3
2
+3
3
)+…….+ 3
97
(1+3+3
2
+3
3
)
= 40. (3 + 3
5
+3
9
+………+3
97
) : 40
3. Do 4 = 1
2
. (- 4) = 2
2
.(-1) nên có c trường hợp sau:
a.
=
=
=
=
=
=
1
3
1
12
43
1)2(
2
y
x
y
x
y
x
hoặc
=
=
=
=
1
1
1
12
y
x
y
x
b.
=
=
=
=
=
=
2
4
2
22
13
2)2(
22
y
x
y
x
y
x
hoặc
=
=
=
=
2
0
2
22
y
x
y
x
1,0đ.
0,5đ.
0,5đ
1,0đ.
1,0đ
Bài 2
(5điểm)
1 .a) 3
2x
= 81 => 3
2x
= 3
4
=> 2x = 4 => x = 2
b). 5
2x-3
2.5
2
= 5
2
.35
2x
: 5
3
= 5
2
.3 + 2.5
2
5
2x
: 5
3
= 5
2
.5
5
2x
= 5
2
.5.5
3
5
2x
= 5
6
=> 2x = 6 => x = 3
2 .
30 18 2 20 27 29 18
9 19 19 29 18 28 18
5.2 .3 2 .3 .2 2 .3 (5.2 3)
2
5 .2 .2 .3 7.2 .3 2 .3 (5.3 7.2)
−−
==
−−
3. Ta có
)
4
1
1
1
(
3
2
4.1
2
)
4
1
1
1
(
3
1
4.1
1
==
);....
10
1
7
1
(
3
2
10.7
2
);
7
1
4
1
(
3
2
7.4
2
==
......;
)
100
1
99
1
(
3
2
100.97
2
=
B=
)
100
1
99
1
.....
10
1
7
1
7
1
4
1
4
1
1
1
(
3
2
++++
B=
50
33
100
99
.
3
2
)
100
1
1
1
(
3
2
==
4.
34
187
2
34
187)34(2
34
1938
+
+=
+
++
=
+
+
=
nn
n
n
n
A
Để A
N thì 187
4n + 3 => 4n +3
187;11;17
+ 4n + 3 = 11 -> n = 2
+ 4n +3 = 187 --> n = 46
+ 4n + 3 = 17 -> 4n = 14 -> không có n
N
0,5đ.
0,5đ.
0,5đ.
1,0đ.
0,5đ.
0,5đ.
0,5đ.
0,5đ.
0,5đ.
Trang 63
Vậy n = 2; 46
Bài 3
(2điểm)
Ta có:
;
2
1
1
1
2.1
1
2
1
2
=
;
3
1
2
1
3.2
1
3
1
2
=
;100
1
99
1
100.99
1
100
1
;...;
4
1
3
1
4.3
1
4
1
22
==
Vậy
++++
2222
010
1
4
1
3
1
2
1
=++++
100.99
1
4.3
1
3.2
1
2.1
1
1 1 1 1 1 1 1
1
2 2 3 3 4 99 100
= + + + +
1 99
1 1.
2 100
= =
0,5đ.
0,5đ.
0,5đ.
0,5đ.
Bài 4
(4điểm)
số trang của mỗi quyển vỡ loại 2 bằng
3
2
số trang của 1 quyển
loại 1. Nên số trang của 3 quyển loại 2 bằng số trang của 2 quyển
loại 1
Mà số trang của 4 quyển loại 3 bằng 3 quyển loại 2.
Nê số trang của 2 quyển loại 1 bằng số trang của 4 quyển loại 3
Do đó số trang của 8 quyển loại 1 bằng :4 .8 : 2 = 16 ( quyển loi 3)
Số trang của 9 quyển loại 2 bằng9 .4 : 3 = 12 (quỷên loại 3)
Vậy 1980 chính là số trang của 16 + 12+ 5 = 33(quyển loại 3)
Suy ra: Số trang 1 quyển vở loại 3 là 1980 : 33 = 60 ( trang)
Số trang 1 quyển vloại 2
80
3
4.60
=
(trang)
Số trang 1 quyển vloại1 là;
120
2
3.80
=
( trang)
0,5đ.
1,0đ.
1,0đ.
0,5đ.
1,0đ.
Bài 5
(5điểm)
a. M, B thuộc 2 tia đối nhau CB và CM
-> C nằm giữa B và M. -> BM = BC + CM = 8 (cm)
b. C nằm giữa B,M -> Tia AC nm giữa tia AB, AM
->
CAM =
BAM -
BAC = 20
0
c. Có
xAy =
x AC +
CAy =
2
1
BAC +
2
1
CAM
=
2
1
(
BAC +
CAM) =
2
1
BAM =
2
1
.80 = 40
0
d. +) Nếu K
tia CM -> C nằm giữa B và K
1
-> BK
1
= BC + CK
1
= 6 (cm)
+)Nếu K
tia CB -> K
2
nằm giữa B và C
-> BK
2
= BC = CK
2
=4 (cm)
0,7
0,5đ.
0,7
1,0đ
1,0đ
1,0đ
B
C
y
x
M
A
Trang 64
Chú ý:
1. Thí sinh có thể làm bài bằng cách khác, nếu đúng vẫn đưc điểm tối đa.
2. Nếu thí sinh chứng minh bài hình mà không vẽ hình thì không chấm điểm bài hình.
3.
Chấm và cho điểm từng phần, điểm của toàni tổng các điểm thành phần không làm tròn.
NG GIÁO DỤC VÀ ĐÀO TẠO
ĐỀ KHẢO SÁT HỌC SINH GIỎI LỚP 6 CẤP HUYỆN
Môn: Toán Lớp 6
Thời gian làm bài: 120 phút (Không kể thời gian giao đề)
ĐỀ 17
i 1: (4 điểm):
Cho biểu thức A =
5
2n
a, Tìm các số nguyên n để biểu thức A là phân số.
b, Tìm các số tự nhiên n để biểu thức A là số nguyên
Bài 2: (5 điểm):
1. Tính nhanh A= 3.136.8 + 4. 14.6 -14.150
11 5 4 4 8
B:
4 9 9 11 33

=


2. Tìm x biết
a/ 3 + 2
x -1
= 24 [4
2
(2
2
- 1)]
b/
5x
= 18 + 2.(-8)
i 3: (5 điểm)
1.Cho đoạn thng AB, điểm O thuc tia đối của tia AB. Gọi M,N thứ tự trung điểm ca OA,
OB.
d) Chng tỏ rằng OA < OB.
e) Trong ba điểm O, M, N điểm nào nằm giữa hai điểm còn lại ?
2. Cho c AOB và góc BOC là hai góc kề bù . Biết c BOC bằng năm lần góc AOB.
a) Tính sđo
AOB,BOC
.
b) Gọi OD là tia phân giác của góc BOC. Tính số đo góc AOD.
i 4. (4 điểm):
Bạn An nghĩ ra một số có 3 chữ s, nếu bớt số đó đi 8 đơn vị tđược một s chia hết cho 7, nếu
bớt số đó đi 9 đơn vị thì được một số chia hết cho 8, nếu bớt số đó đi 10 đơn vị thì được 1 số chia
hết cho 9. Hỏi bạn An nghĩ ra số nào?
i 5. (2 điểm) :
Chng minh rằng
2 2 2 2 2 2
1 1 1 1 1 1
... 1
2 3 4 5 2011 2012
+ + + + + +
------------------------------- Hết --------------------------------
Trang 65
PHÒNG GIÁO DỤC VÀ ĐÀO TẠO
ĐỀ KHẢO SÁT HỌC SINH GIỎI LỚP 6 CẤP HUYỆN
Môn: Toán Lớp 6
Thời gian làm bài: 120 phút (Không kể thời gian giao đề)
HƯỚNG DẪN CHM ĐỀ 17
BÀI
ĐÁP ÁN
ĐIỂM
i 1
(4,0điểm)
a/ n
Z và n
2
b/ (n - 2 ) Ư( -5) =
1; 5
2 1 1
2 1 3
2 5 3
2 5 7
n n N
n n N
n n N
n n N
= =


= =



= =

= =

Vậy n = 1;3;7
2
1
0,5
0,5
i 2
(5 điểm)
1
A= 24.136 + 24.14 - 14.150 = 24.(136 + 14)- 14.150
= 24.150 - 14.150= 150. (24- 14)=150.10 =150
0,5
1
11 5 4 11 8 11 5 4 8 11 8 2
B1
4 9 9 4 33 4 9 9 33 4 33 3
−−
= = = =
1
2
a) 3 + 2
x-1
= 24 [4
2
(2
2
- 1)]
3 + 2
x-1
= 24 4
2
+ 3
2
x-1
= 24 4
2
= 2
2
x -1 = 2
x = 3
0,5
0,5
0,5
b/ x = 7 hoặc x = 3;
1
i 3
(5đ)
1.( 2điểm)
Hai tia AO, AB đối nhau, nên điểm A nằm giữa hai đ O và B
OA< OB.
Ta có M và N thứ tự là trung điểm ca OA, OB, nên :
Vì OA < OB, nên OM < ON.
Hai điểm M và N thuộc tia OB, mà OM < ON nên điểm M nằm giữa hai
điểm O và N.
2.(3 điểm)
Vẽ hình đúng
a)
AOB
BOC
hai c kề bù nên:
0
AOB BOC 180+=
BOC 5.AOB=
nên: 6
AOB
= 180
0
Do đó:
AOB
= 180
0
: 6 = 30
0
;
BOC
= 5. 30
0
= 150
0
b)Vì OD là tia phân giác ca
BOC
nên
BOD
=
DOC
=
BOC
=75
0
.
Vì góc
AOD
và góc
DOC
hai góc kề bù nên:
0
AOD DOC 180+=
Do đó
AOD
=180
0
DOC
= 180
0
- 75
0
= 105
0
KL:
0,5
0,5
0,5
0,5
0,5
0,5
0,5
0,5
0,5
0,25
0,25
b
m
n
a
o
OA OB
OM ; ON
22
= =
2
1
A
B
C
O
D
Trang 66
i 4
(4,0điểm)
Gọi số bạn An nghĩ ra A
Vì (A-8) 7 (A-1) - 7 7 (A-1) 7
Vì (A-9) 8 (A-1) - 8 8 (A-1) 8
Vì (A-10) 9 (A-1) - 9 9 (A-1) 9
Do đó: (A-1) là bội chung của 7,8,9 và A là số có 3 chữ số
nên
99 < A < 1000
Từ đó giải và tìm được A-1 = 504Suy ra :A= 505
1
0,5
0,5
1
0,5
0,5
i 5
(2,0đ)
Ta có
2
11
2 1.2
;
2
11
3 2.3
;
2
11
4 3.4
; … ;
2
11
2012 2011.2012
0,5
2 2 2 2 2
1 1 1 1 1 1 1 1 1
... ...
2 3 4 2011 2012 1.2 2.3 3.4 2011.2012
+ + + + + + + + +
0,5
2 2 2 2 2
1 1 1 1 1 1 1 1 1 1 1 1 1
... ...
2 3 4 2011 2012 1 2 2 3 3 4 2011 2012
+ + + + + + + + +
0,5
2 2 2 2 2
1 1 1 1 1 1 1
...
2 3 4 2011 2012 1 2012
+ + + + +
=
2011
2012
< 1
0,5
Chú ý:
1. Thí sinh có thể làm bài bằng cách khác, nếu đúng vẫn đưc điểm tối đa.
2. Nếu thí sinh chứng minh bài nh mà không vẽ hình thì không chấm điểm bài hình.
PHÒNG GIÁO DỤC VÀ ĐÀO TẠO
ĐỀ KHẢO SÁT HỌC SINH GIỎI LỚP 6 CẤP HUYỆN
Môn: Toán Lớp 6
Thời gian làm bài: 120 phút (Không kể thời gian giao đề)
ĐỀ 18
i 1: ( 2.5 điểm)
a. Cho
số có sáu chữ số. Chứng tỏ số
ababab
bội của 3.
b. Cho S = 5 + 5
2
+ 5
3
+ 5
4
+ 5
5
+ 5
6
…+ 5
2004
. Chng minh S chia hết cho 126 và chia hết cho 65.
i 2 : (2,0 điểm)
Tìm số tự nhiên x biết :
a.
2029099 2010) (x 2)(x 1)(x x =+++++++
b.
210 2x 8 6 4 2 =+++++
Câu 3: (2.0 điểm)
a. Tìm tất cả các số nguyên tp sao cho p + 11 cũng là số nguyên t.
b. Tìm tất cả các số nguyên tố p để p + 8, p + 10 cũng các số nguyên tố.
câu 4 : ( 1.5 điểm)
một phép chia có thương bằng 5 và số dư là 12. nếu lấy số bị chia chia cho tổng số chia và số
ta được thương là 3 và sdư là 18. tìm s bị chia.
Câu 5: (2.0 điểm)
Tn đoạn thẳng AB = 3 cm lấy điểm M. Trên tia đối của tia AB lấy điểm N sao cho AM
= AN.
a. Tính đdài đoạn thẳng BN khi BM = 1 cm.
b. Hãy xác định vị trí của M (trên đon thẳng AB) để BN có đdài ln nhất.
ababab
Trang 67
PHÒNG GIÁO DỤC VÀ ĐÀO TẠO
ĐỀ KHẢO SÁT HỌC SINH GIỎI LỚP 6 CẤP HUYỆN
Môn: Toán Lớp 6
Thời gian làm bài: 120 phút (Không kể thời gian giao đề)
HƯỚNG DẪN CHM ĐỀ 18
i 1: ( 2.5 điểm)
-
ababab
=
ab
.10000 +
ab
.100 +
ab
= 10101
ab
.
0,50
- Do 10101 chia hết cho 3 nên
ababab
chia hết cho 3 hay
ababab
bội
của 3.
0,50
: 5 + 5
2
+ 5
3
+ 5
4
+ 5
5
+ 5
6
= 5(1 + 5
3
) + 5
2
(1 + 5
3
) + 5
3
(1 + 5
3
)
= 5. 126 + 5
2
.126
+ 5
3
.126
5 + 5
2
+ 5
3
+ 5
4
+ 5
5
+ 5
6
chia hết cho 126.
0,50
S = (5 + 5
2
+ 5
3
+ 5
4
+ 5
5
+ 5
6
) + 5
6
(5 + 5
2
+ 5
3
+ 5
4
+ 5
5
+ 5
6
) + … + 5
1998
(5 + 5
2
+ 5
3
+ 5
4
+ 5
5
+ 5
6
).
Tổng tn có (2004: 6 =) 334 số hạng chia hết cho 126 nên nó chia hết cho 126.
0,25
: 5 + 5
2
+ 5
3
+ 5
4
= 5+ 5
3
+ 5(5 + 5
3
) = 130 + 5. 130.
5 + 5
2
+ 5
3
+ 5
4
chia hết cho 130 .
0,25
S = 5 + 5
2
+ 5
3
+ 5
4
+ 5
4
(5 + 5
2
+ 5
3
+ 5
4
) + … + 5
2000
(5 + 5
2
+ 5
3
+ 5
4
)
Tổng tn có (2004: 4 =) 501 số hạng chia hết cho 130 nên nó chia hết cho 130.
0,25
S chia hết cho 130 nên chia hết cho 65.
0,25
i 2 : (2,0 điểm)
-
2029099 2010 21 2011x =++++
0,25
-
2029099
2
2011.2010
2011 =+x
0,25
-
2
2011.2010
- 20290992011 =x
0,25
-
=
= 2011:
2
2011.2010
- 2029099x
4
0,25
-
210 x) 3 2 2(1 =++++
0,25
-
210
2
)1(
2 =
+xx
0,25
-
210)1( =+xx
0,25
- Giải được x = 14 (Do 210 = 2.3.5.7 = 14.15)
0,25
Câu 3: (2.0 điểm)
a) - Nếu p lẻ p + 11 là số chẵn ln hơn 11 nên kng là số nguyên tố.
0,25
- Suy ra p chẵn p = 2.
0,25
Trang 68
b) - Nếu p chia 31 tp + 8 là số lớn hơn 3 và chia hết cho 3 nên kng là số
nguyên tố.
0,25
- Nếu p chia 3 2 thì p + 10 số lớn hơn 3 chia hết cho 3 n không số
nguyên tố.
0,25
- Suy ra p chia hết cho 3, p nguyên tố nên p = 3.
0,5
C©u4 (1.5 điểm)
i sè bÞ chia lµ a; sè chia lµ b (b 0)
PhÐp chia th-¬ng b»ng 5 sè d- lµ 12 a = 5b+12
0,5
Sè bÞ chia chia chong sè chia vµ sè d-®-îc th-¬ng lµ 3 vµ sè d- lµ 18
a = (b +12). 3 + 18 = 3b + 54
0,5
5b + 12 = 3b + 54 b = 21 a = 117
VËy sè chia lµ 117.
0,5
Câu 5:
- Hình vẽ:
i 5: Vẽ hình (0,25đ)
- M nằm giữa hai điểm A, B nên MA = AB - MB = 3 - 1 = 2 (cm)
0,25
- AN = AM = 2 (cm)
0,25
- A nằm giữa hai điểm N, B nên BN = AN + AB = 2 + 3 = 5 (cm).
0,25
- BN = AN + AB, AB không đổi nên BN ln nhất khi AN lớn nhất.
0,25
- AN lớn nhất khi AM lớn nhất.
0,25
- AM lớn nhất khi AM = AB.
0,25
- Lúc đó M trùng với B và BN bằng 6(cm).
0,25
CHÚ Ý :
- Nếu HSm cách khác đúng thì vẫn cho điểm tối đa theo thang điểm của ý đó
- Khi học sinhm bài phải lý luận chặt chẽ mới cho điểm tối đa theo biểu điểm của ý
đó
PHÒNG GIÁO DỤC VÀ ĐÀO TẠO
ĐỀ KHẢO SÁT HỌC SINH GIỎI LỚP 6 CẤP HUYỆN
Môn: Toán Lớp 6
Thời gian làm bài: 120 phút (Không kể thời gian giao đề)
ĐỀ 19
Câu 1 (4 điểm).
a. Tính g trị của biểu thức
930
1
...
12
1
6
1
2
1
5
1
4
2
17
2
1
9
5
31
31
1
+++++
+
=A
N
A
M
B
Trang 69
b. Tính giá trị của biểu thức B biết: B
2
= c(a-b)- b(a-c) a = -50, b-c =2.
Câu 2. (4 điểm)
a. Tìm số tự nhiên x,y biết: (2x+1)(y-3)= 12
b. Tìm số tự nhn x biết:
c. So sánh: 36
25
và 25
36
Câu 3. (3 điểm)
Cho phân s:
a. Chứng minh rằng phân sp là phân số tối giản
b. Với g trnào của n thì pn số p có giá trị lớn nhất? tìm gtrị ln nhất đó.
Câu 4. (7,5 điểm)
1. Cho hai góc kgóc xOy và góc yOt, trong đó xOy =40
0
. Gọi Om tia phân giác của góc
yOt.
a. Tính góc mOx ?
b. Trên nửa mặt phẳng không chứa tia Oy bờ đường thẳng chứa tia Ox, vẽ tia On
sao cho góc xOn=70
0
. Chứng t tia Om và tia On là hai tia đối nhau
2. Vẽ đoạn thẳng AB =6cm. Lấy hai điểm C và D nằm giữa A và B sao cho AC+BD= 9cm
a. Chứng tỏ D nằm giữa A và C
b. Tính đdài đoạn thẳng CD
Câu 5. (1,5 điểm)
Tìm các số nguyên dương x, y thỏa mãn : 2x+3y= 14
---------------- Hết ----------------
PHÒNG GIÁO DỤC VÀ ĐÀO TẠO
ĐỀ KHẢO SÁT HỌC SINH GIỎI LỚP 6 CẤP HUYỆN
Môn: Toán Lớp 6
Thời gian làm bài: 120 phút (Không kể thời gian giao đề)
HƯỚNG DẪN CHM ĐỀ 19
Câu
Nội dung
Điểm
Câu 1
a.
1
0.5
0.5
0.5
b. B
2
= c(a-b)-b(a-c) = ca-cb-ba+bc=ca-ba=a(c-b)
0.5
822...222
2019201521
=++++
+++ xxxx
)(
23
56
Nn
n
n
p
+
+
=
31
47
31
30
31
17
31
30
31
1
1
31
1
30
1
...
4
1
3
1
3
1
2
1
2
1
31.30
1
...
4.3
1
3.2
1
2
1
930
1
...
12
1
6
1
2
1
:
31
17
10
2131
31
17
5
21
2
17
2
17
5
31
31
1
5
1
4
2
17
2
1
9
5
31
31
1
:
930
1
...
12
1
6
1
2
1
5
1
4
2
17
2
1
9
5
31
31
1
=+=+=
==
++++=
++++=++++=
=
=
=
+
=
=+++++
+
=
NMA
NXet
MXet
A
Trang 70
thay a=-50, b-c=2o ta được B
2
=-50.(-2)=100
do nên B=10
0.5
0.5
Câu 2
a. (1,5 điểm)
(2x+1)(y-3)= 12
Với 2x+1 là số lẻ.
Ta có: 12 =1.12=3.4
2x+1=1 2x=0 x=0; y-3=12 y=15
2x+1=3 2x=2 x=1; y-3=1 y=4
Vậy x=0 và y=1 hoặc x=1 và y=4
0.25
0.25
0.25
0.25
0. 5
b. (1,25 điểm)
Ta có :
0.25
0.25
0.25
0.25
0.25
c. (1 điểm)
36
25
= (18.2)
25
=18
25
.2
25
=18
25
.2
6
.2
19
25
36 =
25
25
.25
11
= 25
25
.5
22
= 25
25
.5
3
.5
19
ta có: 5
3
=125, 2
6
=64, 5
3
>2
6
25
25
>18
25
; 5
19
>2
19
Vậy 25
25
.5
3
.5
19
>18
25
.2
6
.2
19
hay 36
25
<25
36
0.25
0.25
0.25
0. 5
Câu 3
a. Gọi d là UC của 6n+5 và 3n+2
ta có:
Vậy phân s phân số tối giản
0.25
0.25
0.25
0.25
0.25
b. Ta có
p đạt giá trị lớn nhất khi đạt giá trị lớn nhất, khi đó 3n+2 đạt giá tr
nhỏ nhất
nên 3n+2 nhỏ nhất bằng 2 khi 3n=0 hay n=0
Vậy với n=0 thì p đạt giá trị lớn nhất là 2+1/2=3/2
0.5
0.5
0.5
0.25
Câu 4
1(4 điểm).
a. Ta có xOy + yOt=180
0
(Vì 2 góc kề bù)
Thay xOy = 40
0
ta có:
Vẽ
hình
0,5
0.25
NB
Nyx ,
( )
3
22
)12(22282)12(2
12122
2...2222
2...221:
822...2212
822...222
3
201633201920192016
20162016
201632
201521
2019201521
2019101521
=
=
===
==
++++=
++++=
=++++
=++++
+++
x
CCC
C
CXet
x
x
x
xxxx
d 56n +
d 23n +
1d
d1d4)(6n-56n
d46nhay d 2)3n(2d 23n
=
++
+++
)(
23
56
Nn
n
n
p
+
+
=
23
1
2
23
146
23
56
+
+=
+
++
=
+
+
=
nn
n
n
n
p
23
1
+n
223n +
Trang 71
40
0
+yOt= 180
0
suy ra yOt=140
0
Ta có: Om là tia phân gc của tOy nên
Vì 2 góc xOy và yOt kề bù nên Ox và Ot hai tia đối nhau
suy ra tOmmOx là hai góc kề
tOm + mOx = 180
0
70
0
+ mOx = 180
0
mOx = 180
0
-70
0
= 110
0
b. Ta có mOx+ xOn = 110
0
+ 70
0
=180
0
mOx và xOn là hai góc bù nhau (1)
- Do Om và Oyng thuộc nửa mp có bờ là đường thẳng chứa tia Ox; -
Lại có On và Oy nằm trên hai nửa mặt phẳng đi nhau có bờ là đường
thẳng chứa tia Ox
nên: Om và On nằm trên hai nửa mặt phẳng đối nhau có bờ là đường
thẳng chứa tia Ox
mOx và xOn là hai góc kề nhau (2)
Từ (1) và (2) suy ra mOxxOn là hai góc kề bù.
0. 5
0.25
0. 5
0.5
0.5
0.5
0.5
2. (3,5đ)
- D nằm giữa A và B nên: AD+DB=AB
Thay AB= 6cm ta có AD+DB = 6 (cm)
Lại có AC+DB=9cm (gt)
AD+DB< AC+DB hay AD<AC (1)
- D và C cùng nằm giữa A và B hay D,C cùng thuộc tia AB (2)
Từ (1) và (2) suy ra D nằm giữa A và C
b, Vì D nằm giữa A và C suy ra: AD+DC= AC
Lại có AC+BD= 9
nên AD+DC+BD = 9 hay (AD+DB)+DC =9
Thay (AD+DB)=6
ta có 6+DC=9 vậy DC= 3(cm)
Vẽ
hình
0.5
0.25
0.25
0.25
0.25
0.25
0.5
0.25
0.5
0.5
Câu 5
Xét 2x+5y= 14
Ta có:
Do (5,2)=1 nên
Ta có 3y<14 y<14 :5
Mà y là số nguyên dương nên y = 2
ta có 2x+5.2=14 2x=4 x=2
vậy x=2, y=2
0.25
0.25
0.25
0.25
0.25
0.25
CHÚ Ý :
- Nếu HS làm cách kc mà đúng thì vn cho điểm tối đa theo thang điểm ca ý đó
00
70140
2
1
2
1
=== tOytOm
A
B
D
C
2522;214 yx
2y
2y
2y
Trang 72
- Khi học sinh làm bài phải lý luận chặt chẽ mới cho điểm tối đa theo biểu điểm của ý đó
---------------- Hết ----------------
PHÒNG GIÁO DỤC VÀ ĐÀO TẠO
ĐỀ KHẢO SÁT HỌC SINH GIỎI LỚP 6 CẤP HUYỆN
Môn: Toán Lớp 6
Thời gian làm bài: 120 phút (Không kể thời gian giao đề)
ĐỀ 20
i 1 (3,0 điểm)
Cho tổng A = 1 + 3
2
+ 3
4
+ 3
6
+…+ 3
2008
Tính giá trị biểu thức: B = 8A - 3
2010
i 2 (4,0 điểm)
Cho A = 1.4.7.10..…58 + 3.12.21.30…..174
a. Tìm chữ số tận cùng của A.
b. Chứng tỏ rằng A chia hết cho 377.
i 3 (4,0 điểm)
Tìm số tự nhiên x biết:
a. x + (x + 1) + (x + 2) + …+ (x + 99) = 5450.
b. 3.(5
x
- 1) - 2 = 70.
c. 2
x
+ 2
x + 1
+ 2
x + 2
= 960 - 2
x + 3
i 4 (4,0 điểm)
a. Tìm số tự nhiên hai chữ số khác nhau. Biết rằng: hai chữ số của số đó đều số nguyên tố.
Tích của số đó với c chữ số của nó là số có 3 chữ số ging nhau được tạo thành từ chsố hàng
đơn vị của s đó.
b. Cho p số nguyên tố (p > 3) và 2p + 1 cũng snguyên t. Hỏi 4p + 1 số nguyên t hay
hợp số? Vì sao?
i 5 (5,0 điểm)
Cho n đường thẳng trong đó bất cứ hai đường thẳng nào ng cắt nhau, không có ba đường thẳng
nào cùng đi qua mt điểm.
a. Biết rằng số giao điểm của các đường thẳng đó là 1128. Tính n.
b. Số giao điểm ca các đường thẳng đó có thể là 2017 được kng? Vì sao?
………….. Hết …………
Trang 73
PHÒNG GIÁO DỤC VÀ ĐÀO TẠO
ĐỀ KHẢO SÁT HỌC SINH GIỎI LỚP 6 CẤP HUYỆN
Môn: Toán Lớp 6
Thời gian làm bài: 120 phút (Không kể thời gian giao đề)
HƯỚNG DẪN CHM ĐỀ 20
i 1
(3,0điểm)
Hướng dẫn giải
Điểm
A = 1 + 3
2
+ 3
4
+ 3
6
+…+ 3
2008
9A = 3
2
+ 3
4
+ 3
6
+ 3
8
+… + 3
2010
1,0
Tính được 8A = 3
2010
- 1
1,0
B = 8A - 3
2010
= 3
2010
- 1 - 3
2010
= -1
1,0
i 2
(4,0điểm)
a,
(2,0 điểm)
Tìm chữ số tn cùng của A
- Tìm đưc chữ số tậnng của tích B = 1.4.7.10…58 0
0,75
- Tìm đưc chữ số tậnng của tích C = 3.12.21.30…174 0
0,75
- Tìm đưc và kết luận chữ số tận cùng của A là 0
0,5
b,
(2,0 điểm)
Chng tỏ rằng A chia hết cho 377
- Nhận xét 377 = 13.29
0,5
- Tìm được quy luật của c thừa số trong tích B là các số tự nhiên chia
3 dư 1, nên B chứa thừa số 13. Do đó B = 1.4.7.10.13…58
B = 1.4.7.10.13…29.2
Suy ra B chia hết cho 377
0,5
- Tìm được quy luật của c thừa số trong tích C là các số tự nhiên chia
9 dư 3, nên C chứa thừa số 39. Do đó C = 3.12.21.30.39…174
C = 3.12.21.30.(3.13)…(6.29)
Suy ra C chia hết cho 377
0,5
- Kết luận A chia hết cho 377
0,5
i 3
(4,0điểm)
a,
(1,5 điểm)
x + (x + 1) + (x + 2) + …+ (x + 99) = 5450.
100x + (1 + 2+ 3+ …+ 99) = 5450
0,5
luận tính tổng: 1 + 2+ 3+ …+ 99 = 4950
khi đó 100x + 4950 = 5450
0,5
100x = 500
0,25
x = 5
0,25
b,
(1,5 điểm)
3.(5
x
- 1) - 2 = 70.
3.(5
x
- 1) = 70 + 2
3.(5
x
- 1) = 72
0,5
5
x
- 1 = 72 : 3
5
x
- 1 = 24
5
x
= 25
0,5
5
x
= 5
2
x = 2
0,5
Trang 74
c,
(1,0 điểm)
2
x
+ 2
x + 1
+ 2
x + 2
= 960 - 2
x + 3
2
x
(1 + 2 + 2
2
+ 2
3
) = 960
2
x
.15 = 960
2
x
= 960: 15
0,5
2
x
= 64
2
x
= 2
6
x = 6
0,5
i 4
(4,0điểm)
a,
(2,0 điểm)
Tìm số tự nhiên có hai chữ skhác nhau…
- Gi số cần tìm là
ab
, (điều kiện của a, b…)
0,25
- Theo đề bài ta có
ab
.a.b =
bbb
Suy ra
ab
.a.b = 111.b
Hay
ab
.a = 111
0,75
Mà 111 = 3.37
Trong đó: 3 là số nguyên t; 7 là số nguyên tố; 3 7 thỏa mãn đề bài
nên
ab
= 37
0,75
Kết luận số cầnm là 37
0,25
b,
(2,0 điểm)
Cho p là số nguyên t (p > 3) và 2p + 1 cũng là số nguyên tố. Hỏi 4p + 1
số nguyên tố hay hợp số? Vì sao?
Vì p là số nguyên tố lớn hơn 3 nên p dạng 3k + 1 hoặc 3k + 2 (với k
N, k
1)
0,5
Nếu p = 3k +1 thì 2p + 1 = 2(3k + 1) + 1 = 3(2k + 1)
và lí luận chỉ ra 2p + 1 hợp số, trái với đề bài
0,75
Do đó p = 3k + 2 khi đó 4p + 1 = 4(3k + 2) + 1 = 3(4k + 3)
và lí luận chỉ ra 4p + 1 hợp s
0,75
Kl…..
i 5
(5,0điểm)
a,
(3,0 điểm)
Với n đưng thẳng trong đó bất cứ hai đường thẳng nào ng cắt nhau,
không có ba đưng thẳng nào cùng đi qua một điểm. Số giao điểm được
xác định như sau: Chọn một đường thng, đường thẳng này cắt n - 1
đường thẳng còn lại tạo ra n - 1 giao điểm, làm như vậy với n đường
thẳng ta được n.(n - 1) giao điểm. Nhưng mỗi giao điểm đã được nh 2
lần, nên số giao điểm là
n.(n - 1):2 giao điểm
1,5
- Khi số giao điểm là 1128 ta có:
n(n - 1):2= 1128
1,0
- Lý luậnm được n = 48
0,5
b,
(2,0 điểm)
- Giả sử số giao điểm bằng 2017
- Áp dụng kết quả câu a ta có n(n - 1):2 = 2017
1,0
- Lý luậnm ra điều vô
- Kết luận: Số giao điểm không thể bằng 2017
1,0
C ý:
- Học sinh có cách giải khác đúng cho điểm tương đương.
- Nếu bài hình phần trên sai, thì vẫn chấm điểm phần dưới
Trang 75
- Bài 2. Câu a chỉ ra được chữ số tận cùng là cho điểm tối đa
- Bài 5. Lí luận không chính xác thì tùy từng ý trừ điểm
| 1/75

Preview text:


PHÒNG GIÁO DỤC VÀ ĐÀO TẠO
ĐỀ KHẢO SÁT HỌC SINH GIỎI LỚP 6 CẤP HUYỆN Môn: Toán – Lớp 6
Thời gian làm bài: 120 phút (Không kể thời gian giao đề) ĐỀ 1
Bài 1: (1,0điểm) Thực hiện phép tính( tính hợp lý nếu có thể )
a/ 1968 : 16 + 5136 : 16 -704 : 16
b/ 23. 53 - 3 {400 -[ 673 - 23. (78 : 76 +70)]}
Bài 2: ( 1,0điểm) M có là một số chính phương không nếu :
M = 1 + 3 + 5 +…+ (2n-1) ( Với n  N , n  0 )
Bài 3: (1,5điểm) Chứng tỏ rằng: a/ (3100+19990) 2
b / Tổng của 4 số tự nhiên liên tiếp không chia hết cho 4
Bài 4 : (1,0điểm) So sánh A và B biết : 1718 + 1 1717 + 1 A = , B = 1719 + 1 1718 + 1
Bài 5: ( 2,0điểm ) Tím tất cả các số nguyên n để: n + a) Phân số
1 có giá trị là một số nguyên n − 2 12n + 1 b) Phân số là phân số tối giản 30n + 2
Bài 6: (2,5điểm)
Cho góc xBy = 550 .Trên các tia Bx, By lần lượt lấy các điểm A, C ( A  B, C  B ). Trên
đoạn thẳng AC lấy điểm D sao cho góc ABD = 300
a/ Tính độ dài AC, biết AD = 4cm, CD = 3cm b/ Tính số đo góc DBC
c/ Từ B vẽ tia Bz sao cho góc DBz = 900 . Tính số đo ABz.
Bài 7: (1,0điểm) Tìm các cặp số tự nhiên x , y sao cho : (2x + 1)( y – 5) = 12
---------- HẾT ----------
(Đề thi gồm có 01 trang).
Thí sinh không được sử dụng tài liệu. Cán bộ coi thi không giải thích gì thêm.
Họ và tên thí sinh:.......................................; Số báodanh......................... Trang 1
PHÒNG GIÁO DỤC VÀ ĐÀO TẠO
ĐỀ KHẢO SÁT HỌC SINH GIỎI LỚP 6 CẤP HUYỆN Môn: Toán – Lớp 6
Thời gian làm bài: 120 phút (Không kể thời gian giao đề)
HƯỚNG DẪN CHẤM ĐỀ 1
Bài 1: (1,0 điểm) Ý/Phần Đáp án Điểm a = 16(123+ 321 - 44):16 0 , 2 5 = 400 0 , 2 5 b =8.125-3.{400-[673-8.50]} 0,25 = 1000-3.{400-273} =619 0,25 Bài 2: (1,0 điểm) Ý/Phần Đáp án Điểm
M = 1 + 3 + 5 +…+ (2n-1) ( Với n  N , n  0 ) 0 , 5
Tính số số hạng = ( 2n-1-1): 2 + 1 = n
Tính tổng = ( 2n-1+1 ) n : 2 = 2n2 : 2 = n 2 0,5đ
KL: M là số chính phương Bài 3: (1,5 điểm) Ý/Phần Đáp án Điểm Ta có:
3100 = 3.3.3….3 (có 100 thừa số 3)
= (34)25 = 8125 có chữ số tận cùng bằng 1
19990 = 19.19…19 ( có 990 thứa số 19 ) 0 , 2 5 a
= (192)495 = 361495 ( có chữ số tận cùng bằng 1
Vậy 3100+19990 có chữ số tận cùng bằng 2 nên tổng này chia 0,25 hết cho 2 0,5
Gọi 4 số tự nhiên liên tiếp là : a ; (a +1) ;( a + 2) ;( a + 3 ) ; ( a  N ) 0,25 b
Ta có : a + (a+1) + (a+2) + (a+3) = 4a + 6 0,25
Vì 4a 4 ; 6 không chia hết 4 nên 4a+ 6 không chia hết 4
Bài 4 : ( 1,0 điểm) Ý/Phần Đáp án Điểm 1718 + 1 1718 + 1 1718 + 1 + 16 0,75 Vì A = < 1  A= < = 1719 + 1 1719 + 1 1719 + 1 + 16 0,25 Trang 2 17(1717 + 1 ) 1717 + 1 = = B 17(1718 + 1 ) 1718 + 1 Vậy A < B Bài 5: (2,0 điểm) Ý/Phần Đáp án Điểm a n +1
là số nguyên khi ( n+1)  (n-2) 0.5 n − 2
Ta có (n+1) = (n − 2) +  3
Vậy (n+1)  (n-2) khi 3 (n-2) (n-2)  Ư(3) =  3 − ; 1 − ;1;  3 => n   1 − ;1;3;  5 0,5
Gọi d là ƯC của 12n+1 và 30n+2 ( dN* )  0,25 n 12 +  1 d n 30 , + 2d n 12 ( 5 + ) 1 − n 30 ( 2 + )
2 d  (60n+5-60n-4)  d  1 d mà d 0,5đ b N*  d = 1
Vậy phân số đã cho tối giản 0,25 Bài 6: (2,5 điểm) Ý/Phần Đáp án Điểm Vẽ hình đúng TH1 TH2 a x x A z A D D 0,25 B B C C y y z
Vì D thuộc đoạn thẳng AC nên D nằm giữa A và C : AC= AD + CD = 4+3 = 7 cm 0,25
Chứng minh được tia BD nằm giữa hai tia BA và BC 0,25 b 0,25
Ta có đẳng thức :  ABC =  ABD +  DBC   DBC =  ABC -  ABD 0,5 =550 – 300 = 250 c Xét hai trường hợp:
- Trường hợp 1: Tia Bz và tia BD nằm về hai phía nửa mặt phẳng 0,25
có bờ là AB nên tia BA nằm giữa hai tiaBz và BD Trang 3
Tính được  ABz = 900 -  ABD = 900- 300 = 600 0,25
- Trường hợp 2 :Tia Bz và tia BD nằm về cùng nửa mặt phẳng có 0,25
bờ là AB nên tia BD nằm giữa hai tia Bz và BA
Tính được  ABz = 900 +  ABD = 900 + 300 = 1200 0,25 Bài 7: (1,0 điểm) Ý/Phần Đáp án Điểm
(2x+ 1); (y - 5) là các ước của 12 0,25 Ư(12) =  12 ; 6 ; 4 ; 3 ; 2 ; 1  0,2 5 Vì 2x + 1 là lẻ nên : 2x + 1= 1  x=0 , y =17 0,25 2x + 1= 3  x=1 , y=9 0,25
Vậy với x = 0 thì y = 17 ; Với x = 1 thì y = 9
PHÒNG GIÁO DỤC VÀ ĐÀO TẠO
ĐỀ KHẢO SÁT HỌC SINH GIỎI LỚP 6 CẤP HUYỆN Môn: Toán – Lớp 6
Thời gian làm bài: 120 phút (Không kể thời gian giao đề) ĐỀ 2 48 46 6 4 2
Bài 1: (5,0 điểm) . Cho 50 44
A = 5 – 5 + 5 − 5 +  +5 - 5 5 + −1. a) Tính A.
b) Tìm số tự nhiên n biết n 26.A +1 = 5
c) Tìm số dư trong phép chia A cho 100.
Bài 2: (3,0 điểm). Tìm số tự nhiên x ,biết: )
a 1+ 3 + 5 + 7 + 9 ++ (2x – ) 1 = 225 x x 1 + x+2 x+3 x+2015 2019 b) 2 + 2 + 2 + 2 +  +2 = 2 − . 8 Bài 3: (5,0 điểm)
a) Cho số abc chia hết cho 37. Chứng minh rằng số cab cũng chia hết cho 37.
b) Tìm số x, y nguyên biết x.y +12 = x + y
Bài 4 (3,0 điểm): Tìm số tự nhiên a nhỏ nhất sao cho: a chia cho 2 dư 1, a chia cho 3 dư 1, a chia
cho 5 dư 4, a chia cho 7 dư 3. Bài 5: (4,0 điểm)
1. Cho 30 điểm phân biệt trong đó có a điểm thẳng hàng, cứ qua 2 điểm ta vẽ được 1 đường
thẳng. Tìm a, biết số đường thẳng tạo thành là 421 đường thẳng.
2. Vẽ đoạn thẳng AB = 6cm . Lấy hai điểm C và D nằm giữa A và B sao cho AC + BD = 9cm.
a) Chứng tỏ D nằm giữa A và C.
b) Tính độ dài đoạn thẳng CD ? --- Hết --- Trang 4
PHÒNG GIÁO DỤC VÀ ĐÀO TẠO
ĐỀ KHẢO SÁT HỌC SINH GIỎI LỚP 6 CẤP HUYỆN Môn: Toán – Lớp 6
Thời gian làm bài: 120 phút (Không kể thời gian giao đề)
HƯỚNG DẪN CHẤM ĐỀ 2 Bài 1: (4,0 điểm) Đáp án Điểm 50 48 46 44 6 4 2 a. A = 5 – 5 + 5 − 5 +  +5 - 5 + 5 −1. 2 50 48 46 44 6 4 2
 25A = 5 .(5 – 5 + 5 − 5 +  +5 - 5 + 5 −1.) 0,25 52 50 48 46 8 6 4 2 0,25 = 5
– 5 + 5 − 5 +  +5 - 5 + 5 −5 . Suy ra 52 25A + A = 5 −1 0,50 Vậy = ( 52 A 5 − ) 1 : 26 0,25
b) Tìm số tự nhiên n biết n 26.A +1 = 5 Ta có n 26.A +1 = 5 mà 52 26A = 5 −1 nên 52 n 5 −1+1 = 5 0,25 52 n Suy ra 5 = 5  n = 52 .Vậy n = 52 0,25
c). Tìm số dư trong phép chia A cho 100. 50 48 46 44 6 4 2
A = 5 – 5 + 5 − 5 +  +5 - 5 + 5 − . 1 ( có 26 số hạng) 0,25 =( 50 48 5 – 5 ) + ( 46 44 5 − 5 ) +  +( 6 4 5 - 5 ) 2 0,25 5 + −1. =( 50 48 ) + ( 46 44 − 5 ) +  ( 6 4 )+ ( 2 5 – 5 5 + 5 - 5 5 − ) 0,25 1 . 48 = .( 2 ) 44 + 5 .( 2 ) 4 +  .( 2 )+ ( 2 5 5 –1 5 –1 +5 5 –1 5 − ) 0,25 1 . 48 44 4
= 5 .24 + 5 .24 +  +5 .24+ 24. 0,25 46 42 2
= 5 .25.24 + 5 .25.24 +  +5 .25.24+ 2 . 4 0,50 46 42 2 = 5 + +  +5 24 = ( 46 42 2 .600 5 .600 .600+ . 6.100. 5 + 5 + ...+ 5 ) + 24 0,25
Suy ra A chia cho 100 dư 24. 0,25
Bài 2: (3,0 điểm). Tìm số tự nhiên x ,biết: Đáp án Điểm
a) 1+ 3+ 5 + 7 + 9 ++ (2x – ) 1 = 225
Với mọi x  N ta có 2x – 1 là số lẻ 0,25
Đặt A = 1 + 3 + 5 + 7 + 9 +…+ ( 2x – ) 1
 A là tổng của các số lẻ liên tiếp từ 1 đến 2x – 1 0,25
Số số hạng của A là: (2x –1– ) 1 : 2 +1 = x (Số hạng) 0,25  = ( ) 2 A 2x –1 +1.x : 2 = x  0,25 Mà 2 2 A = 225  x = 225 = 15 0,25  x = 15 Vậy x = 15 0,25 Trang 5 x x 1 + x+2 x+3 x+2015 2019 b) 2 + 2 + 2 + 2 +  +2 = 2 − . 8 x x x 2 x 3 x 2015 2019 3
2 .1+ 2 .2 + 2 .2 + 2 .2 +  +2 .2 = 2 − 2 . 0,25 x ( 2 3 2015 + + + +  ) 3 = .( 2016 2 . 1 2 2 2 +2 2 2 − ) 1 . 0,25 Đặt 2 3 2015 M = 1+ 2 + 2 + 2 +  +2 Ta được 2 3 4 2016 2.M = 2 + 2 + 2 + 2  +2 0,25 2016 Suy ra M = 2 −1 0,25 Vậy ta có x ( 2016 − ) 3 = ( 2016 2 . 2 1 2 . 2 − ) 1 . 0,25 x 3
 2 = 2  x = 3 .Vậy x = 3 0,25 Bài 3: (5,0 điểm). Đáp án Điểm
a) Cho số abc chia hết cho 37. Chứng minh rằng số cab cũng chia hết cho 37.
Ta có abc 37  100.abc 37  abc00 37 0,50  (ab .1000+c00) 37 0,25  ab .999 +  (c00+ab) 37  0,25  (ab .999+ cab) 37 0,25 Mà ab .999 = ab .37.27 37 0,25  cab 37 0,25
Vậy nếu abc 37 thì cab 37 0,25
b) Tìm số x, y nguyên biết x.y +12 = x + y
Ta có x.y +12 = x + y  x.y − x − y +12 = 0 0,25  x.( y − ) 1 − y +12 = 0 0,25  x.( y − ) 1 − (y − ) 1 +11 = 0 0,25  (x − ) 1 .(y − ) 1 = 1 − 1 ( ) 1 0,25
Vì x, y  Z nên x −1 Z; y −1 Z 0,25 Do đó từ ( )
1  x −1; y −1là các ước của -11 0,25
Các ước của -11 là -11; -1;1;11 0,25 +) Với x −1 = 1
− 1 thì y −1 =1. Suy ra x = 1
− 0; y = 2 ( Thỏa mãn) 0,25 +) Với x −1 = 1
− thì y −1 =11. Suy ra x = 0; y = 12 ( Thỏa mãn) 0,25
+) Với x −1 = 1thì y −1 = 1
− 1. Suy ra x = 2; y = -10 ( Thỏa mãn) 0,25
+) Với x −1 = 11 thì y −1 = 1
− .Suy ra x =12; y = 0 ( Thỏa mãn) 0,25 Vậy (x; y)  (  −10;2);(0;12);(2; 10 − );(12;0). 0,25 Bài 4: (3,0 điểm). Trang 6 Đáp án Điểm
Vì a chia cho 2 dư 1, a chia cho 3 dư 1, a chia cho 5 dư 4, a chia cho 7 dư 3 0,25
Nên a −1 2 ; a −1 3 ; a − 4 5 ; a − 3 7
 a +1 2 ; a + 2 3 ; a +1 5 ; a + 4 7 0,25
 a +11 2 ; a +11 3 ; a +11 5 ; a +11 7 0,50 0,25  a +11 BC(2;3;5; 7).
Mà a là số tự nhiên nhỏ nhất 0,25 0,25  a +11 = BCNN (2;3;5; 7).
Mà các số 2; 3; 5; 7 nguyên tố cùng nhau 0,25 0,25
 BCNN (2;3;5; 7) = 2.3.5.7 = 210  a + 11 = 210. 0,25  a = 199. 0,25
Vậy số tự nhiên cần tìm là 199. 0,25 Bài 5: (4,0 điểm) Đáp án Điểm
1. – Giả sử trong 30 điểm phân biệt không có 3 điểm nào thẳng hàng : 0,25
+ Chọn một điểm bất kì trong 30 điểm đã cho. Qua điểm đó và từng điểm trong
29 điểm còn lại ta vẽ được 29 đường thẳng.
+ Làm như vậy với 30 điểm thì ta vẽ được tất cả là 29.30 đường thẳng. 0,25
+ Nhưng mỗi đường thẳng đã được tính hai lần nên số đường thẳng thực tế vẽ
được là (29.30) : 2 = 435 đường thẳng. 0,25
Vậy qua 30 điểm phân biệt mà không có 3 điểm nào thẳng hàng ta vẽ được 435 đường thẳng.
– Tương tự như trên, giả sử trong a điểm phân biệt không có 3 điểm nào thẳng 0,25
hàng ta vẽ được a.(a − ) 1 : 2 đường thẳng.
Nhưng qua a điểm thẳng hàng ta chỉ vẽ được một đường thẳng nên số đường thẳng 0,25 bị giảm đi là a.(a − ) 1 : 2 −1 đường thẳng.
Theo bài ra ta có : a.(a − ) 1 : 2 −1 = 435 − 421 = 14 0,25  a.(a − ) 1 = 30 = 6.5 0,25
Vì a-1 và a là hai số tự nhiên liên tiếp và a −1  a nên a = 6. 0,25 A D C B 2. Hình a) Ch
vẽ n
: g tỏ D nằm giữa A và C.
Vì D nằm giữa A và B nên: AD + DB = AB 0,25
Thay AB = 6 cm ta có AD + DB = 6 cm . 0,25
Lại có AC + DB = 9 cm  AD + DB  AC + DB hay AD  AC. 0,25
Trên tia AB có : AD  AC suy ra D nằm giữa A và C 0,25
b) Tính độ dài đoạn thẳng CD ?
Vì D nằm giữa A và C suy ra AD + DC = AC. 0,25
Lại có AC + DB = 9 cm , suy ra AD + DC + DB = 9cm 0,25 Hay (AD + DB) + DC = 9cm 0,25 Trang 7
Thay AD + DB = 6 cm , ta có 6cm + DC = 9 (cm) . Vậy DC = 3(cm) 0,25
Chú ý: Học sinh giải theo cách khác mà đúng thì vẫn cho điểm tương ứng với từng câu, từng bài theo hướng dẫn trên./.
PHÒNG GIÁO DỤC VÀ ĐÀO TẠO
ĐỀ KHẢO SÁT HỌC SINH GIỎI LỚP 6 CẤP HUYỆN Môn: Toán – Lớp 6
Thời gian làm bài: 120 phút (Không kể thời gian giao đề) ĐỀ 3 Bài 1 (5 điểm)
1): Rút gọn các biểu thức sau:
M = 3 – 32 + 33 – 34 + … + 32015 – 32016. 2) Chứng tỏ rằng: 1 1 1 1 1 3 a) + + + ... + +  2 2 2 2 2 2 3 4 99 100 4
Bài 2 (3 điểm): Tìm số tự nhiên x biết:
a) 1 + 3 + 5 + 7 + 9 + … + (2x – 1 ) = 225
b) 2x . 2x + 1. 2x + 2 = 1000 … 0 : 518 18 chữ số 0 Bài 3: (5 điểm)
a) Cho 3a + 2b 17 (a , b  N). Chứng minh 10a + b 17
b) Tìm số x,y nguyên biết xy + x – y = 4 Bài 4: (4 điểm)
Cho 30 điểm phân biệt trong đó có a điểm thẳng hàng cứ qua 2 điểm ta vẽ được 1 đường thẳng.
Tìm a, biết số đường thẳng tạo thành là 421 đường thẳng . Bài 5 (3 điểm)
Tìm số tự nhiên a nhỏ nhất sao cho: a chia cho 2 dư 1, a chia cho 3 dư 1, a chia cho 5 dư 4, a chia cho 7 dư 3. = Hết =
PHÒNG GIÁO DỤC VÀ ĐÀO TẠO
ĐỀ KHẢO SÁT HỌC SINH GIỎI LỚP 6 CẤP HUYỆN Môn: Toán – Lớp 6
Thời gian làm bài: 120 phút (Không kể thời gian giao đề)
HƯỚNG DẪN CHẤM ĐỀ 3 Bài Nội dung Điểm Bài 1 (5 điểm) 1.1.a)
M = 3 – 32 + 33 – 34 + + 32015 – 32016
Ta có :3M = 32 – 33 + 34 – 35 + + 32016 – 32017 0,5 Trang 8
(2,0 đ)  3M + M = 3 + (32 – 32) + (33 – 33)+ + (32016 – 32016) – 32017 0,5
4M = 3 + 0 + 0 + . . . + 0 – 32017 0,5 4M = 3 – 32017  M = (3 – 32017) : 4 0,5 1.2. Ta có: 1 1  2 3 2.3 (3,0 đ) 1 1  2 4 3.4 1 1  2 5 4.5 . . . . . . . 1 1  2 99 98.99 1 1  2 100 99.100 0,5  1 1 1 1 1 1 1 1 1 1 1 + + + ...+ +  + + + + ...+ + 2 2 2 2 2 2 3 4 99 100 4 2.3 3.4 4.5 98.99 99.100 0,5 1 1 1 1 1 1 Mà + + + + ... + + 4 2.3 3.4 4.5 98.99 99.100 1 1 1 1 1 1 1 1 1 1 1 = + − + − + − + ... + − + − 4 2 3 3 4 4 5 98 99 99 100 0,5 1 1 1 = + − 4 2 100 0,5 3 1 3 = −  4 100 4 0,5  1 1 1 1 1 3 + + + ... + +  2 2 2 2 2 2 3 4 99 100 4 0,5 Bài 2 (3 điểm) 2.a)
Với mọi x  N ta có 2x – 1 là số lẻ 0,25
Đặt A = 1 + 3 + 5 + 7 + 9 + + (2x – 1) (1,5)
 A là tổng của các số lẻ liên tiếp từ 1 đến 2x – 1 0,25
Số số hạng của A là: (2x – 1 – 1) : 2 + 1 = x (Số hạng) 0,25
 A = [(2x – 1) + 1] . x : 2 = x2 0,25
Mà A = 225  x2 = 225 = 152 0,25  x = 15 Vậy x = 15 0,25 2.b)
2x . 2x + 1. 2x + 2 = 1000 0 : 518 18 chữ số 0
(1,5 đ) 2x + x + 1+ x + 2 = 10 . 10 . 10 . . 10 : 5 . 5 . 5 . . 5
18 thừa số 10 18 thừa số 5 0,25 Trang 9
23x + 3 = (10 : 5 ).(10 : 5). (10 : 5) . .(10 : 5) 18 thừa số (10 : 5) 0,25
23x + 3 = 2 . 2 . 2 . . 2 18 thừa số 2 0,25 23x + 3 = 218 0,25  3x + 3 = 18 3x = 18 – 3 3x = 15 x = 15 : 3 x = 5 0,25 Vậy x = 5 0,25 Bài 3: (5 điểm) 3.a)
Vì 3a + 2b 17  10(3a + 2b) 17 0,5  (30a + 20b) 17 0,25
(2,0đ)  (30a + 3b + 17b) 17 0,25  [3(10a + b) + 17b] 17 0,25 Vì 17b 17 0,25  3(10a + b) 17 0,25
 10a + b 17 (vì 3 và 17 nguyên tố cùng nhau) 0,25 3.b) xy + x – y = 4 x(y + 1) – y = 4 0,25
(3,0đ) x(y + 1) – y – 1 + 1 = 4 0,5 x(y + 1) - ( y + 1 ) + 1= 4 0,25 ( y + 1 )( x – 1) + 1 = 4 0,25 ( y + 1)( x – 1)= 3 0,25
Vì x, y là số nguyên nên y + 1,x – 1 là ước của 3. 0,25
Nếu x -1 = 1 và y + 1 = 3 thì x = 2 và y = 2 0,25
Nếu x -1 = -1 và y + 1 = -3 thì x = 0 và y = -4 0,25
Nếu x -1 = 3 và y + 1 = 1 thì x = 4 và y = 0 0,25
Nếu x -1 = -3 và y + 1 = -1 thì x = -2 và y = -2 0,25
Vậy x = 2 và y = 2 hoặc x = 0 và y = -4 hoặc x = 4 và y = 0 hoặc x = -2 và y = -2 0,25 Bài 4( 4điểm) 4.
Giả sử trong 20 điểm không có 3 điểm nào thẳng hàng.
Gọi 20 điểm đó là A1, A2, A3, ... ,A20. (4 đ)
Vì cứ qua 2 điểm ta vẽ được 1 đường thẳng nên 0,25
Qua điểm A1 và từng điểm trong 19 điểm còn lại A2, A3,...,A20 ta
vẽ được 19 đường thẳng. 0,5
Qua điểm A2 và từng điểm trong 18 điểm còn lại A3, A4,...,A20 ta
vẽ được 18 đường thẳng. 0,5 … ….
Qua điểm A19 và điểm A20 ta vẽ được 1 đường thẳng. 0,5 Trang 10
Do đó số đường thẳng tạo thành là: 1 + 2 + 3 +... + 19 + 20 =
( 1+ 20).20 : 2 = 190 ( đường thẳng) 0,5
Với a điểm trong đó không có 3 điểm nào thẳng hàng thì ta có số
đường thẳng tạo thành là 1 + 2 + 3 +... +( a – 1) = (a- 1). a: 2 0,5
Với a điểm thẳng hàng thì ta chỉ vẽ được 1 đường thẳng 0,25
Vậy trong 20 điểm mà có a diểm thẳng hàng thì sổ đưởng thẳng giảm đi là
( a- 1).a: 2 - 1 = 190 – 170 0,5 ( a- 1).a: 2 - 1 = 20 ( a- 1).a: 2 = 21 0,25 ( a- 1).a = 42 ( a- 1).a = 6.7
Mà a-1 và a là 2 số tự nhiên liên tiếp a -1 < a nên a -1 = 6 và a =7 0,25 Vậy a = 7 5. Gọi số phải tìm là a  a = 2k + 1 (3,0 đ) a = 3q + 1 a = 5m + 4 a = 7r + 3 (k, q, m, r  N) 0,5  a + 11 = 2k + 12 2 a + 11 = 3q + 12 3 a + 11 = 5m + 15 5 a + 11 = 7r + 14 7  a + 11  BC(2; 3; 5; 7) 1,0
Mà a là số tự nhiên nhỏ nhất  a + 11 = BCNN(2; 3; 5; 7) 0,5
Mà 2; 3; 5; 7 nguyên tố cùng nhau
 BCNN(2; 3; 5; 7) = 2.3.5.7 = 210 0,25  a + 11 = 210 a = 210 – 11 a = 199 0,5 Vậy a = 199 0,25 Trang 11
PHÒNG GIÁO DỤC VÀ ĐÀO TẠO
ĐỀ KHẢO SÁT HỌC SINH GIỎI LỚP 6 CẤP HUYỆN Môn: Toán – Lớp 6
Thời gian làm bài: 120 phút (Không kể thời gian giao đề) ĐỀ 4
Bài 1 : (5 điểm) Thực hiện các phép tính sau một cách hợp lý : a) ( 2 2 2 + + ) ( 2 2 10 11 12 : 13 +14 ). b) 2
1.2.3...9 −1.2.3...8 −1.2.3...7.8 (3.4.2 )2 16 c) 13 11 9 11.2 .4 − 16
d) 1152 - (374 + 1152) + (-65 + 374)
e) 13 - 12 + 11 + 10 - 9 + 8 - 7 - 6 + 5 - 4 + 3 + 2 - 1
Bài 2 : (4 điểm) Tìm x, biết: a) ( + ) =( − )2 2 2 19x 2.5 :14 13 8 − 4 b) x + (x + )
1 + (x + 2) + ... + (x + 30) =1240 c) 11 - (-53 + x) = 97 d) -(x + 84) + 213 = -16
Bài 3 :(2 điểm) Tìm hai số tự nhiên a và b, biết: BCNN(a,b)=300; ƯCLN(a,b)=15 và a+15=b. Bài 4 :(3 điểm)
a)Tìm số nguyên x và y, biết : xy - x + 2y = 3. 102 101 +1
b) So sánh M và N biết rằng : M = 103 101 + . 1 103 101 +1 N = 104 101 + . 1
Bài 5 : (6 điểm) Cho đoạn thẳngAB, điểm O thuộc tia đối của tia AB. Gọi M, N thứ tự là trung điểm của OA, OB.
a) Chứng tỏ rằng OA < OB.
b) Trong ba điểm O, M, N điểm nào nằm giữa hai điểm còn lại ?
c) Chứng tỏ rằng độ dài đoạn thẳng MN không phụ thuộc vào vị trí của điểm O (O thuộc tia đối của tia AB).
PHÒNG GIÁO DỤC VÀ ĐÀO TẠO
ĐỀ KHẢO SÁT HỌC SINH GIỎI LỚP 6 CẤP HUYỆN Môn: Toán – Lớp 6
Thời gian làm bài: 120 phút (Không kể thời gian giao đề)
HƯỚNG DẪN CHẤM ĐỀ 7
Bài 1 : (5 điểm) Thực hiện các phép tính sau một cách hợp lý : Đáp án Điểm ( 2 2 2 + + ) ( 2 2 a) 10 11 12
: 13 + 14 ) = (100 +121+144) : (169 +196) 1 = 365 : 365 = 1 Trang 12 2
b) 1.2.3...9 −1.2.3...8 −1.2.3...7.8 = 1.2.3...7.8.(9 −1− 8) = 1.2.3...7.8..0 = 0 1 (3.4.2 )2 (3.2 .2 )2 3 .(2 )2 16 2 16 2 18 c) = = 13 11 9 11.2 .4 −16 ( )11 −( )9 13 22 36 13 2 4 11.2 .2 − 2 11.2 . 2 2 1 2 36 2 36 2 36 2 3 .2 3 .2 3 .2 3 .2 = = = = = 2 13 22 36 35 36 35 11.2 .2 − 2 11.2 − 2 2 (11− 2) 9
d) 1152 - (374 + 1152) + (-65 + 374) = 1152 - 374 - 1152 + (-65) + 374 1
= (1152 - 1152) + (-65) + (374 - 374) = -65
e) 13 - 12 + 11 + 10 - 9 + 8 - 7 - 6 + 5 - 4 + 3 + 2 - 1 = 1
= 13 - (12 - 11 - 10 + 9) + (8 - 7 - 6 + 5) - (4 - 3 - 2 + 1) = 13
Bài 2 : (4 điểm) Tìm x : Câu Đáp án Điểm a. ( + ) =( − )2 2 2 19x 2.5 :14 13 8 − 4  =  ( − )2 2 2 x 14. 13 8 − 4  − 2.5   :19 1  x = 4 b. x + (x + )
1 + (x + 2) + ... + (x + 30) =1240  
  x + x + ...+ x  + (1+ 2 + ...+ 30) =1240    31 So hang  30.(1+ 30)  31x + =1240 1 2  31x =1240 − 31.15 775  x = = 25 31 c. 11 - (-53 + x) = 97  x =11− 97 − ( 5 − 3) = 3 − 3 1 d. -(x + 84) + 213 = -16  −(x + 84) = 1 − 6 − 213  −(x + 84) = 2 − 29 1  x + 84 = 229  x = 229 − 84 =145
Bài 3 :
(3 điểm) Đáp án Điểm
Từ dữ liệu đề bài cho, ta có :
+ Vì ƯCLN(a, b) = 15, nên ắt tồn tại các số tự nhiên m và n khác 0, sao cho: a = 15m; b = 15n (1) 3 và ƯCLN(m, n) = 1 (2)
+ Vì BCNN(a, b) = 300, nên theo trên, ta suy ra : Trang 13
 BCNN(15m; 15n) = 300 =15.20  BCNN(m; n) = 20 (3)
+ Vì a + 15 = b, nên theo trên, ta suy ra : 15m +15 =15n 15.(m + ) 1 = 15n  m +1 = n (4)
Trong các trường hợp thoả mãn các điều kiện (2) và (3), thì chỉ có trường hợp : m
= 4, n = 5 là thoả mãn điều kiện (4).
Vậy với m = 4, n = 5, ta được các số phải tìm là : a = 15 . 4 = 60; b = 15 . 5 = 75 Bài 4 :(2 điểm) Câu Đáp án Điểm Chứng minh đẳng thức:
- (-a + b + c) + (b + c - 1) = (b - c + 6) - (7 - a + b) + c.
Biến đổi vế trái của đẳng thức, ta được :
VT = -(-a + b + c) + (b + c - 1)
= -(-a) - (b + c) + (b + c) + (-1) = a - 1 a.
Biến đổi vế phải của đẳng thức, ta được : 1
VP = (b - c + 6) - (7 - a + b) + c
= b + (-c) + 6 - 7 + a - b + c = [b + (-b)] + [(-c) + c] + a + [6 + (-7)] = a - 1
So sánh, ta thấy : VT = VP = a - 1
Vậy đẳng thức đã được chứng minh.
Với a > b và S = -(-a - b - c) + (-c + b + a) - (a + b), ta có :
 S = −(−a − b − c) + (−c + b + a) − (a + b)
 S = −(−a − b)+c + (−c) + (b + a) − (a + b)  S = −(−a − b) = a + b
Tính S : theo trên ta suy ra :  S = a + b
* Xét với a và b cùng dấu, ta có các trường hợp sau xảy ra :
+ a và b cùng dương, hay a > b > 0, thì a + b > 0 :  S = a + b = a + b
+ a và b cùng âm, hay 0 > a > b, thì a + b < 0  −(a + b)  0 , nên suy ra :
 S = a + b = −(a + b) = −a + (−b)
* Xét với a và b khác dấu : b. 1
Vì a > b, nên suy ra : a > 0 và b < 0  −b  0 , ta cần xét các trường hợp sau xảy ra :
+ a  b ,hay a > -b > 0, do đó a + b = a − (−b)  0 , suy ra:  S = a + b = a + b
+ a  b , hay -b > a > 0, do đó a + b = a − (−b)  0 , hay −(a + b)  0 suy ra :  S = a + b = (a − + b) = a − + (−b)
Vậy, với : + S = a + b (nếu b < a < 0) + S = a
− + (−b) (nếu b < a < 0, hoặc b < 0 < a  b )
Bài 5 : (6 điểm) Câu Đáp án Điểm Hình o m a n b vẽ Trang 14
Hai tia AO, AB đối nhau, nên điểm A nằm giữa hai điểm O và B, suy a. ra : 2  OA < OB.
Ta có M và N thứ tự là trung điểm của OA, OB, nên : OA OB  OM = ; ON = b. 2 2 2
Vì OA < OB, nên OM < ON.
Hai điểm M và N thuộc tia OB, mà OM < ON, nên điểm M nằm giữa hai điểm O và N.
Vì điểm M nằm giữa hai điểm O và N, nên ta có :  OM + MN = ON suy ra :  MN = ON − OM OB − OA AB c. hay :  MN = = 2 2 2
Vì AB có độ dài không đổi, nên MN có độ dài không đổi, hay độ dài
đoạn thẳng MN không phụ thuộc vào vị trí của điểm O (O thuộc tia đối của tia AB).
PHÒNG GIÁO DỤC VÀ ĐÀO TẠO
ĐỀ KHẢO SÁT HỌC SINH GIỎI LỚP 6 CẤP HUYỆN Môn: Toán – Lớp 6
Thời gian làm bài: 120 phút (Không kể thời gian giao đề) ĐỀ 5
Câu 1(3,0 điểm): Tính giá trị của các biểu thức sau: 3 − 28.43 28.5 28.21 a. 4 2
2 .5 −[131− (13 − 4) ] b. + + − 5 5.56 5.24 5.63
Câu 2(4,0 điểm): Tìm các số nguyên x biết. 3  5 −  2 − 4 5 − a.  x  .   b. 3 2 (7x −11) = ( 3
− ) .15 + 208 c. 2x −7 = 20+5.( 3 − )  3  35 6 Câu 3(5,0 điểm):
a, Một số tự nhiên chia cho 7 dư 5,chia cho 13 dư 4. Nếu đem số đó chia cho 91 thì dư bao nhiêu?
b, Học sinh khối 6 khi xếp hàng; nếu xếp hàng 10, hàng 12, hàng15 đều dư 3 học
sinh. Nhưng khi xếp hàng 11 thì vùa đủ. Biết số học sinh khối 6 chưa đến 400 học
sinh.Tính số học sinh khối 6? Câu 4 (6,0 điểm):
Cho góc bẹt xOy. Trên cùng một nửa mặt phẳng có bờ xy,vẽ các tia Oz và Ot sao cho 0 0
xOz = 70 ; yOt = 55 .
a. Chứng tỏ tia Oz nằm giữa hai tia Ox và Ot ?
b. Chứng tỏ tia Ot là tia phân giác của góc yOz?
c.Vẽ tia phân giác On của góc xOz. Tính góc nOt? Câu 5 (2,0 điểm):
Cho n là số nguyên tố lớn hơn 3. Hỏi n2 + 2006 là số nguyên tố hay là hợp số. ---------Hết--------- Trang 15
PHÒNG GIÁO DỤC VÀ ĐÀO TẠO
ĐỀ KHẢO SÁT HỌC SINH GIỎI LỚP 6 CẤP HUYỆN Môn: Toán – Lớp 6
Thời gian làm bài: 120 phút (Không kể thời gian giao đề)
HƯỚNG DẪN CHẤM ĐỀ 5 Câu Nội dung Thang điểm Câu 2 =16.5 − (131− 9 ) 0.5 1(4điểm) = 80 − 50 0.5 a (1,5) = 30 0.5 b (1,5) −3 28 43 5 1 + 0.5 .( + − ) 5 5 56 24 3 −3 28 129 35 56 = + 0.5 .( + − ) 5 5 168 168 168 −3 28 108 = + 0,25 . 5 5 168 −3 18 = + 5 5 0.25 = 3 câu 2 0.5 (4điểm) a (1,0) 0.5 b (1,5) 3 2 (7x −11) = ( 3 − ) .15 + 208 0.5 3 (7x −11) = 9.15 + 208 3 3 (7x −11) = 7 0.5 18
 7x −11 = 7  x = 7 0.5 (không thỏa mãn) c (1,5) 2x − 7 = 20 + 5.( 3 − ) 0.5 2x − 7 = 5 2x 7 − 5 = 2x 12 = x 6 =  [  [  [ 0.5 2x 7 − = 5 − 2x=2 x 1 = 0.5 Vậy x1;  6
Câu3(4,0) Gọi số đó là a 0.25
Vì a chia cho 7 dư 5, chia cho 13 dư 4 a (2,0)
a + 9 7;a + 9 13 mà (7,13)=1 nên 1.0 a + 9 7.13
 a+9=91k  a=91k-9 =91k-91+82=91(k-1)+82 (kN) Vậy a chia cho 91 dư 82. 1.0 0.25 b (2,0)
Gọi số Hs khối 6 là a (30.25 Trang 16
Vì khi xếp hàng 10,hàng 12, hàng 15 đều dư 3
a −3 10;12;15  a −3BC(10,12,15) ta có 0.5
BCNN(10,12,15)=60  a − 360;120;180;240;300;360;420;...  .  0.5
a 63;123;183;243;303;363;423;.. 
. mà a 11; a  400  a=363 0.75
Vậy số HS khối 6 là 363 học sinh. 0.5 Câu 4 (6,0) z t n Vẽ hình 0.5 x O y a (1,5)
Vì góc xOy là góc bẹt nên suy ra trên cùng một
nưả mặt phẳng có bờ xy có xOt tOy là hai góc kề bù.  xOt + tOy = 0 180  0 0 0
xOt = 180 − 55  xOt = 125
Trên cùng một nửa mặt phẳng có bờ chứa tia Ox có: 0.75 0 0
xOz xOt(70  125 )  Tia Oz nằm giữa hai tia Ox và Ot. 0.75 b (2,0)
Trên cùng một nửa mặt phẳng có bờ xy ,ta có xOz
zOy là hai góc kề bù 0
xOz + zOy =180 hay 0.75 0 0 0 0 0
70 + zOy = 180  zOy = 180 − 70 = 110
Trên cùng một nửa mặt phẳng có bờ chứa tia Oy có: 0 0
yOt yOz(55  110 )  Tia Ot nằm giữa hai tia Oy và Oz (1)
nên ta có: yOt + tOz = yOz hay 0.75 0 0 0 0 0
55 + tOz = 110  tOz = 110 − 55 = 55 0
yOt = tOz(= 55 ) (2).Từ (1) và (2) suy ra Ot là tia phân giác 0.5 của góc yOz. Trang 17 c (2,0)
xOy là góc bẹt nên suy ra tia Ox và tia Oy là hai tia đối
nhau  Hai tia Ox và Oy nằm trên hai nửa mặt phẳng đối nhau có bờ chứa tia Oz (1) 0.5 0
Vì On là tia phân giác của góc xOz nên xOz 70 0 nOz = = = 35 và 2 2
hai tia On và Ox cùng nằm trên mặt phẳng có bờ chứa tia Oz (2)
Ta lại có tia Ot là tia phân giác của góc yOz (theo b,) 0.5
 Hai tia Ot và Oy cùng nằm trên một nửa mặt phẳng có bờ
chứa tia Oz (3) . Từ (1),(2), (3) suy ra tia On và tia Ot nằm
trên hai nửa mặt phẳng đối nhau có bờ chứa tia Oz  tia Oz 0.5
nằm giữa hai tia On và Ot nên ta có: 0.5
nOz + zOt = nOt hay 0 0 0
nOt = 35 + 55 = 90 .Vậy 0 nOt = 90 Câu 5
n là số nguyên tố, n > 3 nên n không chia hết cho 3. 0.5 (2,0)
Vậy n2 chia hết cho 3 dư 1 0.5
do đó n2 + 2006 = 3m + 1 + 2006 = 3m+2007 0.75
= 3( m+669) chia hết cho 3.
Vậy n2 + 2006 là hợp số. 0.25 Trang 18
PHÒNG GIÁO DỤC VÀ ĐÀO TẠO
ĐỀ KHẢO SÁT HỌC SINH GIỎI LỚP 6 CẤP HUYỆN Môn: Toán – Lớp 6
Thời gian làm bài: 120 phút (Không kể thời gian giao đề) ĐỀ 6
Câu 1. (4 điểm)  12 12 12 5 5 5  12 − − − 5 + + +  
a) Thực hiện phép tính: A = 81. 158158158 7 289 85 13 169 91  : . 4 4 4 6 6 6 711711711  4 − − − 6 + + +   7 289 85 13 169 91  b) Tìm x biết: 1) 2 1 1 1 x 1 x+ 1 1 - (x − ) = (2x −1) 2) 1 7 8 .2 + .2 = .2 + .2 3 4 3 5 3 5 3
c. T×m hai sè tù nhiªn a vµ b biÕt tæng BCNN vµ ¦CLN cña chóng lµ 15
d. Tìm x nguyên thỏa mãn: x +1 + x − 2 + x + 7 = 5x −10
Câu 2. (4 điểm) 2 2 9 2 6 2 14 4
5.(2 .3 ) .(2 ) − 2.(2 .3) .3
a. Thực hiện phép tính: A = 28 18 29 18 5.2 .3 − 7.2 .3
b. Tìm các số nguyên n sao cho: n2 + 5n + 9 là bội của n + 3
c. Chứng minh rằng bình phương của một số nguyên tố khác 2 và 3 khi chia cho 12 đều dư 1
d. Tìm x, y nguyên sao cho: xy + 2x + y + 11 = 0
Câu 3. (4 điểm)
a) Tìm số tự nhiên nhỏ nhất sao cho khi chia cho 11 dư 6, chia cho 4 dư 1và chia cho 19 dư 11.
b) Tìm 3 số có tổng bằng 210, biết rằng 6 9 9 2 số thứ nhất bằng số thứ 2 và số thứ 2 bằng số thứ 3. 7 11 11 3
c. Tìm số tự nhiên a, b, c, d nhỏ nhất sao cho: a 15 b 9 c 9 = ; = ; = b 21 c 12 d 11
d. Tìm hai số biết tỉ số của chúng bằng 5 : 8 và tích của chúng bằng 360. Câu 4. (5 điểm)
1. a) Cho đoạn thẳng AB dài 7cm. Trên tia AB lấy điểm I sao cho AI = 4 cm. Trên tia BA lấy điểm K sao cho BK = 2 cm.
Hãy chứng tỏ rằng I nằm giữa A và K. Tính IK.
b) Trên tia Ox cho 4 điểm A, B, C, D. biết rằng A nằm giữa B và C; B nằm giữa C và D ; OA = 5cm; OD
= 2 cm ; BC = 4 cm và độ dài AC gấp đôi độ dài BD. Tìm độ dài các đoạn BD; AC.
2. Trªn n÷a mÆt ph¼ng cho tr-íc cã bê Ox vÏ hai tia Oy vµ Oz sao cho sè ®o  xOy = 700 vµ sè ®o  yOz = 300.
a) X¸c ®Þnh sè ®o cña  xOz
b) Trªn tia Ox lÊy 2 ®iÓm A vµ B (§iÓm A kh«ng trïng víi ®iÓm O vµ ®é dµi OB lín h¬n ®é dµi OA).
Gäi M lµ trung ®iÓm cña OA. H·y so s¸nh ®é dµi MB víi trung b×nh céng ®é dµi OB vµ AB.
Câu 5. ( 3 điểm) a.
Chứng minh rằng: 32 + 33+ 34 +……+ 3101 chia hết cho 120. a b.
Cho hai số a và b thỏa mãn: a – b = 2(a + b) = b Trang 19 a Chứng minh a = -3b ; Tính ; Tìm a và b b
c. Tìm x, y, z biết: ( x – y2 + z)2 + ( y – 2)2 + ( z +3)2 = 0
Giám thị coi thi không giải thích gì thêm - SBD:....................... Trang 20
PHÒNG GIÁO DỤC VÀ ĐÀO TẠO
ĐỀ KHẢO SÁT HỌC SINH GIỎI LỚP 6 CẤP HUYỆN Môn: Toán – Lớp 6
Thời gian làm bài: 120 phút (Không kể thời gian giao đề)
HƯỚNG DẪN CHẤM ĐỀ 6 Câu Phần Nội dung Điểm a  12 12 12 5 5 5  12 − − − 5 + + + 2đ   158158158 Ta có: . 7 289 85 13 169 91 A = 81. : . Câu 1 4 4 4 6 6 6 711711711  4 − − − 6 + + +  (4 điểm)  7 289 85 13 169 91    1 1 1   1 1 1   12 1− − − 5 1+ + +        7 289 85   13 169 91  158.1001001 = 81.  : .   1 1 1   1 1 1   711.1001001 4 1− − − 6 1+ + +     1  7 289 85 13 169 91       12 5  158 0,5 = 81. : .    4 6  711 18 2 324 0,5 = 81. . = 5 9 5 b
(x + 1) + ( x + 2 ) + . . . . . . . . + (x + 100) = 5750 2đ
=> x + 1 + x + 2 + x + 3 + . . . . . . .. . .. . . . + x + 100 = 5750
=> ( 1 + 2 + 3 + . . . + 100) + ( x + x + x . . . . . . . + x ) = 5750 0.5 101 . 50 + 100 x = 5750 0.5 100 x + 5050 = 5750 100 x = 5750 – 5050 0.5 100 x = 700 x = 7 0.5 2 2 9 2 6 2 14 4
5.(2 .3 ) .(2 ) − 2.(2 .3) .3 = a Ta có: A 28 18 29 18 5.2 .3 − 7.2 .3 2đ 18 18 12 28 14 4 5.2 .3 .2 − 2.2 .3 .3 = 0.5 28 18 29 18 − 5.2 .3 7.2 .3 30 18 29 18 Câu 2 5.2 .3 − 2 .3 0.5 = ( 4 điểm ) 28 18 2 .3 (5 − 7.2) 29 18 2 .3 (5.2 −1) 2.9 1 = = = − 2 28 18 2 .3 (5 −14) 9 − b 2đ
S =(3)0+(3)1 + (3)2+(3)3+...+ (3)2015.
3S = (3).[(3)0+(3)1+(3)2 + ....+(3)2015] 0,5 0,5 = (3)1+ (3)2+ ....+(3)2016] Trang 21
3S – S = [(3)1 + (3)2+...+(3)2016] - (3)0-(3)1-...-(3)2015. 0,5 2S = (3)2016 -1. 2016 (3) −1 0,5 S = 2
Gọi số cần tìm là a ta có: (a-6) 11 ;(a-1) 4; (a-11) 19. 0.5 a
(a-6 +33) 11 ; (a-1 + 28) 4 ; (a-11 +38 ) 19.
(a +27) 11 ; (a +27) 4 ; (a +27) 19. 0.5
Do a là số tự nhiên nhỏ nhất nên a+27 nhỏ nhất 0.5 Câu 3
Suy ra: a +27 = BCNN (4 ;11 ; 19 ) . 0.5 (4 điểm)
Từ đó tìm được : a = 809 b Số thứ nhất bằng: 9 6 21 : = (số thứ hai) 2đ 11 7 22 0.5 9 2 27 Số thứ ba bằng: : = (số thứ hai) 0.5 11 3 22
Số thứ hai bằng: 22 (số thứ hai) 22
Tổng của 3 số bằng: 22 + 21 + 27 70 (số thứ hai) = (số thứ hai) 0.5 22 22 Số thứ hai là : 210 : 70 21
= 66 ; số thứ nhất là: . 66 = 63 ; số thứ 0.5 22 22 27 3 là: .66 = 81 22
1) Trên tia BA ta có BK = 2 cm. BA = 7cm nên BK< BA do
đó điểm K nằm giữa A và B. Suy ra AK + KB = AB hay AK + 2 2,5 a
= 7  AK = 5 cm. Trên tia AB có điểm I và K mà AI < AK (và 4
<5) nên điểm I nằm giữa A và K
2) Do I nằm giữa A và K nên AI + IK = AK. Hay 4 + IK = 5  1,5 IK = 5 – 4 = 1. Câu 4 b
Vì A nằm giữa B và C nên BA +AC = BC  BA +AC = 4 (1) 0,5 (6 điểm )
Lập luân  B nằm giữa A và D. 2đ
Theo gt OD < OA  D nằm giữa O và A.
Mà OD + DA = OA  2 + DA =5  DA =3 cm 0,5
Ta có DB + BA = DA  DB +BA = 3 (2) 0,5
Lấy (1) – (2): AC – DB = 1 (3)
Theo đề ra : AC = 2BD thay và (3)
Ta có 2BD – BD = 1  BD = 1 0, 5  AC = 2BD  AC = 2 cm Câu 5
Ta có 32 + 33+ 34+…… + 3101 ( 2 điểm )
= (32+ 33+ 34 + 35) + (36 + 37 + 38 + 39)+…+ (398 + 399 + 3100 + 3101) 0,5
= 31(3+32+33+34) + 35(3+32+33+34) +…+397(3+32+33+34) 0,5 Trang 22
= 31.120 + 35.120 +…+397.120 0,5
= 120(31 + 35 +…+397) 120 (đpcm) 0,5
Lưu ý .Học sinh có cách giải khác đúng vẫn cho điểm tối đa.
PHÒNG GIÁO DỤC VÀ ĐÀO TẠO
ĐỀ KHẢO SÁT HỌC SINH GIỎI LỚP 6 CẤP HUYỆN Môn: Toán – Lớp 6
Thời gian làm bài: 120 phút (Không kể thời gian giao đề) ĐỀ 7 Bài 1: (4,0 điểm) a) Cho n = 7 5
a + 8b4. Biết a – b = 6 và n chia hết cho 9. Tìm a và b.
b) Tìm các số tự nhiên x, y sao cho: 5x + 12y = 26. Bài 2: (4,0 điểm) a
a)Tìm các số nguyên a, b biết rằng: 1 1 − = 7 2 b + 3 b) Tìm x, biết : ( 1 1 1 22 + + . . . + ) . x = 3 . 2 . 1 2.3.4 10 . 9 . 8 45 Bài 3: (4,0 điểm)
a) Cùng một công việc nếu mỗi người làm riêng thì 3 người A, B, C hoàn thành
công việc trong thời gian lần lượt là 6 giờ, 8 giờ, 12 giờ. Hai người B và C làm chung
trong 2 giờ sau đó người C chuyển đi làm việc khác, người A cùng làm với người B tiếp
tục công việc cho đến khi hoàn thành. Hỏi người A làm trong mấy giờ?
b) Cho D = 5 + 52 + 53 + 54 + ... + 519 + 520. Tìm số dư khi chia D cho 31. Bài 4: (4,0 điểm) 1930 + 5 1931 + 5
a) So sánh M và N biết: M = ; N = 1931 + 5 1932 + 5
b) Thực hiện tính: 1 1 1 1
E = 1+ (1 + 2) + (1 + 2 + 3)+ (1 + 2 + 3 + 4) + ... + (1 + 2 + ... + 200) 2 3 4 200 Bài 5: (4,0 điểm)
a) Cho: xOy = 1200, xOz = 500. Gọi Om là tia phân của góc yOz . Tính xOm
b) Cho 20 điểm phân biệt trong đó có đúng 7 điểm thẳng hàng, ngoài ra không có ba
điểm nào thẳng hàng. Cứ qua hai điểm ta vẽ được một đường thẳng. Hỏi từ 20 điểm đó vẽ
được tất cả bao nhiêu đường thẳng? Trang 23
PHÒNG GIÁO DỤC VÀ ĐÀO TẠO
ĐỀ KHẢO SÁT HỌC SINH GIỎI LỚP 6 CẤP HUYỆN Môn: Toán – Lớp 6
Thời gian làm bài: 120 phút (Không kể thời gian giao đề)
HƯỚNG DẪN CHẤM ĐỀ 7 Câu Phần
Nội dung cần trình bày Điểm 1 a Cho n = 7 5
a + 8b4. Biết a – b = 6 và n chia hết cho 9. Tìm a và b. (4đ) (2đ)
Ta có: n = 7a5 + 8b4 9
M  7 + a + 5 + 8 + b + 4 9 M 0,5 0,5  24 + a +b 9
M  a + b  3;1  2 (vì a + b < 19).
Mà a – b = 6 nên a + b > 3. Do đó a + b = 12. 0,5
Kết hợp với a – b = 6, suy ra a = 9, b = 3. 0,5 b
Tìm các số tự nhiên x, y sao cho: 5x + 12y = 26. (2đ)
Ta có 122 = 144 > 26 và y ∈ N => 0  y  1 => y ∈ {0; 1} 0,5
+) Với y = 1 => 5x + 121 = 26 => 5x = 14 => không tìm được 0,5 x ∈ N.
+) Với y = 0 => 5x + 120 = 26 => 5x = 25=52 => x = 2 1 2 a
Tìm các số nguyên a, b biết rằng: a 1 1 − = (4đ) (2đ) 7 2 b + 3 − a 1 1 2a 7 1 − =  =
 (2a − 7)(b + 3) =14. 0,5 7 2 b + 3 14 b + 3 0,5 Do ,
a bZ nên 2a – 7  Ư(14) 0,5
Vì 2a – 7 lẻ nên 2a – 7   7 − ; 1 − ;1; 
7  a 0;3;4;  7 .
Từ đó tính được: (a; b) = (0; -5), (3; -17), (4; 11), (7; -1) 0,5 b Tìm x , biết : ( 1 1 1 22 + + . . . + ) . x = (2đ) 3 . 2 . 1 2.3.4 10 . 9 . 8 45 1 1 1 22 0,5 ( + + . . . + ) . x = 3 . 2 . 1 2.3.4 10 . 9 . 8 45 1 1 1 1 1 1 1 22 0,5 ( − + − + + − ) . x = 2 1.2 2.3 2.3 3.4 8.9 9.10 45  1 1 1 − 22 0,5 ( ) . x = 2 2 90 45  x = 2 0,5 3 a
Cùng một công việc nếu mỗi người làm riêng thì 3 người A, B, (4đ) (2đ)
C hoàn thành công việc trong thời gian lần lượt là 6 giờ, 8 giờ,
12 giờ. Hai người B và C làm chung trong 2 giờ sau đó người
C chuyển đi làm việc khác, người A cùng làm với người B tiếp
tục công việc cho đến khi hoàn thành. Hỏi người A làm trong mấy giờ? 1
Trong 1 giờ mỗi người A, B, C lần lượt làm được (CV ) , 6 Trang 24 1 1 1 1 5 (CV ) ,
(CV ) , B và C làm được + = (CV ) 8 12 8 12 24 5 5
2 giờ B và C làm được  2 = (CV ) 24 12 0,5 5 7 A và B làm được 1− = (CV ) 12 12 0,5 1 1 7
1 giờ A và B cùng làm được: + = (CV ) 6 8 24 0,5 7 7
Thời gian A cùng làm với B là: : = 2 giờ. 12 24 0,5 b
Cho D = 5 + 52 + 53 + 54 + ... + 519 + 520. Tìm số dư khi chia D (2đ) cho 31.
D + 1 = (1+ 5 + 52 ) + 53(1+ 5 + 52 ) + 56(1+ 5 + 52 ) + ... + 0,5 518(1+ 5 + 52 )
Do 1 + 5 + 52 = 31 nên D + 1 chia hết cho 31 0,5 => D chia 31 dư 30. 1 4 a 30 + 31 + So sánh M và N biết M = 19 5 19 5 (4đ) ; N = (2đ) 1931 + 5 1932 + 5 30 + 30 + 31 + 19 5 19 .( 19 ) 5 19 95 90 M = nên 19M = = = 1 + 31 31 31 31 0,5 19 + 5 19 + 5 19 + 5 19 + 5 31 31 32 19 + 5 19 .( 19 + ) 5 19 + 95 90 N = nên 19N = = = 1 + 32 32 32 32 0,5 19 + 5 19 + 5 19 + 5 19 + 5 90 90 Vì > 31 32 19 + 5 19 + 5 90 90 Suy ra 1 + > 1 + 31 32 0,5 19 + 5 19 + 5 Hay 19M > 19N 0,5 Nên M > N b Thực hiện tính: (2đ) 1 1 1 1
E = 1+ (1 + 2) + (1 + 2 + 3)+ (1 + 2 + 3 + 4) + ... + (1 + 2 + ... + 200) 2 3 4 200 ( n n + 1)
Từ 1+ 2 + 3+ ... + n = được: 2 1 2.3 1 3.4 1 4.5 1 200.201 E = 1+ . + . + . +...+ . 2 2 3 2 4 2 200 2 3 4 5 201 =1+ + + +...+ 0,5 2 2 2 2 1 1 2 3 4 5 201 E + = + + + + + ...+ 2 2 2 2 2 2 2 0,5 1 = (1+ 2 + 4 + ...+ 20 ) 1 2 1 1 1 201.202 1 + + + + − = − E = (1 2 4 ... 20 ) 1 . 2 2 2 2 2 0,5 Trang 25 = 10150 0,5 5 a Cho:
xOy = 1200, xOz = 500. Gọi Om là tia phân của góc yOz . (4đ) (2đ) Tính xOm
a, Trường hợp 1 : 2 tia Oy, Oz thuộc cùng một nửa mặt phẳng có bờ Ox
xOz = 500 < 1200 = xOy nên tia Oz nằm giữa Ox và Oy. 0,5
yOz = xOy xOz = 1200 – 500 = 700 yOz 0,5 zOm = = 350 2
xOm = 350 + 500 = 850
b, Trường hợp 2 : 2 tia Oy, Oz thuộc hai nửa mặt phẳng đối nhau bờ Ox. 0,5
Từ đầu bài ta có Ox nằm giữa 2 tia Oy và Oz.
yOz = 1200 + 500 = 1700 0,5 170 zOm = = 850 2
xOm = 850 – 500 = 350 b
Cho 20 điểm phân biệt trong đó có đúng 7 điểm thẳng hàng, (2đ)
ngoài ra không có ba điểm nào thẳng hàng. Cứ qua hai điểm ta
vẽ được một đường thẳng. Hỏi từ 20 điểm đó vẽ được tất cả bao nhiêu đường thẳng?
Nếu trong 20 điểm không có ba điểm nào thẳng hàng thì vẽ được 20.(20 − ) 1 =190 . (Đường thẳng). 0,5 2
Trong 7 điểm không có ba điểm nào thẳng hàng thì tạo thành 7 .( 7 − ) 1 0,5 = 21 (Đường thẳng). 2
Vì 7 điểm thẳng hàng tạo thành 1 đường thẳng nên số đường
thẳng giảm 21 - 1 = 20 (Đường thẳng). 0,5
Vậy có 190 – 20 = 170 (Đường thẳng). 0,5
Lưu ý : Mọi cách giải khác đúng đều cho điểm tối đa Trang 26
PHÒNG GIÁO DỤC VÀ ĐÀO TẠO
ĐỀ KHẢO SÁT HỌC SINH GIỎI LỚP 6 CẤP HUYỆN Môn: Toán – Lớp 6
Thời gian làm bài: 120 phút (Không kể thời gian giao đề) ĐỀ 8
Bài 1: (4.0 điểm) Thực hiện phép tính
a) A=1.2.3…9 - 1.2.3…8 - 1.2.3…8.8 (3.4.2 )2 16 b) B= 13 11 9 11.2 .4 −16 131313 131313 131313 c) C = 70.( + + ) 565656 727272 909090 1 1 1 1
d) Thực hiện phép tính: B = + + +...+ 4.9 9.14 14.19 64.69
Bài 2: (4.0 điểm) Tìm x biết : 1 2 7 a) − 2x + = 2 3 3
b) (3x − 54).8 : 4 = 18 
c) ( x − )5 = ( x − )3 2 15 2 15
d) x + (x + 1) + (x + 2) +…+ ( x + 2013) = 2035147
Bài 3: (4.0 điểm)
a). Tìm số tự nhiên nhỏ nhất, biết rằng số đó khi chia cho 3, cho 4, cho 5, cho 6 đều dư là
2, còn chia cho 7 thì dư 3.
b) Tìm x, y nguyên biết: x + y + xy = 40
c) Khi chia một số tự nhiên a cho 4 ta được số dư là 3 còn khi chia a cho 9 ta được số
dư là 5. Tìm số dư trong phép chia a cho 36.
Bài 4: (6.0 điểm) Cho góc x
By = 550. Trên các tia Bx; By lần lượt lấy các điểm A, C sao
cho A  B; C  B. Trên đoạn thẳng AC lấy điểm D sao cho ABD = 300
a. Tính độ dài AC, biết AD = 4cm, CD = 3cm.
b. Tính số đo của DBC .
c. Từ B vẽ tia Bz sao cho D
Bz = 900. Tính số đo ABz . 2 3 4 2016 2017
Bài 5: (2.0 điểm) Cho tổng T = + + +......... ..... + + 1 2 2 2 3 2 2015 2 2016 2 So sánh T với 3
---------------------------------------
- Họ và tên thí sinh: …………………………………..; Số báo danh ………………
Chú ý: Cán bộ coi giao lưu không được giải thích gì thêm. Trang 27
PHÒNG GIÁO DỤC VÀ ĐÀO TẠO
ĐỀ KHẢO SÁT HỌC SINH GIỎI LỚP 6 CẤP HUYỆN Môn: Toán – Lớp 6
Thời gian làm bài: 120 phút (Không kể thời gian giao đề)
HƯỚNG DẪN CHẤM ĐỀ 8
Bài 1: Thực hiện phép tính (4.0
a) A = 1.2.3…9- 1.2.3…8- 1.2.3…8.8
điểm) = 1.2.3…8.(9 - 1 - 8) 0,5 = 0 0,5 ( 3.4.2 )2 16 b) B = 13 11 9 11.2 .4 −16 (3.2 .2 )2 2 16 = 13 22 36 11.2 .2 − 2 0,25 36 9.2 = 35 36 11.2 − 2 0,25 36 9.2 = 35 2 .(11− 2) 0,25 36 9.2 = = 2 0,25 35 2 .9 131313 131313 131313 c) C = 70.( + + ) 565656 727272 909090 13 13 13 = 70.( + + ) 0,25 56 72 90 1 1 1 = 70.13.( + + ) 7.8 8.9 10 . 9 0,25 1 1 = 70.13.( - ) 7 10 0,25 = 39 1 1 1 1 0,25 d ) B = + + +...+ 4.9 9.14 14.19 64.69 1 1 1 1 1 1 1 1 1 = ( − + − + − +...+ − ) 5 4 9 9 14 14 19 64 69 0,5 1 1 1 = ( − ) 5 4 69 0,25 13 = 276 0,25 Bài 2: 1 2 1 2 1 10 a)
− 2x + = 4  − 2x = 4 -  − 2x = (4.0 2 3 2 3 2 3 0,25 điểm) 1 10 1 10 −17 −17 TH1: - 2x =  2x = -  2x =  x = 0,25 2 3 2 3 6 12 1 10 1 10 23 23 TH2: - 2x = −  2x = +  2x =  x = 0,25 2 3 2 3 6 12 Trang 28 −17 23 0,25 Vậy x= ; x = 12 12
b) (3x − 54).8 : 4 = 18  ( 3x − 54).8 = 72 0,25 3x − 54 = 9 0,25 3x = 63 0,25 x = 21 Vậy x = 21 0,25
c) ( x − )5 = ( x − )3 2 15 2 15 (
2x −15)5 − (2x −15)3 = 0
(2x −15)3.(2x −15)2 −1 = 0   ( 2x −15)3 = 0   0,25
(2x −15)2 −1= 0
*2x −15 = 0  x = 7,5 0,25
*(2x −15)2 −1 = 0  (2x −15)2 2 =1  2x −15 = 1 x = 8     0,25 2x −15 = 1 − x = 7
Vậy x 7;7,5;  8 0,25
d) x + (x + 1) + (x + 2) +…+ ( x + 2013) = 2035147 0,25
2014x + (1+2+3+…+2013) = 2035147 0,25 2014x + 2027091 = 2035147 0,25 2014x = 8056 x = 4 Vậy x = 4 0,25 Bài 3:
a) Gọi số tự nhiên cần tìm là a (4.0
Vì a chia cho 3, cho 4, cho 5, cho 6 đều dư là 2 nên a - 2 chia hết cho 3,
điểm) cho 4, cho 5, cho 6 do đó a - 2 là BC(3, 4, 5, 6) 0,5 + BCNN(3, 4, 5, 6) = 60 + Lập luận a - 2  0;60;120;180;..  . a 2;62;122;182;..  . 0,5
Mà a là số tự nhiên nhỏ nhất và chia cho 7 thì dư 3 nên a = 122 0,5 b) x + y + xy = 40 (y+1)x + y + 1= 41 (x + 1)(y + 1) = 41 0,25
Mà x, y nguyên => x +1 và y + 1 là ước của 41 0,25 Tính được (x, y)  (  40;0);(0;40);( 2 − ; 4 − 2);( 4 − 2; 2 − ) 0,5 Trang 29
c) Theo đề bài ta có: a = 4p+3 = 9q + 5 ( p, q nguyên) 0,25
Suy ra a + 13 = 4p + 3 + 13 = 4(p + 4) (1) 0,25
a + 13 = 9q + 5 + 13 = 9(q + 2) (2)
Từ (1) và (2) ta nhận thấy a + 13 là bội của 4 và 9 mà (4,9) = 1 nên a 0,5 + 13 là bội của 4.9 = 36 0,25 0,25
Ta có a + 13 = 36k (k nguyên) => a = 36k – 13 = 36(k - 1) + 23 Vậy a chia cho 36 dư 23 Bài 4: (6.0 y điểm) z C D B x A z
a) Vì D thuộc đoạn thẳng AC nên D nằm giữa A và C 0,75 => AC = AD + CD 0,5 = 4 + 3 = 7(cm) 0,5 Vậy AC = 7cm 0.25 1.0
b) Chứng minh tia BD nằm giữa hai tia BA và BC 0.5
ta có đẳng thức: ABC = ABD + DBC 0.5 => DBC = ABC - A
BD = 550 – 300 = 250
c) Xét hai trường hợp ( Học sinh vẽ hình trong hai trường hợp)
- Trường hợp 1: Tia Bz và BA nằm trên cùng 1 nửa mặt phẳng có bờ là BD
+ Lập luận tia BA nằm giữa hai tia Bz và BD 0.5 Tính được ABz = DBz - ABD= 0 0 0 90 − 30 = 60 0.5
- Trường hợp 2: Tia Bz, và BA nằm trên hai nửa mặt phẳng đối nhau có bờ là BD
+ Lập luận tia BD nằm giữa hai tia Bz và BA 0.5 Tính được ABz = DBz + ABD = 0 0 0 90 + 30 = 120 0.5 Bài 5: 2 3 4 2016 2017 T = + + + ......... . . ... + + (2.0 1 2 2 2 3 2 2015 2 2016 2 điểm) 3 4 2016 2017 2T = 2 + + +......... ..... + + 1 2 2 2 2014 2 2015 2 2T –T= Trang 30 3 2 4 3 2016 2015 2017 2016 2017 2 + - + - +…….+ - + - - 1 2 1 2 2 2 2 2 2014 2 2014 2 2015 2 2015 2 2016 2 1 1 1 2017 T= 2+ + +………+ - 1 2 2 2 2015 2 2016 2 0.75 Đặt N = 1 1 1 + +………+ 1 2 2 2 2015 2 1 1 1 Ta có 2N = 1+ + +………+ 1 2 2 2 2014 2 1 2N-N= 1- 2015 2 Vậy N < 1 0.5 2017 2017 Nên T< 2+1- =3- 0.5 2016 2 2016 2 Vậy T<3 0.25 Ghi chú:
- Bài hình không có hình vẽ hoặc hình vẽ sai thì không chấm điểm.
- Học sinh làm cách khác mà đúng thì vẫn cho điểm tối đa.
PHÒNG GIÁO DỤC VÀ ĐÀO TẠO
ĐỀ KHẢO SÁT HỌC SINH GIỎI LỚP 6 CẤP HUYỆN Môn: Toán – Lớp 6
Thời gian làm bài: 120 phút (Không kể thời gian giao đề) ĐỀ 9
Câu 1: (4 điểm) Tính:
a) A =1+ 2 − 3 − 4 + 5 + 6 − 7 −8 + 9 +... + 2013 + 2014 − 2015 − 2016 2.4.10 + 4.6.8 +14.16.20 b) B = 3.6.15 + 6.9.12 + 21.24.30
Câu 2: (6 điểm) 2014 10 + 2016 2015 10 + 2016 a) So sánh A = và B = 2015 10 + 2016 2016 10 + 2016 b) Tìm x biết: 1 1 1 1 119 ( + + +...+ ).x = 1.2.3.4 2.3.4.5 3.4.5.6 7.8.9.10 720
c) Chứng minh rằng: nếu p và p2+2 là các số nguyên tố thì p3+2 cũng là số nguyên tố.
Câu 3: (4 điểm) n +
a) Tìm số tự nhiên n để phân số 2
1 là phân số rút gọn được. n + 2
b) Trong đợt tổng kết năm học tại một trường THCS, tổng số học sinh giỏi của ba lớp 6A,
6B, 6C là 90 em. Biết rằng 2 số học sinh giỏi của lớp 6A bằng 1 số học sinh giỏi của lớp 6B và 5 3
bằng 1 số học sinh giỏi của lớp 6C. Tính số học sinh giỏi mỗi lớp. 2 Trang 31
Câu 4: (4 điểm) Cho tam giác ABC có · 0
ACB = 60 , AB=6cm. Trên cạnh AB lấy điểm D (D khác A,B) sao cho AD=2cm.
a) Tính độ dài đoạn thẳng BD. b) Tính số đo của · DCB biết · 0 ACD = 20 . c) Dựng tia Cx sao cho · 0 DCx = 90 . Tính · ACx .
d) Trên cạnh AC lấy điểm E (E khác A,C). Chứng minh hai đoạn thẳng CD và BE cắt nhau.
Câu 5: (2 điểm) Tìm bộ ba số nguyên dương a, b, c sao cho: 1 1 1 4 + + = a b c 5
------ HẾT ------
Cán bộ coi thi không giải thích gì thêm
Họ và tên thí sinh: ................................................Số báo danh:...........................
Giám thị 1 (Họ tên và ký)...........................................................................................................
Giám thị 2 (Họ tên và ký)...........................................................................................................

PHÒNG GIÁO DỤC VÀ ĐÀO TẠO
ĐỀ KHẢO SÁT HỌC SINH GIỎI LỚP 6 CẤP HUYỆN Môn: Toán – Lớp 6
Thời gian làm bài: 120 phút (Không kể thời gian giao đề)
HƯỚNG DẪN CHẤM ĐỀ 9 Câu Đáp án Điểm 1.1 (2.0
Tính A =1+ 2 − 3 − 4 + 5 + 6 − 7 −8 + 9 +... + 2013 + 2014 − 2015 − 2016 điểm)
A = 1+ 2 − 3 − 4 + 5 + 6 − 7 −8 + 9 +... + 2013 + 2014 − 2015 − 2016
Tính được số các số hạng của A là (2016 - 1) : 1 + 1 = 2016 số hạng 0,75
Nhóm 4 số hạng liên tiếp vào một nhóm:
A = (1+ 2 −3− 4) + (5 + 6 − 7 −8) +...+ (2013+ 2014 − 2015− 2016) 0.75 A = 4 − + ( 4 − ) +...+ ( 4 − ) = 4 − .504 = 2 − 016 144444442 444444443 ó c 504 sô' 0.5 Vậy A=-2016 1.2 2.4.10 + 4.6.8 +14.16.20 B = (2.0 3.6.15 + 6.9.12 + 21.24.30 điểm) 2.4.10 + 4.6.8 +14.16.20 8.(1.2.5 + 2.3.4 + 7.8.10) 8 B = = = 3.6.15 + 6.9.12 + 21.24.30 27.(1.2.5 + 2.3.4 + 1.75 7.8.10) 27 Vậy B= 8 0.25 27 2.1 2014 10 + 2016 2015 10 + 2016 (2.0 So sánh A = B = 2015 2016 điểm + và + ) 10 2016 10 2016 Trang 32 2014 2014 2016 10 + 2016 (10 + 2016)(10 + 2016) Ta có A = = 2015 2015 2016 10 + 2016 (10 + 2016)(10 + 2016) 4030 2014 2016 2 10 + 2016.(10 +10 ) + 2016 0.75 = 2015 2016 (10 + 2016)(10 + 2016) 4030 2014 2 10 + 2016.10 .101+ 2016 = (1) 2015 2016 (10 + 2016)(10 + 2016) 2015 2015 2015 10 + 2016 (10 + 2016)(10 + 2016) Ta có B = = 2016 2016 2015 10 + 2016 (10 + 2016)(10 + 2016) 4030 2015 2 10 + 2.2016.10 + 2016 = 2016 2015 (10 + 2016)(10 + 2016) 0.75 4030 2014 2 10 + 20.2016.10 + 2016 = (2) 2016 2015 (10 + 2016)(10 + 2016)
Từ (1) và (2) suy ra A>B 0.25 Vậy A>B 0.25 2.2 Tìm x biết: 1 1 1 1 119 ( + + +...+ ).x = (1) (2.0 1.2.3.4 2.3.4.5 3.4.5.6 7.8.9.10 720 điểm) 1 1 1 1 Ta có: + + +...+ 1.2.3.4 2.3.4.5 3.4.5.6 7.8.9.10 1 1 1 1 1 1 1 = ( − + − + ...+ − ) 3 1.2.3 2.3.4 2.3.4 2.3.4 7.8.9 8.9.10 1 1 1 1 119 1,25 = ( − ) = . 3 6 720 3 720 Nên từ (1) suy ra: 1 119 119 0.5 . .x = =>x=3 3 720 720 Vậy x=3 0.25 2.3
Chứng minh rằng: nếu p và p2+2 là các số nguyên tố thì p3+2 cũng là số nguyên tố. (2.0 điểm)
Ta nhận xét rằng mọi số nguyên tố lớn hơn 3 thì chia cho 3 đều có dạng 0.5 p=3k+1 hoặc p=3k+2 ( * k N )
Với p=3k+1 thì p2+2=9k2+6k+3 chia hết cho 3.
Với p=3k+2 thì p2+2=9k2-6k+6 chia hết cho 3 0.5
Vì p là nguyên tố nên p  2 khi đó trong cả 2 trường hợp trên thì p2+2 đều lớn
hơn 3 và chia hết cho 3. Tức là p2+2 là hợp số 0.75
=> p2+2 chỉ là nguyên tố khi p=3 (khi đó p2+2=11 là số nguyên tố)
=> p3+2=27+2=29 là số nguyên tố
Vậy nếu p và p2+2 là các số nguyên tố thì p3+2 cũng là số nguyên tố. 0.25 3.1 +
Tìm số tự nhiên n để phân số 2n 1 là phân số rút gọn được. (2.0 n + 2 điểm) Gọi d là ƯCLN(2n+1,n+2) (d * N ) 0.75 Trang 33
Ta có 2n+1Md, n+2Md => [(2n+4)-(2n+1)]Md => 3Md Vì d * N nên d{1;3} + 0.75
Để phân số 2n 1 rút gọn được thì d=3 n + 2 => n+2=3k ( * k N ) => n=3k-2 ( * k N ) 0.5 + Vậy với n=3k n -2 ( *
k N ) thì phân số 2
1 là phân số rút gọn được. n + 2
Trong đợt tổng kết năm học tại một trường THCS, tổng số học sinh giỏi của ba lớp 6A, 3.2
6B, 6C là 90 em. Biết rằng 2 số học sinh giỏi của lớp 6A bằng 1 số học sinh giỏi của (2.0 5 3
điểm) lớp 6B và bằng 1 số học sinh giỏi của lớp 6C. Tính số học sinh giỏi mỗi lớp. 2
Số học sinh giỏi của lớp 6B bằng 2 1 6 :
= ( số học sinh giỏi lớp 6A) 0.5đ 5 3 5
Số học sinh giỏi lớp 6C bằng 2 1 4 :
= ( số học sinh giỏi lớp 6A) 0.5đ 5 2 5
Số học sinh giỏi của cả 3 lớp bằng 6 4 0.5đ 1 +
+ = 3 ( số học sinh giỏi lớp 6A) 5 5
Vậy số học sinh giỏi lớp 6A là 90: 3 = 30 học sinh, của lớp 6B là 36 học sinh và
của lớp 6C là 24 học sinh 0.5đ Cho tam giác ABC có · 0
ACB = 60 , AB=6cm. Trên cạnh AB lấy điểm D sao cho AD=2cm. 4
a) Tính độ dài đoạn thẳng BD. (4.0 điểm
b) Tính số đo của góc DCB biết · 0 ACD = 20 .
) c) Dựng tia Cx sao cho · 0 DCx = 90 . Tính · ACx .
d) Trên cạnh AC lấy điểm E. Chứng minh hai đoạn thẳng CD và BE cắt nhau. E E
Trường hợp 1 Trường hợp 2
a) D nằm giữa A và B => AD+BD=AB=>BD=6-2=4cm 0.75 KL... 0.25 Trang 34
b) Tia CD nằm giữa hai tia CA và tia CB => · · ·
ACD + DCB = ACB 0.75 => · DCB =400 KL... 0.25 c) Xét hai trường hợp:
- Trường hợp 1: Hai tia CD và Cx nằm về một phía so với đường thẳng CB
Tính được góc ACx = 900- · ACD = 700 0.5 K.L...
- Trường hợp 2: Hai tia CD và Cx nằm về hai phía so với đường thẳng CB
Tính được góc ACx = 900 + · ACD = 1100 0.5 K.L ...
- Xét đường thẳng CD.
Do CD cắt AB nên đường thẳng CD chia mặt phẳng làm 2 nửa: 1 nửa MP có bờ
CD chứa điểm B và nửa MP bờ CD chứa điểm A => tia CA thuộc nửa MP chứa điểm A.
E thuộc đoạn AC => E thuộc nửa MP bờ CD chứa điểm A
=> E và B ở 2 nửa MP bờ CD 0.5
=> đường thẳng CD cắt đoạn EB
- Xét đường thẳng BE.
Lập luận tương tự: ta có đường thẳng EB cắt đoạn CD.
Vậy 2 đoạn thẳng EB và CD cắt nhau. 0.5 5
Tìm bộ ba số nguyên dương a, b, c sao cho: 1 1 1 4 + + = (1.0 a b c 5 điểm)
Không làm mất tính tổng quát, ta giả sử: a  b c khi đó ta có: 3 4 15  , a  0.5 a 5 4
Nếu a=1 thì không thể được, do đó a= 2 hoặc a=3 Nếu a=2 thì 1 1 3 + = b c 10 2 3 20 Suy ra  , b  b 10 3 0.5
Suy ra b=4 hoặc b= 5 hoặc b=6 vì 3 1 < 10 3
Suy ra các số a, b, c thỏa mãn là (a=2,b=4,c=20) và (a=2,b=5,c=10) Nếu a=3 thì 1 1 7 + = b c 15 0.5 từ đó 2 7 30  , b 
suy ra b=3 hoặc b=4. Không có trường hợp nào thỏa mãn b 15 7
K.L có 12 bộ số thỏa mãn là các hoán vị của hai bộ ba số (2,4,20) và (2,5,10) 0.5 Điểm toàn bài 20 điểm Ghi chú:
- Bài hình không có hình vẽ hoặc hình vẽ sai thì không chấm điểm.
- Học sinh làm cách khác mà đúng thì vẫn cho điểm tối đa. Trang 35
PHÒNG GIÁO DỤC VÀ ĐÀO TẠO
ĐỀ KHẢO SÁT HỌC SINH GIỎI LỚP 6 CẤP HUYỆN Môn: Toán – Lớp 6
Thời gian làm bài: 120 phút (Không kể thời gian giao đề) ĐỀ 10
Câu 1(2,0 điểm): Tính hợp lí
a) 21.72 - 11.72 + 90.72 + 49.125.16 15 9 20 9 5.4 .9 − 4.3 .8 c) 9 19 29 6 5.2 .6 − 7.2 .27
Câu 2(6,0 điểm): Tìm x là số tự nhiên, biết: 2 2 , 0 4 + − 1 3 x + 1 8 a) x : ( 9 - ) = 9 11 b) = 2 2 8 8 2 x + 1 6 , 1 + − 9 11
c) 52x - 3 – 2.52 = 52.3 d) 2x − 7 = 20 + 5.( 3 − ) Câu 3(6,0 điểm):
a) Tìm số nguyên x và y, biết : xy - x + 2y = 3.
b) Tìm các số tự nhiên x, y biết: 2x + 1 . 3y = 12x
c) Cho số 155*710* 4*16 có 12 chữ số. Chứng minh rằng nếu thay các dấu (*) bởi
các chữ số khác nhau trong ba chữ số 1; 2; 3 một cách tuỳ ý thì số đó luôn chia hết cho 396.
d) Tìm số tự nhiên n để biểu thức sau là số tự nhiên: 2n + 2 5n +17 3n B = + − n + 2 n + 2 n + 2
Câu 4(5,0 điểm):
Cho đoạn thẳng AB = 5cm. Lấy điểm M thuộc đoạn thẳng AB, trên tia đối của tia AB lấy điểm N sao cho AN = AM. a) Tính BN khi BM = 2cm.
b) Trên cùng một nửa mặt phẳng có bờ là đường thẳng AB, vẽ các tia Ax và Ay sao cho 0 0
BAx = 40 , BAy =110 . Tính yAx, NAy .
c) Xác định vị trí của điểm M trên đoạn thẳng AB để đoạn thẳng BN có độ dài lớn nhất. Câu 5(1,0 điểm):
Tìm số tự nhiên n và chữ số a biết rằng: 1 + 2 + 3 + …….+ n = aaa
.................................... Hết ......................................
Họ và tên thí sinh: ........................................................ Số báo danh: .................... Trang 36
PHÒNG GIÁO DỤC VÀ ĐÀO TẠO
ĐỀ KHẢO SÁT HỌC SINH GIỎI LỚP 6 CẤP HUYỆN Môn: Toán – Lớp 6
Thời gian làm bài: 120 phút (Không kể thời gian giao đề)
HƯỚNG DẪN CHẤM ĐỀ 10 Câu Nội dung Điểm
a) 21.72 - 11.72 + 90.72 + 49.125.16 = 72(21 – 11 + 90) + 49.125.16 0.5đ
= 49. 100 + 49. 100. 20 = 49.100(1 + 20) = 49.100.21 0.5đ 15 9 20 9 0.5đ 1 5.4 .9 − 4.3 .8 30 18 2 20 27 5.2 .3 − 2 .3 .2 b) = (3,0đ) 9 19 29 6 5.2 .6 − 7.2 .27 9 19 19 29 18 5 .2 .2 .3 − 7.2 .3 29 18 2 2 .3 (5.2 − 3 ) = = 2 0.5đ 28 18 2 .3 (5.3 − 7.2) 2 2 2 2 , 0 4 + − 0, 4 + − 1 3 0.5đ a) x : ( 7 11 9 - ) = 9 11  x :8 = 2 2 8 8  2 2  6 , 1 + − 4 0, 4 + −   9 11  7 11  1 1.0đ x : 8 = x = 2 .Vậy x = 2 4 x + 1 8 0.75đ b) =
(x + 1) 2 = 16 = (  4)2 2 x + 1 2 *) x + 1 = 4 x = 3
(6,0đ) *) x + 1 = - 4 x = - 5 . 0.5đ Do x N nên x = 3. 0.25đ
c) 52x - 3 – 2.52 = 52.3  52x - 3 = 52.3 + 2.52 0.5đ
52x - 3 = 52.5 52x - 3 = 53 0.5đ
 2x - 3 = 3  2x = 6 x = 3. Vậy x = 3 0.5đ
d) 2x − 7 = 20 + 5.( 3
− )  2x − 7 = 5  2x − 7 = 5  0.75đ
*) 2x – 7 = 5 2x =12  x = 6
*) 2x – 7 = - 5 2x = 2  x = 1 0.5đ Vậy x 6;  1 0.25đ
a) Tìm số nguyên x và y, biết : xy - x + 2y = 3.
xy - x + 2y = 3  ( xy – x) + (2y – 2) = 1
x( y – 1) + 2( y – 1) = 1 (y – 1)( x + 2) = 1 0.75đ y −1 =1 y = 2 *)    x + 2 =1 x = 1 − y −1 = 1 − y = 0   *)  x + 2 = 1 − x = 3 − 3 (6,0đ)
Vậy x = - 1 ; y = 2 hoặc x = -3 ; y = 0 0.75đ Trang 37
b) 2x + 1 . 3y = 12x  2x + 1 . 3y = (4.3)x = 22x.3x 0.5đ 2 x y  2 3 x 1 =
 2 − = 3yx x 1 0.5đ 2 + 3x 0.5đ
Nhận thấy : ( 2, 3) = 1  x – 1 = y - x = 0  x = y = 1
c) Ta thấy, vị trí của các chữ số thay thế ba dấu sao trong số trên đều ở
hàng chẵn và vì ba chữ số đó đôi một khác nhau, lấy từ tập hợp   3 ; 2 ; 1
nên tổng của chúng luôn bằng 1+ 2+ 3 = 6.
Mặt khác 396 = 4.9.11 trong đó 4;9;11 đôi một nguyên tố cùng nhau nên ta cần chứng minh
A = 155*710* 4*16 chia hết cho 4 ; 9 và 11. 0.5đ Thật vậy :
*) A  4 vì số tạo bởi hai chữ số tận cùng của A là 16 chia hết cho 4
*) A  9 vì tổng các chữ số chia hết cho 9 :
1+ 5+ 5 +7+ 1 + 4 + 1+ 6 + (*+*+*) = 30 + 6 = 36 chia hết cho 9
*) A  11 vì hiệu số giữa tổng các chữ số hàng chẵn và tổng các chữ số 0.75đ
hàng lẻ là 0, chia hết cho 11.
{1+5+7+4+1)-(5+1+6+(*+*+*)} = 18 – 12 – 6 = 0 0.25đ Vậy A  396 2n + 2 5n +17 3n
2n + 2 + 5n +17 − 3n 4n +19 d) B = + − = = n + 2 n + 2 n + 2 n + 2 n + 2 0,5đ 4n +19 4(n + 2) +11 11 B = = = 4 + n + 2 n + 2 n + 2 0,5đ
Để B là số tự nhiên thì 11 là số tự nhiên n + 2 
11  (n+2)  n + 2  Ư(11) =  1  ; 1   1
Do n + 2 > 1 nên n + 2 = 11  n = 9 0,5đ Vậy n = 9 thì B  N Vẽ hình y x 0.5đ 400 M N ) 4 B (5.0đ) A
a) Vì M thuộc AB nên AM + MB = AB Þ AM + 2 = 5  AM = 3 cm Có AN = AM  AN = 3 cm 1.5đ
Do N thuộc tia đối của tia AB nên điểm A nằm giữa N và B
BN = AB + AN = 5 + 3 = 8 cm. Vậy BN = 8cm
b) + Trên cùng một nửa mặt phẳng có bờ chứa tia AB có: 0.75đ 0 0
BAx  BAy (40 110 ) Þ Tia Ax nằm giữa hai tia AB và Ay nên ta có: BAx + xAy = BAy hay 0 · 0 · 0 0 0
40 + xAy = 110 Þ xAy = 110 - 40 = 70 Trang 38
+ Trên cùng một nửa mặt phẳng có bờ AB, ta có BAy và NAy là hai góc kề bù · · 0 Þ BAy + NAy = 180 0.75đ hay 0 · 0 110 + NAy = 180 · 0 0 0 Þ NAy = 180 - 110 = 70
c) Vì BN = AB + AN = 5 + AN  BN có độ dài lớn nhất khi AN có độ dài lớn nhất
Mà AN = AM  BN có độ dài lớn nhất khi AM có độ dài lớn nhất 1,5đ
Có AM  AB  AM lớn nhất khi AM = AB khi đó điểm M trùng với điểm B.
Vậy khi điểm M trùng với điểm B thì BN có độ dài lớn nhất.
Dãy số 1; 2; ………; n có n số hạng  (n + ). 1 n 1 + 2 +…+ n = 2
Mà 1 + 2 + 3+…..+ n = aaa (n + ). 1 n Suy ra
= aaa = a . 111 = a . 3.37  n(n + 1) = 2.3.37.a 2
Vì tích n(n + 1) Chia hết cho số nguyên tố 37 nên n 0.5đ 37 hoặc n + 1 5 37 (1.0đ) Vì số (n + ).
1 n có 3 chữ số  n+1 < 74  n = 37 hoặc n + 1 = 37 2 +) Với n = 37 thì 37 38 . = 703 ( loại) 2 +) Với n + 1 = 37 thì 36 37 . = 666 ( thoả mãn) 2 0.5đ
Vậy n = 36 và a = 6. Ta có: 1+ 2 + 3+…..+ 36 = 666 Chú ý:
1. Thí sinh có thể làm bài bằng cách khác, nếu đúng vẫn được điểm tối đa.
2. Nếu thí sinh chứng minh bài hình mà không vẽ hình thì không chấm điểm bài hình. Trang 39
PHÒNG GIÁO DỤC VÀ ĐÀO TẠO
ĐỀ KHẢO SÁT HỌC SINH GIỎI LỚP 6 CẤP HUYỆN Môn: Toán – Lớp 6
Thời gian làm bài: 120 phút (Không kể thời gian giao đề) ĐỀ 11
Câu 1: (4 điểm).
1) Tìm tự nhiên n sao cho 4n – 5 chia hết cho 2n – 1.
2) Cho S = 31 + 33 + 35 + ... + 32011 + 32013 + 32015. Chứng tỏ: a) S không chia hết cho 9 b) S chia hết cho 70.
Câu 2: (5 điểm)
a) Tìm x biết: ( x + 1) + ( x + 2) + . . . + ( x + 100) = 5750.
b) Tìm số nguyên x, y biết x2y – x + xy = 6
c) Cho A = 1- 5 + 9 -13 +17 - 21+ ... Biết A = 2013. Hỏi A có bao nhiêu số hạng? Giá trị
của số hạng cuối cùng là bao nhiêu?
Câu 3: (2 điểm) ab
Tìm giá trị nhỏ nhất của phân số a + (ab là số có 2 chữ số) b
Câu 4. (4 điểm) 1
Trong một buổi đi tham quan, số nữ đăng kí tham gia bằng
số nam. Nhưng sau đó một 4 1
bạn nữ xin nghỉ, một bạn nam xin đi thêm nên số nữ đi tham quan bằng số nam. Tính 5
số học sinh nữ và học sinh nam đã đi tham quan.
Câu 5: (5 điểm) 0 1 Cho xOy = 120 , xOz =
xOy . Kẻ tia Om là tia phân giác của góc xOy.Tính số đo mOz 3 . -Hết- Trang 40
HD CHẤM THI HỌC SINH GIỎI CẤP HUYỆN
NĂM HỌC 2019 – 2020
MÔN THI: TOÁN LỚP 6 Câu Nội dung Điểm
1) 4n – 5 = 4n – 2 – 3 = 2(2n – 1) – 3 2n – 1 0.5đ
Vì 2(2n – 1) 2n – 1 nên 3 2n – 1 0.5đ -> 2n – 1  Ư(3)
Lí luận đi đến 2n – 1  − ; 1 ; 1 3 0.5đ Kết luận n   ; 0 ; 1 2 0.5đ
Câu 1 2) a) Vì 3 không chia hết cho 9 0.5đ (4đ)
Các hạng tử còn lại đều chia hết cho 9 Nên S không chia hết cho 9 0.5đ
b) Tính được số số hạng của tổng S là 1008 số hạng 0.5đ
S = (31 + 33 + 35) + (37 + 39 + 311) + ... + (32011 + 32013 + 32015)
S = 3. 91+ 37. 91 + …..+ 32011. 1 91 Kết luận S 7 0.5đ
S = (31 + 33) + (35 + 37) + ... + (32013 + 32015)
S = 3. 10+ 35. 10 + …..+ 32013. 10 10 0.5đ Kết luận S 10 Vì (10, 7) = 1 nên S 70 0.5đ a)
( x + 1) + ( x + 2) + . . . + ( x + 100) = 5750. 1 đ
Tính được 100x + 101. 100 : 2 = 5750 1 đ Kết luận đúng x = 7 Câu 2 b) x2y – x + xy = 6 1 đ (4đ)
 (xy – 1)(x + 1) = 5 = 1.5 = (-1)(-5)
Xét 4 trường hợp và kết luận 1 đ (x;y) = (-2;2), (-4;0).
c) Số số hạng của A là (2013 1) : 4.2 +1 =  1007 (số hạng) 1 đ
Số hạng cuối cùng là: (1007 – 1).4 + 1 = 4025 1 đ ab a 10 + b 9a 9 1 + 1 + Đặt A = = = 0.5đ a + = b a + b a + b b 1 + C â u 3 a Trang 41 (2đ) b b 1đ A Có GTNN 1 + có GTLN  Có GTLN  b = 9 và a = 1 a a Khi đó số ab = 19 GTNN của A là 1,9 0.5đ
Tổng số học sinh nam và nữ dự định đi tham quan và đã đi tham 0.5đ
quan là như nhau nên ta lấy làm đơn vị. 1 1
Số hs nữ đăng kí đi tham quan bằng số nam nên bằng tổng số. 4 5 0.5đ 1
Số hs nữ đã đi tham quan bằng
số nam đã đi tham quan nên bằng 5 0.5đ 1 Câu 4 tổng số. (4đ) 6 1 1 1
Số nữ dự định đi nhiều hơn số nữ đã đi là: - = tổng số hay 5 6 30 1đ 1 học sinh 0.5đ Tổng số hs là 1 1 : = 30 (học sinh) 30 0.5đ
Số hs nữ đã đi tham quan là: 30 . 1 = 5 (học sinh) 6 0.5đ
Số hs nam đã đi tham quan là: 30 – 25 = 5 (học sinh) Tính 0 xOz = 40 0.5đ 2 trường hợp : 0.5đ
a, Trường hợp 1: 2 tia Oy, Oz thuộc cùng một nửa mặt phẳng có bờ Ox m y z Câu 4 (5đ) x O 0.5đ Lí luận để tính 0 xOm = 0 6 Chứng minh tia 0.5đ Oz nằm giữa Ox và Oy. Tính đúng 0 zOm = 0 2 0.5đ
b, Trường hợp 2 : 2 tia Oy và Oz thuộc hai nửa mặt phẳng đối nhau bờ Ox. Trang 42 m y x O z
Chỉ được Ox nằm giữa Om và Oz. 1đ Tinh đúng 0 zOm = 0 10 Chú ý:
1. Thí sinh có thể làm bài bằng cách khác, nếu đúng vẫn được điểm tối đa.
2. Nếu thí sinh chứng minh bài hình mà không vẽ hình thì không chấm điểm bài hình. Trang 43
PHÒNG GIÁO DỤC VÀ ĐÀO TẠO
ĐỀ KHẢO SÁT HỌC SINH GIỎI LỚP 6 CẤP HUYỆN Môn: Toán – Lớp 6
Thời gian làm bài: 120 phút (Không kể thời gian giao đề) ĐỀ 12 Bài 1( 4 điểm)
a, Chứng tỏ 4x + 3y chia hết cho 7 khi 2x + 5y chia hết cho 7
b, Tìm các số tự nhiên có bốn chữ số sao cho khi chia nó cho 130 , cho 150 được các số dư lần lượt là 88 và 108.
Bài 2 ( 5,0 điểm) :  7777 77 7777 77  123498766 a) Tính A = − + − .  
 8585 85 16362 162  987661234
b, Tìm phân số lớn nhất, khi chia các phân số 24 18 và
cho nó ta đều được các thương là 7 11 số nguyên. Bài 3 (2,0 điểm) : a, Cho biết S = 1 1 1 + + 91 ... +
. Chứng minh rằng 1 < S < 101 102 130 4 330
Bài 4 (4,0 điểm): Tổng bình phương của 3 số tự nhiên là 2596. Biết rằng tỉ số giữa số thứ
nhất và số thứ hai là 2 , giữa số thứ hai và số thứ ba là 5 . Tìm ba số đó. 3 6
Bài 5 ( 5,0 điểm) :
Cho tia Oz nằm trong góc vuông xOy. Vẽ tia Ot sao cho Ox là tia phân giác của góc tOz.
Vẽ tia Om sao cho tia Oy là phân giác của góc zOm.
a, Chứng minh rằng tia Om và tia Ot là hai tia đối nhau .
b, Gọi Ox’ là tia đối của tia Ox, biết góc x’Om bằng 300 . Tính góc tOz .
c, Vẽ thêm 2014 tia phân biệt gốc O (không trùng với các tia Ox,Oz,Oy,Om,Ox’ và Ot ).
Hỏi trong hình vẽ có tất cả bao nhiêu góc ? Trang 44
PHÒNG GIÁO DỤC VÀ ĐÀO TẠO
ĐỀ KHẢO SÁT HỌC SINH GIỎI LỚP 6 CẤP HUYỆN Môn: Toán – Lớp 6
Thời gian làm bài: 120 phút (Không kể thời gian giao đề)
HƯỚNG DẪN CHẤM ĐỀ 12 Bài
Tóm tắt nội dung hướng dẫn Điểm Câu a ( 2 điểm) Ta có 4x + 3y 7  4( 4x + 3 y) 7 0,5 đ Bài 1  16x + 12 y 7
(4,0 đ)  14x + 7y + 2x + 5y 7 0,5 đ Mà 14x + 7y = 7(2x + y) 7 0,5 đ Nên 2x + 5y 7 Vậy 4x + 3y 7 khi 2x + 5y 7 0,5 đ Câu b ( 2 điểm)
Gọi số phải tìm là a .
Ta có a + 42 chia hết cho 130 và 150 nên a + 42 là BC(130,150) 0,75đ
Tìm đúng a = 1908; 3858 ;5808; 7758; 9708 ( mỗi giá trị 0,25 đ) 1,25đ Câu a ( 2,0 điểm) 7777 7777 :101 77 = = 0,5 đ 8585 8585 :101 85 Ta có 7777 7777 :101 77 = = 16362 16362 :101 162 0,5 đ Bài 2  75 75 77 77  123498766 0,5 đ A = − + − .  
 85 85 162 162  987661234
( 5,0đ) Vậy A = ( 0 + 0) . 123498766 = 0 0,5 đ 987661234 Câu b(3,0 điểm) x 3 1 3 x 1 x 2 − 1 Từ − = ta có: = − = (x,y  N) 9 y 18 y 9 18 18 0,5 đ 0,5 đ
Suy ra: y(2x-1) = 54 do đó yƯ(54) =  ; 1 ; 2 ; 3 ; 6 ; 18 ; 9 ; 27  54 , 0,5 đ vì 54 là số chẵn mà 2x
-1 là số lẻ nên y là ước chẵn của 54. Trang 45 Vậy y  2;6 18 ; ;  54 0,5 đ Ta có bảng sau: y 2 6 18 54 0,75 đ 2x-1 27 9 3 1 x 14 5 2 1 Vậy (x;y)   ); 2 ; 14 ( ); 6 ; 5 ( ); 18 ; 2 ( ) 54 ; 1 ( 0,25 đ * Chứng minh S < 91 330  1 1 1   1 1   1 1  S = + +...+ + +... + +...        101 102 110  111 120  121 130   1 1 1   1 1   1 1  S < + +...+ + +... + +...       100 100 100  110 110  120 120  0,25 đ Bài 3 1 1 1 1 1 1 S < 10 + 10 + 10 = + + (2 đ) 100 110 120 10 11 12 0,25 đ 66 + 60 + 55 S< 660 181 182 91 S < < hay S < (1) 660 660 330 0,5 đ * Chứng minh 1 < S 4  1 1   1 1   1 1  S > +...+ + +...+ + +...        110 110  120 120  130 130  0,25 đ 1 1 1 1 1 1 S > 10 + 10 + 10 = + + 110 120 130 11 12 13 156 +143 +132 0,25 đ S > 1716 431 429 1 S > > Hay S > 1716 1716 4 Trang 46 Từ (1) và (2) ta có 1 91 < S < 4 330 0,5 đ
Gọi a, b, c là 3 số tự nhiên phải tìm.
Theo đề bài ta có: a 2 b 5 = 1 đ ; = ( 1) và 2 2 2
a + b + c = 2596 (2) b 3 c 6 Bài 4 Từ ( 1) suy ra 2 6 a = ; b c =
b , thay vào (2) ta có: 3 5 (4 đ) 4 36 2 2 2 b + b + b = 2596 9 25 1 đ 649 2  b = 2596 225 2  1 đ b = 900 Tính được b = 30, 2 6 a =
30 = 20;c = 30 = 36 3 5 0,75 đ
Vậy 3 số tự nhiên cần tìm là: 30; 20; 36 0,25 đ y m z x’ 0,5 đ O x Bài 5 (5,0 đ) t
- Bài làm không có hình vẽ không cho điểm.
- Hình vẽ chính xác phần a, b được 0,5 điểm Câu a : 2,0 điểm
* Chứng minh góc tOz + góc zOm = 1800
Tia Oz nằm trong góc xOy nên góc xOz + góc zOy = góc xOy = 0,25 đ 900 Trang 47
Theo giả thiết có các tia phân giác nên góc xOz = 1 góc tOz 2 1 góc zOy = góc zOm 0,25 đ 2 Từ đó suy ra 1 1 góc tOz + góc zOm = 900 2 2
Hay góc tOz + góc zOm = 1800 0,5 đ
* Chứng minh góc tOz và góc zOm là hai góc kề nhau: 0,5 đ
* Kết luận : Cho 0,5 điểm 0,25 Câu b : 1,5 điểm
Chứng minh góc tOx = mOx’ = 300 ( Cùng kề bù với góc mOx) 0,5 đ Góc tOx = góc xOz = 300 0,5 đ Góc tOz = 600 0,5 đ Câu c : 1,0 điểm
Giả sử vẽ thêm n tia phân biệt gốc O không trùng với các tia
Ox,Oy,Oz,Ot,Om,Ox’. Tất cả trong hình vẽ có n+6 tia phân biệt .
Cứ 1 tia trong n+6 tia đó tạo với n+5 tia còn lại thành n+5 góc . 0,25 đ
Có n+6 tia nên tạo thành (n+5)(n+6) góc , nhưng như thế mỗi góc (n +5)(n + 6) 0,5 đ
được tính 2 lần . Vậy có tất cả là góc 2
Thay = 2014 ta được số góc có là 0,25 đ
(2014+6)(2014+5) : 2 = 2 039 190 góc Chú ý:
1. Thí sinh có thể làm bài bằng cách khác, nếu đúng vẫn được điểm tối đa.
2. Nếu thí sinh chứng minh bài hình mà không vẽ hình thì không chấm điểm bài hình. Trang 48
PHÒNG GIÁO DỤC VÀ ĐÀO TẠO
ĐỀ KHẢO SÁT HỌC SINH GIỎI LỚP 6 CẤP HUYỆN Môn: Toán – Lớp 6
Thời gian làm bài: 120 phút (Không kể thời gian giao đề) ĐỀ 13
Câu 1: (5 điểm)
a) Tìm x biết (x + 1) + (x + 2) + (x + 3) + ... + (x + 100) = 5750.
b) Tìm x; y  Z biết 2x + 124 = 5y .
c) Tìm kết quả của phép nhân A = 666    .. 6 . . 999    9 ... 10 c 0 / s c 100 / s
Câu 2 : (4 điểm) 10 2014 + 8 a) Chứng minh rằng : là một số tự nhiên. 72
b) Cho abc  7. Chứng tỏ rằng 2a + 3b + c  7
c) Cho các số tự nhiên từ 11 đến 21 được viết theo thứ tự tùy ý, sau đó đem cộng
mỗi số đó với số chỉ thứ tự của nó ta được một tổng . Chứng minh rằng trong các tổng
nhận được bao giờ cũng tìm ra hai tổng mà hiệu của chúng là một số chia hết cho 10. 5 5 5 5 5
Câu 3 : (2 điểm) Cho S = + + + + ...+
. Chứng minh rằng 3 < S < 8. 20 21 22 23 49 6 9
Câu 4 : (4 điểm) Tìm 3 số có tổng bằng 420, biết rằng số thứ nhất bằng số thứ hai 7 11
và bằng 2 số thứ ba. 3
Câu 5 : (5 điểm)
a) Cho góc xOy bằng 800, góc xOz bằng 300 . Tính số đo góc yOz ?
b) Cho 4 điểm A; B; C; D không nằm trên đường thẳng a. Chứng minh rằng đường
thẳng a hoặc không cắt, hoặc cắt ba, hoặc cắt bốn đoạn thẳng trong số các đoạn thẳng sau : AB; AC; BC; BD; CD; AD. Trang 49
PHÒNG GIÁO DỤC VÀ ĐÀO TẠO
ĐỀ KHẢO SÁT HỌC SINH GIỎI LỚP 6 CẤP HUYỆN Môn: Toán – Lớp 6
Thời gian làm bài: 120 phút (Không kể thời gian giao đề)
HƯỚNG DẪN CHẤM ĐỀ 13 Câu Nội dung Điểm Câu 1
a) Ta có (x + 1) + (x + 2) + (x + 3) + ... + (x + 100) = 5750.
(5 điểm) => 100x + 101.50 = 5750 0 , 7 5
=> 100x = 700 = > x = 7 0,75
b) +) x = 0 => 20 + 124 = 5y => 125 = 5y 0,5 => 53 = 5y => y = 3 0,25
+) x  0 => 2x + 124 là số chẵn => 2x + 124 = 5y là vô lý 0,5
Vậy x = 0 và y = 5 thì thỏa mãn đề bài. 0,25 c) A = 666    .. 6 . . 999    9 ... = A = 666    .. 6 . .(1    0 ... 00 - 1) 0,5 10 c 0 / s c 100 / s 10 c 0 / s c 100 / s = 666 0,5    .. 6 . 000    .. 0 . - 666    .. 6 . 10 c 0 / s 10 c 0 / s 10 c 0 / s = 666    .. 6 . 5 333    3 ... 4 c 99 / s c 99 / s 1,0 Câu 2
a) Chứng minh : 102014 + 8  8 0,25 (4 điểm) 102014 + 8  9 0,25
Mà (8; 9) = 1 => 102014 + 8  72 0,25 10 2014 + 8 0,25 => là một số tự nhiên. 72
b) abc  7 => 100a + 10b + c  7 => 98a + 7b + ( 2a + 3b + c)  7 0,25
=> 7(14a + b) + ( 2a + 3b + c)  7 0,25
Mà 7(14a + b)  7 => ( 2a + 3b + c)  7 0,5
c) Khi xét 1 số tự nhiên khi chia cho 10
=> Có thể xảy ra 10 trường hợp về số dư   ; 2 ; 1 ; 0 ...;  9 (1) 0,5
Mà các số tự nhiên từ 11 --> 21 gồm (21 - ) + 1 = 11 số.
Biết mỗi số cộng với đúng số thứ tự của nó được 1 tổng
=> Có 11 tổng , mỗi tổng đều có giá trị là 1 số tự nhiên (2) 0,5
Từ (1) và (2) => Trong 11 tổng trên chắc chắn có 2tổng có
cùng số dư khi chia cho 11 0,5 0,5 Trang 50
=> Luôn  hai tổng có hiệu chia hết cho 10. Câu 3 Xét tổng S = 5 5 5 5 5 + + + + 0,25 ... + có 30 số hạng (2 điểm) 20 21 22 23 49 5 5 5 5 5 5 5 5 Mà  ;  ;  ;...;  0,5 20 50 21 50 22 50 49 50 5 0,25 => S  30. = S  3 (1) 50 Lại có : 5 5 5 5 5 5 5 5 = ;  ;  ;...;  0,5 20 20 21 20 22 20 49 20 5 150 => S < 30. = => S < 8 (2) 20 20 0,25
Từ (1) và (2) => 3 < S < 8. 0,25 Câu 4
Lập luận => Số thứ nhât bằng 21 số thứ hai. 0,75 (4 điểm) 22 27 Số thứ ba bằng số thứ hai. 0,75 22
=> Tổng của ba số bằng 22 + 21 + 27 70 = số thứ hai 0,75 22 22
=> Số thứ hai là : 420 : 70 = 132 0,75 22
=> Số thứ nhất là : 21 1 . 32 = 126 0,5 22 27 => Số thứ nhất là : 1 . 32 = 162 0,5 22 Câu 5
a) +) TH1: Hai tia Oy và Oz nằm trên hai nửa mp đối nhau bờ (5 điểm) chứa tia Ox :
Lập luận => Tia Ox nằm giữa hai tia Oy và Oz Trang 51 y 0,5 0,5 x O z
=> góc yOz = 800 + 300 = 1100
+) TH2: Hai tia Oy và Oz cùng nằm trên một nửa mp bờ chứa tia Ox y 0,5 z 0,5 0,5 x O
Lập luận => Tia Oz nằm giữa hai tia Ox và Oy . 0,5
=> góc yOz = 800 - 300 = 500
b) +) TH1: Bốn điểm A; B; C; D cùng thuộc một nửa mp bờ là a. 0,5
=> Đường thẳng a không cắt đoạn thẳng nào trong
các đoạn thẳng AB; AC; AD; BC; BD; CD 0,5
+) TH2: Trong hai nửa mp đói nhau bờ a, mỗi nửa mp chứa 2
trong bốn điểm A; B; C; D 0,5
=> Đường thẳng a cắt 4 đoạn thẳng trong số 6 đoạn
thẳng AB; AC; AD; BC; BD; CD. 0,5
+) TH2: Trong hai nửa mp đối nhau bờ a, một nửa mp chứa 1
điểm, nửa mp còn lại chứa 3 trong số bốn điểm A; B; C; D
=> Đường thẳng a cắt 3 đoạn thẳng trong số 6 đoạn
thẳng AB; AC; AD; BC; BD; CD.
Suy ra điều phải chứng minh . Trang 52 Chú ý:
1. Thí sinh có thể làm bài bằng cách khác, nếu đúng vẫn được điểm tối đa.
2. Nếu thí sinh chứng minh bài hình mà không vẽ hình thì không chấm điểm bài hình.
PHÒNG GIÁO DỤC VÀ ĐÀO TẠO
ĐỀ KHẢO SÁT HỌC SINH GIỎI LỚP 6 CẤP HUYỆN Môn: Toán – Lớp 6
Thời gian làm bài: 120 phút (Không kể thời gian giao đề) ĐỀ 14 Câu 1: ( 4 điểm)
1) Chứng minh rằng số A = 10n + 18n - 1 chia hết cho 27 ( n là số tự nhiên)
2) Chứng tỏ rằng với mọi số tự nhiên n phân số sau tối giản: 16n + 3 12n + 2 Câu 2: (5 điểm)
1) Tìm các số nguyên x, y sao cho: (x - 1)(3 - y) = 2
2) Tìm tập hợp số nguyên x , biết : 3 6 1 2 1 3 2 3
(1 − ) : (1 + 2 + 20%)  x  1 .1 + 3 : 2 4 4 5 5 5 4 11 21
3) Tìm số tự nhiên x biết: 1 1 1 2 2013 + + + ...+ = 3 6 10 x(x + 1) 2015 Câu3:(2điểm) Chứng minh rằng : 1 + 1 1 1 + +...+ 1000 1999 2 3 2 Câu 4: (4 điểm)
Sau buổi biểu diễn văn nghệ, nhà trường tặng cam cho các tiết mục. Lần đầu tiết mục
đồng ca hết 5 số cam và 1 quả; lần 2 tặng tiết mục tốp ca hết 6 số cam còn lại và 1 quả; 6 6 7 7
lần 3 tặng tiết mục đơn ca hết 3 số cam còn lại lần 2 và 1 quả thì vừa hết. Tính số cam 4 4
trường đó đã tặng và số cam riêng cho các tiết mục đồng ca, tốp ca và đơn ca. Câu 5: ( 5 điểm)
Cho tia Ox. Trên hai nửa mặt phẳng đối nhau có bờ là Ox. Vẽ hai tia Oy và Oz sao cho
góc xOy và xOz bằng 1200. Chứng minh rằng:
a. xOy = xOz = yOz
b. Tia đối của mỗi tia Ox, Oy, Oz là phân giác của góc hợp bởi hai tia còn lại.
..........................................HẾT........................................... Trang 53
PHÒNG GIÁO DỤC VÀ ĐÀO TẠO
ĐỀ KHẢO SÁT HỌC SINH GIỎI LỚP 6 CẤP HUYỆN Môn: Toán – Lớp 6
Thời gian làm bài: 120 phút (Không kể thời gian giao đề)
HƯỚNG DẪN CHẤM ĐỀ 14 Câu Nội dung Điểm Câu 1
1) Ta viết số A dưới dạng sau : (4điểm) 0,5đ
A = 10n + 18n - 1 = 10n - 1 - 9n + 27n 0,5đ
= 99...9 - 9n + 27n = 9( 11...1 - n) + 27n n n
n là tổng các chữ số của 11...1 nên ( 11...1 - n) 3 n n 0,5đ Từ đó  A 27 0,5đ
2) Gọi ƯCLN của tử và mẫu là d (d  N, d  1) thì 16n + 3 d và 0,5đ 12n + 2 d
 3(16n + 3) d và 4(12n + 2) d Do đó 3(16n + 3) 0,5đ - 4(12n + 2) d  48n + 9 - 48n - 8 d 0,5đ  1 d Vậy d = 1  n + Phân số 16
3 là phân số tối giản. 0,5đ 12n + 2 Câu 2 1) Ta có: (x - 1)(3 - y) = 2 (5điểm)
Vì 4 có các ước là - 2; -1; 1; 2 và ;x yZ nên (x - 4), (3 - y) Z .Do đó ta có: . x −1 = 1 − x = 1 − +1 x = 0 +      . 0,5đ 3  − y = −2 y = 3 + 2 y = 5 x −1 = −2 x = −2 +1 x = 1 − +      3  − y = −1  y = 3 +1 y = 4 x −1 = 2 x = 2 +1 x = 3 +      3 − y = 1  y = 3 −1 y = 2 0,5đ  x −1 = 1 x = 1+1 x = 2 +      3 − y = 2  y = 3 − 2  y = 1
Vậy các cặp số nguyên x; y thỏa mãn là: 0,5đ (x; y) = (0;5);( 1 − ;4);(3;2);(2;1  ) Trang 54 3 6 1 2 1 3 2 3
2) (1 − ) : (1 + 2 + 20%)  x  1 .1 + 3 : 2 4 4 5 5 5 4 11 21 0,5đ 7 6 1 2 1 12 7 32 245
 ( − ) : (1 + 2 + )  x  . + : 4 4 5 5 5 7 4 11 121 1đ 1 4  35 121 : 3  x  3+ 5 32 .   x
mà x  Z nên x 1;2;3;  4 4 5 11 245 76 7 0,5đ 1
3. Nhân vào hai vế ta được : 1 1 1 1 1 2013 + + + ...+ = . 2 6 12 20 x(x +1) 2 2015 1 1 1 1 1 1 1 1 1 2013 0,5đ
 − + − + − + ...+ − = . 2 3 3 4 4 5 x x +1 2 2015  1 1 1 2013 − = . 2 x +1 2 2015 x −1 1 2013  = . 0,5đ 2(x +1) 2 2015  x −1 2013 = x +1 2015 0,5đ
 2015x − 2015 = 2013x + 2013
 2x = 4028  x = 2014 Câu 3 (2điểm) 1 1 1 1 1 1 1 1 1 1 1+ + +...+ = 1 + + ( + ) + ( + ... + ) + ( + ...+ ) + 1999 2 3 2 2 2 3 2 3 5 2 4 9 2 …+( 1 1 + 0,5đ ... + ) 1998 1999 2 +1 2 1 1 1 1 1 > 1 + 2 3 1998 + 1đ .2 + .2 + .2 + ... + .2 2 3 4 1999 2 2 2 2 2 1
= 1+ . 1999 = 1000,5 > 1000 ( ĐPCM) 0,5đ 2 1 - Nhận xét :
quả cuối cùng chính là 4 3 1 − = số cam còn lại sau Câu 4 4 4 4 4
(4điểm) lần 3. Vậy bài này phải tính ngược từ dưới lên. 1đ
Tiết mục đơn ca được tặng 1 1 1 4 : = . = 1 (quả). 4 4 4 1 0,75đ
Tương tự trên, tiết mục đơn ca và tốp ca được tặng : 1 1 ( +1) : = 8 (quả). 0,75đ 7 7
Tương tự số cam của trường đó đã tặng : 1 1 (8 + ) : = 49 (quả) 0,5đ 6 6
Số cam tặng tiết mục tốp ca : 8 - 1 = 7 (quả) 0,5đ
Số cam tặng tiết mục đồng ca : 49 - 8 = 41 (quả). 0,5đ Câu 5 Trang 55 (5điểm) z, y x, O x 0,5đ z y,
a) Kẻ tia Ox, là tia đối của tia Ox Ta có: ,
x Oy + yOx = 1800 (kề bù) 0,5đ '
x Oy = 1800 - 1200 = 600 0,5đ Tương tự: ' 0 x Oz = 60 0,5đ Ta có : ' 0 ' 0
x Oy = 60 , x Oz = 60 và tia Ox’ nằm giữa hai tia Oy, Oz nên ' ' 0
yOz = yOx + x Oz = 120 vậy xOy = yOz = zOx
b) Do tia Ox’ nằm giữa hai tia Oy, Oz và ' ' x Oy = x Oz 0,5đ
nên Ox’ là tia phân giác của góc hợp bởi hai tia Oy, Oz. 0,5đ
-Tương tự tia Oy’ (tia đối của Oy) và tia Oz’ (tia đối của tia Oz) là 1đ
phân giác của góc xOz và xOy. Chú ý:
1. Thí sinh có thể làm bài bằng cách khác, nếu đúng vẫn được điểm tối đa.
2. Nếu thí sinh chứng minh bài hình mà không vẽ hình thì không chấm điểm bài hình.
PHÒNG GIÁO DỤC VÀ ĐÀO TẠO
ĐỀ KHẢO SÁT HỌC SINH GIỎI LỚP 6 CẤP HUYỆN Môn: Toán – Lớp 6
Thời gian làm bài: 120 phút (Không kể thời gian giao đề) ĐỀ 15 Bài 1( 4 điểm)
a) Cho A = 5 - 52 + 53 - 54 + …- 598 + 599 . Tính tổng A.
b) Chứng tỏ ( 2n + 1).( 2n + 2) chia hết cho 3 với mọi n là số tự nhiên. Bài 2 ( 5 điểm)
a) Tìm các số nguyên x, y biết rằng : (x - 2)2.(y - 3) = - 4
b) Tìm n ∈ Z để (4n - 3) (3n – 2) Bài 3 ( 2 điểm) 1 1 1 1 1 1 3 Chứng minh A = + + + +...+ + 1 2 2 2 2 2 2 1 2 3 4 99 100 4 Bài 4 ( 4 điểm) Trang 56
Trong một buổi đi tham quan, số nữ đăng kí tham gia bằng 1 số nam. Nhưng sau đó có 4
một bạn nữ xin nghỉ, một bạn nam xin đi thêm nên số nữ đi tham quan bằng 1 số nam. 5
Tính số học sinh nữ và nam đã đi tham quan. Bài 5: (5 điểm) 1
Cho 4 tia chung gốc theo thứ tự Ox, Oy, Oz, Ot sao cho xOy = zOt ; 2 1 yOz = x
Oy , biết số đo góc zOt bằng 600. 2
a) Tính số đo các góc xOy; yOz; tOx?
b) Vẽ tia Om sao cho số đo góc mOt bằng 200 . Tính số đo góc zOm?
c) Vẽ thêm 10 tia phân biệt chung gốc với các tia Ox, Oy, Oz, Ot, Om. Hỏi có bao
nhiêu góc tạo thành từ tất cả các tia trên?
-----------------------Hết-----------------------
PHÒNG GIÁO DỤC VÀ ĐÀO TẠO
ĐỀ KHẢO SÁT HỌC SINH GIỎI LỚP 6 CẤP HUYỆN Môn: Toán – Lớp 6
Thời gian làm bài: 120 phút (Không kể thời gian giao đề)
HƯỚNG DẪN CHẤM ĐỀ 15 Bài Nội dung Điểm Câu a( 2 điểm)
A = 5 – 52 + 53 – 54 + …- 598 + 599 0,5 đ Bài 1
 5A = 52 – 53 + 54 - …+ 598 – 599 + 5100 1 đ
(4điểm) Tính và rút gọn được 6A = 5 + 5100 100 0,5 đ  5 + 5 A = 6 Câu b ( 2 điểm)
Ta có: 2n . ( 2n +1).( 2n + 2) là tích của 3 số tự nhiên liên tiếp nên 0,75 đ chia hết cho 3. 0,5 đ
Mà 2n không chia hết cho 3 0,5đ
nên ( 2n + 1).( 2n + 2) 3  n N 0,25đ Câu a( 3 điểm)
Ta có : -4 = 12.(-4) = 22.(-1) nên ta có các trường hợp sau: 0,5 đ Bài 2
TH1: ( x - 2)2 = 12 và y - 3 = -4 0,5 đ
(5điểm)  x - 2 = 1 ; y = -1 hoặc x - 2 = -1; y = -1
 x = 3; y = -1 hoặc x = 1; y = -1 0,5 đ
TH2: ( x - 2)2 = 22 và y - 3 = -1 0,5 đ Trang 57
 x - 2 = 2 ; y = 2 hoặc x - 2 = -2; y = 2 0,5 đ
 x = 4; y = 2 hoặc x = 0; y = 2 0,5 đ
KL: Vậy ta có các cặp (x, y) nguyên thỏa mãn là:
(3; -1); (1; -1); (4; 2); (0; 2) Câu b( 2 điểm) Ta có: 4n – 3 + 3n – 2 Mà 3n + 2 + 3n + 2 0,5đ
→ 3(4n – 3) – 4(3n – 2) + 3n – 2
→ ( 12n - 9 - 12n + 8 ) + 3n – 2 → - 1 + 3n – 2 0,75đ → 3n – 2  Ư(-1) 0,25đ +) 3n – 2 = 1 suy ra n = 1 0,25đ
+) 3n – 2 = -1 suy ra n = 1/3 0,25đ
Kết hợp điều kiện n nguyên ta được n = 1 1 1 + Giữ nguyên 2 2 1 2 1 1 1 1 1 1 Bài 3 Ta có:  ;  ;...;  0,25 đ 2 2 2 3 2.3 4 3.4 100 99.100 (2điểm) 1 1 1 1 1 1  A = + + + +...+ + 2 2 2 2 2 2 1 2 3 4 99 100 1 1 1 1 1 1 1  ( + ) + ( + + +...+ ) = 1+ + A' 2 2 1 2 2.3 3.4 4.5 99.100 4 0,5 đ 1 1 1 1 (A' = + + +...+ ) 2.3 3.4 4.5 99.100 Chứng minh A’ = 1 1 − 0,5 đ 2 100 Do đó 1 1 1 3 1 3 A  1+ + − =1+ − 1 0,75 đ 4 2 100 4 100 4 Bài 4
Tổng số học sinh nam và nữ dự định đi tham quan và đã đi tham
(4điểm) quan là như nhau, ta lấy làm đơn vị. 0,5 đ
Số nữ dự định đi tham quan bằng 1 số nam nên bằng 1 tổng số 4 5 0,5đ nam và nữ.
Số nữ đi tham quan bằng 1 số nam nên bằng 1 tổng số nam và nữ. 0,5đ 5 6
Số nữ dự định đi nhiều hơn số nữ đã đi là: 1 1 1 − = ( tổng số học 0,5đ 5 6 30 1 sinh) hay
tổng số học sinh tương ứng với 1 học sinh. 30 0,5đ
 Tổng số học sinh là: 1 : 1 = 30 ( học sinh) 30 0,5đ Trang 58
Số học sinh nữ đã đi tham quan là: 30 . 1 = 5 (học sinh) 6 0,5đ
Số học sinh nam đã đi tham quan là: 30 – 5 = 25 ( học sinh) 0,5đ
Vậy có 5 học sinh nữ và 25 học sinh nam đi tham quan. Bài 5
Vẽ hình đúng câu a, b được 0,5 điểm ( hs không vẽ được hình không
(5điểm) tính điểm bài làm) x y z 0 60 0,5 đ O t Câu a ( 1 điểm) 1 1 xOy = zOt; yOz = xOy Vì 2 2 0 Mà zOt = 60 nên 1 1 1 0 0 0 0 x
Oy = 60 = 30 ; yOz = xOy = 30 =15 2 2 2 0,5 đ 0 0 0 0 Tính được xOt = xOy + zOy + z
Ot = 30 +15 + 60 =105 0,5 đ Câu b ( 2,5 điểm) Ta có 2 trường hợp:
TH1: Tia Om nằm giữa tia Oz và tia Ot 1,25 đ Trang 59 x y z m 0 20 O 1,25 đ t Tính được 0 0 0 zOm = zOt t
Om = 60 − 20 = 40
TH2: Tia Ot nằm giữa 2 tia Om và Oz x y z O 0 20 t 0,5 đ m Tính được 0 0 0 zOm = mOt + tOz = 20 + 60 = 80 Câu c ( 1 điểm) 0,5 đ
Từ hai tia chung gốc ta vẽ được 1 góc.
Vẽ thêm 10 tia phân biệt gốc O không trùng với các tia Ox, Oy, Oz,
Ot, Om. Tất cả trong hình vẽ có 15 tia phân biệt .
Cứ 1 tia trong 15 tia đó tạo với 14 tia còn lại thành 14 góc .
Có 15 tia nên tạo thành 15.14 ( góc)
nhưng như thế mỗi góc được tính 2 lần .
nên có tất cả số góc tạo thành từ 15 tia phân biệt chung gốc là : Trang 60 15.14 =105 góc 2 Chú ý:
1. Thí sinh có thể làm bài bằng cách khác, nếu đúng vẫn được điểm tối đa.
2. Nếu thí sinh chứng minh bài hình mà không vẽ hình thì không chấm điểm bài hình.
PHÒNG GIÁO DỤC VÀ ĐÀO TẠO
ĐỀ KHẢO SÁT HỌC SINH GIỎI LỚP 6 CẤP HUYỆN Môn: Toán – Lớp 6
Thời gian làm bài: 120 phút (Không kể thời gian giao đề) ĐỀ 16 Bài 1: (4,0 điểm ) ,
1. Chứng tỏ rằng: 2x + 3y chia hết cho 17  9x + 5y chia hết cho 17.
2. Cho C = 3 + 32 + 33 + 34 ………+ 3100 chứng tỏ C chia hết cho 40.
3. Tìm các nguyên tố x, y thỏa mãn : (x-2)2 .(y-3) = - 4 Bài 2 :(5,0đ) Tìm x, biết: 1. a) 32x = 81 ; b) 52x-3 – 2.52 = 52.3 15 9 20 9 5.4 .9 − 4.3 .8 2. Tính 9 19 29 6 5.2 .6 − 7.2 .27 3. Tính tổng: B = 2 2 2 2 + + +....+ 4 . 1 7 . 4 10 . 7 97 1 . 00
4. Tìm số tự nhiên n để phân số 8n +193 A =
Có giá trị là số tự nhiên. 4n + 3 1 1 1 1
Bài 3: (2,0đ) Chứng minh rằng : + + ++ 1 22 32 42 100 2 Bài 4: ( 4,0 điểm)
Tổng số trang của 8 quyển vở loại 1 ; 9 quyển vở loại 2 và 5 quyển vở loại 3 là 1980
trang. Số trang của một quyển vở loại 2 chỉ bằng 2 số trang của 1 quyển vở loại 1. Số 3
trang của 4 quyển vở loại 3 bằng số trang của 3 quyển vở loại 2. Tính số trang của mỗi quyển vở mỗi loại.
Bài 5: (5,0đ) Cho tam giác ABC và BC = 5cm. Điểm M thuộc tia đối của tia CB sao cho CM = 3 cm. a. Tình độ dài BM
b. Cho biết góc BAM = 800 , góc BAC = 600 . Tính góc CAM.
c. Vẽ các tia Ax, Ay lần lượt là tia phân giác của góc BAC và CAM . Tính góc xAy.
d. Lấy K thuộc đoạn thẳng BM và CK = 1 cm. Tính độ dài BK. Trang 61
PHÒNG GIÁO DỤC VÀ ĐÀO TẠO
ĐỀ KHẢO SÁT HỌC SINH GIỎI LỚP 6 CẤP HUYỆN Môn: Toán – Lớp 6
Thời gian làm bài: 120 phút (Không kể thời gian giao đề)
HƯỚNG DẪN CHẤM ĐỀ 16 Bài Nội dung Điểm Bài 1
1. Ta có 4 (2x + 3y ) + ( 9x + 5y ) = 17x + 17y chia hết cho 17
(4điểm) Do vậy ; 2x + 3y chia hết cho 17  4 ( 2x +3y ) chia hết cho 17  9x + 5y chia hết cho 17 1,0đ.
Ngược lại Ta có 4 ( 2x + 3y ) chia hết cho 17 mà ( 4 ; 17 ) = 1  2x + 3y chia hết cho 17
2. B = (3 + 32 + 33+ 34) +……+ (397+398+399+3100)
= 3 (1 + 3 + 32+33)+…….+ 397(1+3+32+33)
= 40. (3 + 35 +39 +………+397 ) : 40 0,5đ.
3. Do –4 = 12 . (- 4) = 22.(-1) nên có các trường hợp sau:  0,5đ (x − ) 2 2 = 1 x − 2 = 1 x = 3 a.      y − 3 = −4 y = −1 y = −1 x − 2 = −1 x = 1 hoặc    y = −1 y = −1 1,0đ. (x − ) 2 2 = 22 x − 2 = 2 x = 4 b.      y − 3 = −1 y = 2 y = 2 x − 2 = −2 x = 0 1,0đ hoặc    y = 2 y = 2 Bài 2
1 .a) 32x = 81 => 32x = 34 => 2x = 4 => x = 2
(5điểm) b). 52x-3 – 2.52 = 52.352x: 53 = 52.3 + 2.5252x: 53 = 52.5 0,5đ. 52x = 52.5.53 52x = 56 0,5đ. => 2x = 6 => x = 3 0,5đ. 30 18 2 20 27 29 18 5.2 .3 − 2 .3 .2 2 .3 (5.2 − 3) 2 . = = 2 1,0đ. 9 19 19 29 18 28 18 5 .2 .2 .3 − 7.2 .3 2 .3 (5.3 − 7.2) 1 1 1 1 2 2 1 1 3. Ta có = ( − )  = ( − ) 4 . 1 3 1 4 4 . 1 3 1 4 0,5đ.  2 2 1 1 2 2 1 1 2 2 1 1 = ( − ); = ( − );.... ......; = ( − ) 7 . 4 3 4 7 10 . 7 3 7 10 97 100 . 3 99 100 0,5đ.  2 1 1 1 1 1 1 1 1 B= ( − + − + − +.....+ − ) 3 1 4 4 7 7 10 99 100  2 1 1 2 99 33 B= ( − ) = . = 0,5đ. 3 1 100 3 100 50 8n +193 ( 2 4n + ) 3 +187 187 4. A = = = 2 + 0,5đ. 4n + 3 4n + 3 4n + 3
Để A N thì 187  4n + 3 => 4n +3    187 ; 11 ; 17 + 4n + 3 = 11 -> n = 2 + 4n +3 = 187 --> n = 46 0,5đ.
+ 4n + 3 = 17 -> 4n = 14 -> không có n N Trang 62 Vậy n = 2; 46 Bài 3 Ta có: (2điểm) 1 1 1 1 1 1 1 1  = − ;  = − ; 0,5đ. 22 2 . 1 1 2 32 3 . 2 2 3 1 1 1 1 1 1 1 1  = − ;...;  = − 42 4 . 3 3 4 100 2 99 1 . 00 99 100; 0,5đ. Vậy 1 + 1 + 1 + 1 1 1 1 + 1  + + ++ = 0,5đ. 2 2 2 2 2 3 4 10 0 2 . 1 3 . 2 4 . 3 99 100 . 1 1 1 1 1 1 1 = 1 99 1− + − + − + + − = 1− = 1. 0,5đ. 2 2 3 3 4 99 100 2 100 2
Vì số trang của mỗi quyển vỡ loại 2 bằng số trang của 1 quyển Bài 4 3 0,5đ.
(4điểm) loại 1. Nên số trang của 3 quyển loại 2 bằng số trang của 2 quyển loại 1
Mà số trang của 4 quyển loại 3 bằng 3 quyển loại 2. 1,0đ.
Nê số trang của 2 quyển loại 1 bằng số trang của 4 quyển loại 3
Do đó số trang của 8 quyển loại 1 bằng :4 .8 : 2 = 16 ( quyển loại 3)
Số trang của 9 quyển loại 2 bằng9 .4 : 3 = 12 (quỷên loại 3) 1,0đ.
Vậy 1980 chính là số trang của 16 + 12+ 5 = 33(quyển loại 3)
Suy ra: Số trang 1 quyển vở loại 3 là 1980 : 33 = 60 ( trang) 60 4 .
Số trang 1 quyển vở loại 2 là = 80 (trang) 0,5đ. 3 80 3 . 1,0đ.
Số trang 1 quyển vở loại1 là; = 120 ( trang) 2 Bài 5 A (5điểm) 0,75đ B M x C y
a. M, B thuộc 2 tia đối nhau CB và CM
-> C nằm giữa B và M. -> BM = BC + CM = 8 (cm) 0,5đ.
b. C nằm giữa B,M -> Tia AC nằm giữa tia AB, AM
->  CAM =  BAM -  BAC = 200 1 1 0,75đ
c. Có  xAy =  x AC +  CAy =  BAC +  CAM 2 2 1 1 1
= (  BAC +  CAM) =  BAM = .80 = 400 2 2 2 1,0đ
d. +) Nếu K  tia CM -> C nằm giữa B và K1 -> BK1 = BC + CK1 = 6 (cm) 1 , 0đ
+)Nếu K tia CB -> K2 nằm giữa B và C -> BK2 = BC = CK2 =4 (cm) 1,0đ Trang 63 Chú ý:
1. Thí sinh có thể làm bài bằng cách khác, nếu đúng vẫn được điểm tối đa.
2. Nếu thí sinh chứng minh bài hình mà không vẽ hình thì không chấm điểm bài hình.
3. Chấm và cho điểm từng phần, điểm của toàn bài là tổng các điểm thành phần không làm tròn.
HÒNG GIÁO DỤC VÀ ĐÀO TẠO
ĐỀ KHẢO SÁT HỌC SINH GIỎI LỚP 6 CẤP HUYỆN Môn: Toán – Lớp 6
Thời gian làm bài: 120 phút (Không kể thời gian giao đề) ĐỀ 17
Bài 1: (4 điểm): − Cho biểu thức A = 5 n − 2
a, Tìm các số nguyên n để biểu thức A là phân số.
b, Tìm các số tự nhiên n để biểu thức A là số nguyên Bài 2: (5 điểm):
1. Tính nhanh A= 3.136.8 + 4. 14.6 -14.150 11 5 − 4 4  8 B =  − :     4 9 9 11 33 2. Tìm x biết
a/ 3 + 2x -1 = 24 – [42 – (22 - 1)]
b/ x − 5 = 18 + 2.(-8) Bài 3: (5 điểm)
1
.Cho đoạn thẳng AB, điểm O thuộc tia đối của tia AB. Gọi M,N thứ tự là trung điểm của OA, OB.
d) Chứng tỏ rằng OA < OB.
e) Trong ba điểm O, M, N điểm nào nằm giữa hai điểm còn lại ?
2. Cho góc AOB và góc BOC là hai góc kề bù . Biết góc BOC bằng năm lần góc AOB. a) Tính số đo AOB,BOC .
b) Gọi OD là tia phân giác của góc BOC. Tính số đo góc AOD. Bài 4. (4 điểm):
Bạn An nghĩ ra một số có 3 chữ số, nếu bớt số đó đi 8 đơn vị thì được một số chia hết cho 7, nếu
bớt số đó đi 9 đơn vị thì được một số chia hết cho 8, nếu bớt số đó đi 10 đơn vị thì được 1 số chia
hết cho 9. Hỏi bạn An nghĩ ra số nào? Bài 5. (2 điểm) : 1 1 1 1 1 1 Chứng minh rằng + + + + ... + +  1 2 2 2 2 2 2 2 3 4 5 2011 2012
------------------------------- Hết -------------------------------- Trang 64
PHÒNG GIÁO DỤC VÀ ĐÀO TẠO
ĐỀ KHẢO SÁT HỌC SINH GIỎI LỚP 6 CẤP HUYỆN Môn: Toán – Lớp 6
Thời gian làm bài: 120 phút (Không kể thời gian giao đề)
HƯỚNG DẪN CHẤM ĐỀ 17 BÀI ĐÁP ÁN ĐIỂM a/ n Z và n  2 2  1  ;  5 1 b/ (n - 2 ) Ư( -5) = n − 2 = 1 −  n =1 N Bài 1   (4,0điểm) n − 2 = 1 n = 3 N     n − 2 = 5 − n = 3 −  N 0,5    n − 2 = 5  n = 7 N Vậy n = 1;3;7 0,5 1
A= 24.136 + 24.14 - 14.150 = 24.(136 + 14)- 14.150 0,5
= 24.150 - 14.150= 150. (24- 14)=150.10 =150 1 11 5 − 4 11  8 11  5 − 4  8 11 8 2 B =  −   =  −  = 1 =      4 9 9 4  33 4  9 9  33 4 33 3 1 Bài 2 2
a) 3 + 2x-1 = 24 – [42 – (22 - 1)] (5 điểm) 3 + 2x-1 = 24 – 42 + 3 0,5 2x-1 = 24 – 42 = 22 0,5 x -1 = 2 x = 3 0,5 b/ x = 7 hoặc x = 3; 1 0,5 o m a n b 1.( 2điểm)
Hai tia AO, AB đối nhau, nên điểm A nằm giữa hai đ O và B  OA< OB. 0,5
Ta có M và N thứ tự là trung điểm của OA, OB, nên : OA OB  OM = ; ON = 2 2
Vì OA < OB, nên OM < ON. 0,5
Hai điểm M và N thuộc tia OB, mà OM < ON nên điểm M nằm giữa hai 0,5 điểm O và N. 2.(3 điểm) B D Bài 3 Vẽ hình đúng 0,5 (5đ) A O C a)Vì 0,5
AOB và BOC là hai góc kề bù nên: 0 AOB + BOC = 180 mà
BOC = 5.AOB nên: 6 AOB = 1800 0,5
Do đó: AOB = 1800 : 6 = 300 ; BOC = 5. 300 = 1500
b)Vì OD là tia phân giác của 1 BOC nên BOD = DOC = BOC =750. 0,5 2
Vì góc AOD và góc DOC là hai góc kề bù nên: 0 AOD + DOC = 180 0,5
Do đó AOD =1800 – DOC = 1800- 750 = 1050 0,25 KL: 0,25 Trang 65 Bài 4
Gọi số bạn An nghĩ ra là A
(4,0điểm) Vì (A-8) 7  (A-1) - 7 7 (A-1) 7 1
Vì (A-9) 8  (A-1) - 8 8 (A-1) 8 0,5
Vì (A-10) 9  (A-1) - 9 9 (A-1) 9 0,5
Do đó: (A-1) là bội chung của 7,8,9 và A là số có 3 chữ số nên 1 99 < A < 1000 0,5
Từ đó giải và tìm được A-1 = 504Suy ra :A= 505 0,5 1 1 1 1 1 1 1 1 Ta có  ;  ;  ; … ;  0,5 2 2 1.2 2 3 2.3 2 4 3.4 2 2012 2011.2012 1 1 1 1 1 1 1 1 1 + + + 0,5 ... + +  + + +...+ 2 2 2 2 2 Bài 5 2 3 4 2011 2012 1.2 2.3 3.4 2011.2012 (2,0đ) 1 1 1 1 1 1 1 1 1 1 1 1 1 + + + 0,5 ... + +  − + − + − +...+ − 2 2 2 2 2 2 3 4 2011 2012 1 2 2 3 3 4 2011 2012 1 1 1 1 1 1 1 + + + ... + +  − 2011 = < 1 0,5 2 2 2 2 2 2 3 4 2011 2012 1 2012 2012 Chú ý:
1. Thí sinh có thể làm bài bằng cách khác, nếu đúng vẫn được điểm tối đa.
2. Nếu thí sinh chứng minh bài hình mà không vẽ hình thì không chấm điểm bài hình.
PHÒNG GIÁO DỤC VÀ ĐÀO TẠO
ĐỀ KHẢO SÁT HỌC SINH GIỎI LỚP 6 CẤP HUYỆN Môn: Toán – Lớp 6
Thời gian làm bài: 120 phút (Không kể thời gian giao đề) ĐỀ 18
Bài 1: ( 2.5 điểm)

a. Cho ababab là số có sáu chữ số. Chứng tỏ số ababab là bội của 3.
b. Cho S = 5 + 52 + 53 + 54 + 55 + 56 …+ 52004. Chứng minh S chia hết cho 126 và chia hết cho 65. Bài 2 : (2,0 điểm)
Tìm số tự nhiên x biết : a. x + (x + 1) + (x + 2) ++ (x + 2 010 ) = 2 029099 b. 2 + 4 + 6 + 8 + + 2x = 210 Câu 3: (2.0 điểm)
a. Tìm tất cả các số nguyên tố p sao cho p + 11 cũng là số nguyên tố.
b. Tìm tất cả các số nguyên tố p để p + 8, p + 10 cũng là các số nguyên tố.
câu 4 : ( 1.5 điểm)
một phép chia có thương bằng 5 và số dư là 12. nếu lấy số bị chia chia cho tổng số chia và số dư
ta được thương là 3 và số dư là 18. tìm số bị chia. Câu 5: (2.0 điểm)
Trên đoạn thẳng AB = 3 cm lấy điểm M. Trên tia đối của tia AB lấy điểm N sao cho AM = AN.
a. Tính độ dài đoạn thẳng BN khi BM = 1 cm.
b. Hãy xác định vị trí của M (trên đoạn thẳng AB) để BN có độ dài lớn nhất. Trang 66
PHÒNG GIÁO DỤC VÀ ĐÀO TẠO
ĐỀ KHẢO SÁT HỌC SINH GIỎI LỚP 6 CẤP HUYỆN Môn: Toán – Lớp 6
Thời gian làm bài: 120 phút (Không kể thời gian giao đề)
HƯỚNG DẪN CHẤM ĐỀ 18
Bài 1: ( 2.5 điểm)
- ababab= ab .10000 + ab .100 + ab = 10101 ab . 0,50
- Do 10101 chia hết cho 3 nên ababab chia hết cho 3 hay ababab là bội 0,50 của 3.
Có: 5 + 52 + 53 + 54 + 55 + 56 = 5(1 + 53) + 52(1 + 53) + 53(1 + 53) = 5. 126 + 52.126 + 53.126 0,50
 5 + 52 + 53 + 54 + 55 + 56 chia hết cho 126.
S = (5 + 52 + 53 + 54 + 55 + 56) + 56(5 + 52 + 53 + 54 + 55 + 56) + … + 51998(5 + 52 + 53 + 54 + 55 + 56). 0,25
Tổng trên có (2004: 6 =) 334 số hạng chia hết cho 126 nên nó chia hết cho 126.
Có: 5 + 52 + 53 + 54 = 5+ 53 + 5(5 + 53) = 130 + 5. 130.  0,25
5 + 52 + 53 + 54 chia hết cho 130 .
S = 5 + 52 + 53 + 54 + 54 (5 + 52 + 53 + 54 ) + … + 52000(5 + 52 + 53 + 54 )
Tổng trên có (2004: 4 =) 501 số hạng chia hết cho 130 nên nó chia hết cho 130. 0,25
Có S chia hết cho 130 nên chia hết cho 65. 0,25
Bài 2 : (2,0 điểm)
-  2011x +1 + 2 + + 2010 = 2029099 0,25 2010 2 . 011 -  2011x + = 2029099 0,25 2 2010 2 . 011 -  2011x = 2029099 - 0,25 2  2010 20 . 11 -  x = 2029099-  : 2011= 4 0,25  2  -  2( 1+ 2 + 3 + + x )= 210 0,25 x(x + ) 1 -  2 = 210 0,25 2 -  ( x x + ) 1 = 210 0,25
- Giải được x = 14 (Do 210 = 2.3.5.7 = 14.15) 0,25 Câu 3: (2.0 điểm)
a) - Nếu p lẻ  p + 11 là số chẵn lớn hơn 11 nên không là số nguyên tố. 0,25 - Suy ra p chẵn  p = 2. 0,25 Trang 67
b) - Nếu p chia 3 dư 1 thì p + 8 là số lớn hơn 3 và chia hết cho 3 nên không là số nguyên tố. 0,25
- Nếu p chia 3 dư 2 thì p + 10 là số lớn hơn 3 và chia hết cho 3 nên không là số nguyên tố. 0,25
- Suy ra p chia hết cho 3, p nguyên tố nên p = 3. 0,5
C©u4 (1.5 điểm)
Gäi sè bÞ chia lµ a; sè chia lµ b (b  0) 0,5
PhÐp chia cã th-¬ng b»ng 5 sè d- lµ 12  a = 5b+12
Sè bÞ chia chia cho tæng sè chia vµ sè d-®-îc th-¬ng lµ 3 vµ sè d- lµ 18 
a = (b +12). 3 + 18 = 3b + 54 0,5
 5b + 12 = 3b + 54  b = 21  a = 117 VËy sè bÞ chia lµ 117. 0,5 Câu 5: - Hình vẽ: N A M B
Bài 5: Vẽ hình (0,25đ)
- M nằm giữa hai điểm A, B nên MA = AB - MB = 3 - 1 = 2 (cm) 0,25 - AN = AM = 2 (cm) 0,25
- A nằm giữa hai điểm N, B nên BN = AN + AB = 2 + 3 = 5 (cm). 0,25
- BN = AN + AB, AB không đổi nên BN lớn nhất khi AN lớn nhất. 0,25
- AN lớn nhất khi AM lớn nhất. 0,25
- AM lớn nhất khi AM = AB. 0,25
- Lúc đó M trùng với B và BN bằng 6(cm). 0,25 CHÚ Ý :
- Nếu HS làm cách khác mà đúng thì vẫn cho điểm tối đa theo thang điểm của ý đó
- Khi học sinh làm bài phải lý luận chặt chẽ mới cho điểm tối đa theo biểu điểm của ý đó

PHÒNG GIÁO DỤC VÀ ĐÀO TẠO
ĐỀ KHẢO SÁT HỌC SINH GIỎI LỚP 6 CẤP HUYỆN Môn: Toán – Lớp 6
Thời gian làm bài: 120 phút (Không kể thời gian giao đề) ĐỀ 19 Câu 1 (4 điểm).     
a. Tính giá trị của biểu thức 1 31 1 17 1 1 1 1 1 A = 9 −  − 4 +  + + + + ...+   31  5  2  2  5  2 6 12 930 Trang 68
b. Tính giá trị của biểu thức B biết: B2 = c(a-b)- b(a-c) và a = -50, b-c =2.
Câu 2.
(4 điểm)
a. Tìm số tự nhiên x,y biết: (2x+1)(y-3)= 12
b. Tìm số tự nhiên x biết: 2x + 2x 1
+ + 2x+2 +...+ 2x+2015 = 22019 −8 c. So sánh: 3625 và 2536 Câu 3. (3 điểm) 6n + 5 Cho phân số: p = (n N ) 3n + 2
a. Chứng minh rằng phân số p là phân số tối giản
b. Với giá trị nào của n thì phân số p có giá trị lớn nhất? tìm giá trị lớn nhất đó. Câu 4. (7,5 điểm)
1. Cho hai góc kề bù góc xOy và góc yOt, trong đó xOy =400. Gọi Om là tia phân giác của góc yOt. a. Tính góc mOx ?
b. Trên nửa mặt phẳng không chứa tia Oy và có bờ là đường thẳng chứa tia Ox, vẽ tia On
sao cho góc xOn=700 . Chứng tỏ tia Om và tia On là hai tia đối nhau
2. Vẽ đoạn thẳng AB =6cm. Lấy hai điểm C và D nằm giữa A và B sao cho AC+BD= 9cm
a. Chứng tỏ D nằm giữa A và C
b. Tính độ dài đoạn thẳng CD Câu 5. (1,5 điểm)
Tìm các số nguyên dương x, y thỏa mãn : 2x+3y= 14
---------------- Hết ----------------
PHÒNG GIÁO DỤC VÀ ĐÀO TẠO
ĐỀ KHẢO SÁT HỌC SINH GIỎI LỚP 6 CẤP HUYỆN Môn: Toán – Lớp 6
Thời gian làm bài: 120 phút (Không kể thời gian giao đề)
HƯỚNG DẪN CHẤM ĐỀ 19 Câu Nội dung Điểm Câu 1 a. 1 31  1  17  1  1 1 1 1 A = 9 −  − 4 +  + + + + ...+ =   31  5  2  2  5  2 6 12 930 1 31  1  17  1  1  31 17 17 21  17  31 − 21  17 Xet : M = 9 −  − 4 +  =    −  =   = 1 31  5  2  2  5  31  5 2 2 5  31  10  31 1 1 1 1  1 1 1 1  0.5 Xet : N = + + + ... + =  + + + ...+  2 6 12 930  2 3 . 2 4 . 3 30 31 .  0.5  1 1 1 1 1 1 1  1 30 =  + − + − + ...+ −  = 1− =  2 2 3 3 4 30 31  31 31 0.5 17 30 47
A = M + N = + = 31 31 31
b. B2= c(a-b)-b(a-c) = ca-cb-ba+bc=ca-ba=a(c-b) 0.5 Trang 69
thay a=-50, b-c=2 vào ta được B2=-50.(-2)=100 0.5
do B N nên B=10 0.5 Câu 2 a. (1,5 điểm) (2x+1)(y-3)= 12
Với x, y N  2x+1 là số lẻ. 0.25 Ta có: 12 =1.12=3.4 0.25
2x+1=1 2x=0 x=0; y-3=12  y=15 0.25
2x+1=3 2x=2 x=1; y-3=1  y=4 0.25
Vậy x=0 và y=1 hoặc x=1 và y=4 0. 5 b. (1,25 điểm) Ta có : 2x + 2 x 1
+ + 2x+2 + ...+ 2x 1+015 = 22019 − 8 x 1 2 2015 2019 0.25  2 (1+ 2 + 2 + ...+ 2 )= 2 −8
Xet : C = 1 + 21 + 22 + ... + 22015
2C = 2 + 22 + 23 + ... + 22016 2016 2016 0.25 2C C = 2 −1  C = 2 −1 0.25  2x (22016 − )
1 = 22019 − 8 = 22019 − 23 = 23 (22016 − ) 1 0.25  2x = 23 0.25  x = 3 c. (1 điểm)
3625 = (18.2)25 =1825 .225 =1825 .26 .219 0.25
2536 =2525.2511= 2525.522= 2525.53.519 0.25
ta có: 53=125, 26=64,  53>26 2525>1825; 519>219 0.25
Vậy 2525.53.519 >1825 .26 .219 hay 3625 <2536 0. 5
Câu 3 a. Gọi d là UC của 6n+5 và 3n+2 ta có: 6n + 5d và 3n + 2 d 0.25 3n + 2d → 3n ( 2 + 2) h d ay 6n + 4d 0.25
→ 6n + 5 - (6n + 4)d →1d 0.25 0.25 d = 1 0.25 + Vậy phân số 6n 5 p =
(n N ) là phân số tối giản 3n + 2 6n + 5 6n + 4 +1 1 0.5 b. Ta có p = = = 2 + 3n + 2 3n + 2 3n + 2 1
p đạt giá trị lớn nhất khi
đạt giá trị lớn nhất, khi đó 3n+2 đạt giá trị n + 0.5 3 2 nhỏ nhất
vì 3n + 2  2 nên 3n+2 nhỏ nhất bằng 2 khi 3n=0 hay n=0 0.5
Vậy với n=0 thì p đạt giá trị lớn nhất là 2+1/2=3/2 0.25 Câu 4 1(4 điểm). Vẽ hình
a. Ta có xOy + yOt=1800 0,5 (Vì 2 góc kề bù) 0.25 Thay xOy = 400 ta có: Trang 70 400+yOt= 1800 suy ra yOt=1400 0. 5 1 1
Ta có: Om là tia phân giác của tOy nên 0 0
tOm = tOy = 140 = 70 2 2
Vì 2 góc xOy và yOt kề bù nên Ox và Ot là hai tia đối nhau 0.25
suy ra tOm và mOx là hai góc kề bù tOm + mOx = 1800 0. 5 700 + mOx = 1800  mOx = 1800-700= 1100 0.5
b. Ta có mOx+ xOn = 1100+ 700=1800 
mOx và xOn là hai góc bù nhau (1)
- Do Om và Oy cùng thuộc nửa mp có bờ là đường thẳng chứa tia Ox; - 0.5
Lại có On và Oy nằm trên hai nửa mặt phẳng đối nhau có bờ là đường thẳng chứa tia Ox
nên: Om và On nằm trên hai nửa mặt phẳng đối nhau có bờ là đường thẳng chứa tia Ox
 mOx và xOn là hai góc kề nhau (2)
Từ (1) và (2) suy ra mOx và xOn là hai góc kề bù. 0.5 0.5 2. (3,5đ) Vẽ A D C B hình 0.5
- Vì D nằm giữa A và B nên: AD+DB=AB
Thay AB= 6cm ta có AD+DB = 6 (cm) 0.25 Lại có AC+DB=9cm (gt) 0.25
 AD+DB< AC+DB hay AD0.25
- Mà D và C cùng nằm giữa A và B hay D,C cùng thuộc tia AB (2) 0.25
Từ (1) và (2) suy ra D nằm giữa A và C 0.25
b, Vì D nằm giữa A và C suy ra: AD+DC= AC 0.5 Lại có AC+BD= 9 0.25
nên AD+DC+BD = 9 hay (AD+DB)+DC =9 Thay (AD+DB)=6 0.5 ta có 6+DC=9 vậy DC= 3(cm) 0.5 Câu 5 Xét 2x+5y= 14 Ta có: 14 ; 2 2
x2 → 5y2 0.25 0.25 Do (5,2)=1 nên y2
Ta có 3y<14  y<14 :5  y  2 0.25
Mà y là số nguyên dương và y2 nên y = 2 0.25 0.25
ta có 2x+5.2=14 2x=4 x=2 vậy x=2, y=2 0.25 CHÚ Ý :
- Nếu HS làm cách khác mà đúng thì vẫn cho điểm tối đa theo thang điểm của ý đó Trang 71
- Khi học sinh làm bài phải lý luận chặt chẽ mới cho điểm tối đa theo biểu điểm của ý đó
---------------- Hết ----------------
PHÒNG GIÁO DỤC VÀ ĐÀO TẠO
ĐỀ KHẢO SÁT HỌC SINH GIỎI LỚP 6 CẤP HUYỆN Môn: Toán – Lớp 6
Thời gian làm bài: 120 phút (Không kể thời gian giao đề) ĐỀ 20
Bài 1 (3,0 điểm)
Cho tổng A = 1 + 32 + 34 + 36 +…+ 32008
Tính giá trị biểu thức: B = 8A - 32010
Bài 2 (4,0 điểm)
Cho A = 1.4.7.10..…58 + 3.12.21.30…..174
a. Tìm chữ số tận cùng của A.
b. Chứng tỏ rằng A chia hết cho 377.
Bài 3 (4,0 điểm)
Tìm số tự nhiên x biết:
a. x + (x + 1) + (x + 2) + …+ (x + 99) = 5450. b. 3.(5x - 1) - 2 = 70.
c. 2x + 2x + 1 + 2x + 2 = 960 - 2x + 3
Bài 4 (4,0 điểm)
a. Tìm số tự nhiên có hai chữ số khác nhau. Biết rằng: hai chữ số của số đó đều là số nguyên tố.
Tích của số đó với các chữ số của nó là số có 3 chữ số giống nhau được tạo thành từ chữ số hàng đơn vị của số đó.
b. Cho p là số nguyên tố (p > 3) và 2p + 1 cũng là số nguyên tố. Hỏi 4p + 1 là số nguyên tố hay hợp số? Vì sao?
Bài 5 (5,0 điểm)
Cho n đường thẳng trong đó bất cứ hai đường thẳng nào cũng cắt nhau, không có ba đường thẳng
nào cùng đi qua một điểm.
a. Biết rằng số giao điểm của các đường thẳng đó là 1128. Tính n.
b. Số giao điểm của các đường thẳng đó có thể là 2017 được không? Vì sao?
………….. Hết ………… Trang 72
PHÒNG GIÁO DỤC VÀ ĐÀO TẠO
ĐỀ KHẢO SÁT HỌC SINH GIỎI LỚP 6 CẤP HUYỆN Môn: Toán – Lớp 6
Thời gian làm bài: 120 phút (Không kể thời gian giao đề)
HƯỚNG DẪN CHẤM ĐỀ 20 Bài 1 Hướng dẫn giải (3,0điểm) Điểm
A = 1 + 32 + 34 + 36 +…+ 32008
9A = 32 + 34 + 36 + 38 +… + 32010 1,0 Tính được 8A = 32010 - 1 1,0
B = 8A - 32010 = 32010 - 1 - 32010 = -1 1,0 Bài 2 (4,0điểm) a,
Tìm chữ số tận cùng của A
(2,0 điểm) - Tìm được chữ số tận cùng của tích B = 1.4.7.10…58 là 0 0,75
- Tìm được chữ số tận cùng của tích C = 3.12.21.30…174 là 0 0,75
- Tìm được và kết luận chữ số tận cùng của A là 0 0,5 b,
Chứng tỏ rằng A chia hết cho 377
(2,0 điểm) - Nhận xét 377 = 13.29 0,5
- Tìm được quy luật của các thừa số trong tích B là các số tự nhiên chia
3 dư 1, nên B chứa thừa số 13. Do đó B = 1.4.7.10.13…58 0,5 B = 1.4.7.10.13…29.2 Suy ra B chia hết cho 377
- Tìm được quy luật của các thừa số trong tích C là các số tự nhiên chia
9 dư 3, nên C chứa thừa số 39. Do đó C = 3.12.21.30.39…174 0,5
C = 3.12.21.30.(3.13)…(6.29) Suy ra C chia hết cho 377
- Kết luận A chia hết cho 377 0,5 Bài 3 (4,0điểm) a,
x + (x + 1) + (x + 2) + …+ (x + 99) = 5450.
(1,5 điểm) 100x + (1 + 2+ 3+ …+ 99) = 5450 0,5
Lí luận tính tổng: 1 + 2+ 3+ …+ 99 = 4950 0,5 khi đó 100x + 4950 = 5450 100x = 500 0,25 x = 5 0,25 b,
(1,5 điểm) 3.(5x - 1) - 2 = 70. 3.(5x - 1) = 70 + 2 3.(5x - 1) = 72 0,5 5x - 1 = 72 : 3 5x - 1 = 24 0,5 5x = 25 5x = 52 0,5 x = 2 Trang 73 c,
2x + 2x + 1 + 2x + 2 = 960 - 2x + 3
(1,0 điểm) 2x (1 + 2 + 22 + 23) = 960 2x .15 = 960 0,5 2x = 960: 15 2x = 64 2x = 26 0,5 x = 6 Bài 4 (4,0điểm) a,
Tìm số tự nhiên có hai chữ số khác nhau… (2,0 điểm)
- Gọi số cần tìm là ab , (điều kiện của a, b…) 0,25
- Theo đề bài ta có ab .a.b = bbb Suy ra ab .a.b = 111.b 0,75 Hay ab .a = 111 Mà 111 = 3.37
Trong đó: 3 là số nguyên tố; 7 là số nguyên tố; 3 7 thỏa mãn đề bài 0,75 nên ab = 37
Kết luận số cần tìm là 37 0,25 b,
Cho p là số nguyên tố (p > 3) và 2p + 1 cũng là số nguyên tố. Hỏi 4p + 1 (2,0 điểm)
là số nguyên tố hay hợp số? Vì sao?
Vì p là số nguyên tố lớn hơn 3 nên p có dạng 3k + 1 hoặc 3k + 2 (với k  0,5 N, k  1)
Nếu p = 3k +1 thì 2p + 1 = 2(3k + 1) + 1 = 3(2k + 1) 0,75
và lí luận chỉ ra 2p + 1 là hợp số, trái với đề bài
Do đó p = 3k + 2 khi đó 4p + 1 = 4(3k + 2) + 1 = 3(4k + 3) 0,75
và lí luận chỉ ra 4p + 1 là hợp số Kl….. Bài 5 (5,0điểm ) a,
Với n đường thẳng trong đó bất cứ hai đường thẳng nào cũng cắt nhau,
(3,0 điểm) không có ba đường thẳng nào cùng đi qua một điểm. Số giao điểm được
xác định như sau: Chọn một đường thẳng, đường thẳng này cắt n - 1
đường thẳng còn lại tạo ra n
- 1 giao điểm, làm như vậy với n đường thẳng ta được n.(n 1,5
- 1) giao điểm. Nhưng mỗi giao điểm đã được tính 2
lần, nên số giao điểm là n.(n - 1):2 giao điểm
- Khi số giao điểm là 1128 ta có: n(n - 1):2= 1128 1,0
- Lý luận tìm được n = 48 0,5 b,
- Giả sử số giao điểm bằng 2017 (2,0 điểm) 1,0
- Áp dụng kết quả câu a ta có n(n - 1):2 = 2017
- Lý luận tìm ra điều vô lý 1,0
- Kết luận: Số giao điểm không thể bằng 2017 Chú ý:
- Học sinh có cách giải khác đúng cho điểm tương đương.
- Nếu bài hình phần trên sai, thì vẫn chấm điểm phần dưới Trang 74
- Bài 2. Câu a chỉ ra được chữ số tận cùng là cho điểm tối đa
- Bài 5. Lí luận không chính xác thì tùy từng ý trừ điểm Trang 75